Location via proxy:   [ UP ]  
[Report a bug]   [Manage cookies]                

AMC 10 真题集 2000-2021

Download as pdf or txt
Download as pdf or txt
You are on page 1of 200

AoPS Community 2000 AMC 10

AMC 10 2000
www.artofproblemsolving.com/community/c4798
by 4everwise, worthawholebean, IsabeltheCat, Mrdavid445, catcurio, rrusczyk

1 In the year 2001, the United States will host the International Mathematical Olympiad. Let I, M ,
and O be distinct positive integers such that the product I · M · O = 2001. What’s the largest
possible value of the sum I + M + O?
(A) 23 (B) 55 (C) 99 (D) 111 (E) 671

2 2000(20002000 ) =
(A) 20002001 (B) 40002000 (C) 20004000 (D) 4, 000, 0002000 (E) 20004,000,000

3 Each day, Jenny ate 20% of the jellybeans that were in her jar at the beginning of the day. At
the end of the second day, 32 remained. How many jellybeans were in the jar originally?
(A) 40 (B) 50 (C) 55 (D) 60 (E) 75

4 Chandra pays an on-line service provider a fixed monthly fee plus an hourly charge for connect
time. Her December bill was $12.48, but in January her bill was $17.54 because she used twice
as much connect time as in December. What is the fixed monthly fee?
(A)$2.53 (B)$5.06 (C)$6.24 (D)$7.42 (E)$8.77

5 Points M and N are the midpoints of sides P A and P B of 4P AB. As P moves along a line
that is parallel to side AB, how many of the four quantities listed below change?
(A) the length of the segmentM N
(B) the perimeter of 4P AB
(C) the area of 4P AB
(D) the area of trapezoidABN M

M N

A B

© 2019 AoPS Incorporated 1


AoPS Community 2000 AMC 10

(A) 0 (B) 1 (C) 2 (D) 3 (E) 4

6 The Fibonacci Sequence 1, 1, 2, 3, 5, 8, 13, 21, . . . starts with two 1s and each term afterwards
is the sum of its predecessors. Which one of the ten digits is the last to appear in the units
position of a number in the Fibonacci Sequence?
(A) 0 (B) 4 (C) 6 (D) 7 (E) 9

7 In rectangle ABCD, AD = 1, P is on AB, and DB and DP trisect ∠ADC. What is the perimeter
of 4BDP ?

A P B

D C


3

4 3
√ √
3+3 5

5 3
(A) 3 + 3 (B) 2 + 3 (C) 2 + 2 2 (D) 2 (E) 2 + 3

8 At Olympic High School, 52 of the freshmen and 45 of the sophomores took the AMC-10. Given
that the number of freshmen and sophomore contestants was the same, which of the follow-
ing must be true?
(A) There are five times as many sophomores as freshmen. (B) There are twice as many sopho-
mores as freshmen. (C) There are as many freshmen as sophomores. (D) There are twice as
many freshmen as sophomores. (E) There are five times as many freshmen as sophomores.

9 If |x − 2| = p, where x < 2, then x − p =


(A) −2 (B) 2 (C) 2 − 2p (D) 2p − 2 (E) |2p − 2|

10 The sides of a triangle with positive area have lengths 4, 6, and x. The sides of a second triangle
with positive area have lengths 4, 6, and y. What is the smallest positive number that is not a
possible value of |x − y|?
(A) 2 (B) 4 (C) 6 (D) 8 (E) 10

11 Two different prime numbers between 4 and 18 are chosen. When their sum is subtracted from
their product, which of the following numbers could be obtained?
(A) 21 (B) 60 (C) 119 (D) 180 (E) 231

12 Figures 0, 1, 2, and 3 consist of 1, 5, 13, and 25 nonoverlapping squares, respectively. If the


pattern were continued, how many nonoverlapping squares would there be in figure 100?

© 2019 AoPS Incorporated 2


AoPS Community 2000 AMC 10

Figure Figure Figure Figure


0 1 2 3

(A) 10401 (B) 19801 (C) 20201 (D) 39801 (E) 40801

13 There are 5 yellow pegs, 4 red pegs, 3 green pegs, 2 blue pegs, and 1 orange peg on a triangular
peg board. In how many ways can the pegs be placed so that no (horizontal) row or (vertical)
column contains two pegs of the same color?

(A) 0 (B) 1 (C) 5! · 4! · 3! · 2! · 1! (D) 15!


5!·4!·3!·2!·1! (E) 15!

14 Mrs. Walter gave an exam in a mathematics class of five students. She entered the scores in
random order into a spreadsheet, which recalculated the class average after each score was
entered. Mrs. Walter noticed that after each score was entered, the average was always an
integer. The scores (listed in ascending order) were 71, 76, 80, 82, and 91. What was the last
score Mrs. Walter entered?
(A) 71 (B) 76 (C) 80 (D) 82 (E) 91

15 Two non-zero real numbers, a and b, satisfy ab = a − b. Which of the following is a possible
value of ab + ab − ab?
(A) − 2 (B) − 12 (C) 13 (D) 21 (E) 2

16 The diagram show 28 lattice points, each one unit from its nearest neighbors. Segment AB
meets segment CD at E. Find the length of segment AE.

© 2019 AoPS Incorporated 3


AoPS Community 2000 AMC 10

A
C
E

D B

√ √ √ √ √
(A) 4 5
3 (B) 5 5
3 (C) 12 5
7 (D) 2 5 (E) 5 65
9

17 Boris has an incredible coin changing machine. When he puts in a quarter, it returns five nick-
els; when he puts in a nickel, it returns five pennies; and when he puts in a penny, it returns five
quarters. Boris starts with just one penny. Which of the following amounts could Boris have
after using the machine repeatedly?
(A) $3.63 (B) $5.13 (C) $6.30 (D) $7.45 (E) $9.07

18 Charlyn walks completely around the boundary of a square whose sides are each 5 km long.
From any point on her path she can see exactly 1 km horizontally in all directions. What is the
area of the region consisting of all points Charlyn can see during her walk, expressed in square
kilometers and rounded to the nearest whole number?
(A) 24 (B) 27 (C) 39 (D) 40 (E) 42

19 Through a point on the hypotenuse of a right triangle, lines are drawn parallel to the legs of the
triangle so that the trangle is divided into a square and two smaller right triangles. The area of
one of the two small right triangles is m times the area of the square. The ratio of the area of
the other small right triangle to the area of the square is
(A) 2m+1
1
(B) m (C) 1 − m (D) 4m
1
(E) 8m
1
2

20 Let A, M , and C be nonnegative integers such that A + M + C = 10. What is the maximum
value of A · M · C + A · M + M · C + C · A?
(A) 49 (B) 59 (C) 69 (D) 79 (E) 89

21 If all alligators are ferocious creatures and some creepy crawlers are alligators, which state-
ment(s) must be true?

I. All alligators are creepy crawlers.


II. Some ferocious creatures are creepy crawlers.
III. Some alligators are not creepy crawlers.
(A) I only (B) II only (C) III only (D) II and III only (E) None must be true

22 One morning each member of Angela’s family drank an 8-ounce mixture of coffee with milk.
The amounts of coffee and milk varied from cup to cup, but were never zero. Angela drank a

© 2019 AoPS Incorporated 4


AoPS Community 2000 AMC 10

quarter of the total amount of milk and a sixth of the total amount of coffee. How many people
are in the family?
(A) 3 (B) 4 (C) 5 (D) 6 (E) 7

23 When the mean, median, and mode of the list 10, 2, 5, 2, 4, 2, x are arranged in increasing order,
they form a non-constant arithmetic progression. What is the sum of all possible real values
of x?
(A) 3 (B) 6 (C) 9 (D) 17 (E) 20

Let f be a function for which f x3 = x2 + x + 1. Find the sum of all values of z for which

24
f (3z) = 7.
(A) − 31 (B) − 19 (C) 0 (D) 95 (E) 53

25 In year N , the 300th day of the year is a Tuesday. In year N + 1, the 200th day is also a Tuesday.
On what day of the week did the 100th of year N − 1 occur?
(A) Thursday (B) Friday (C) Saturday (D) Sunday (E) Monday


These problems are copyright © Mathematical Association of America (http://maa.org).

© 2019 AoPS Incorporated 5


Art of Problem Solving is an ACS WASC Accredited School.
AoPS Community 2001 AMC 10

AMC 10 2001
www.artofproblemsolving.com/community/c4799
by #H34N1, Silverfalcon, mysmartmouth, worthawholebean, rrusczyk

1 The median of the list

n, n + 3, n + 4, n + 5, n + 6, n + 8, n + 10, n + 12, n + 15

is 10. What is the mean?


(A) 4 (B) 6 (C) 7 (D) 10 (E) 11

2 A number x is 2 more than the product of its reciprocal and its additive inverse. In which interval
does the number lie?
(A) − 4 ≤ x ≤ −2 (B) − 2 < x ≤ 0 (C) 0 < x ≤ 2 (D) 2 < x ≤ 4 (E) 4 < x ≤ 6

3 The sum of two numbers is S. Suppose 3 is added to each number and then each of the re-
sulting numbers is doubled. What is the sum of the final two numbers?
(A) 2S + 3 (B) 3S + 2 (C) 3S + 6 (D) 2S + 6 (E) 2S + 12

4 What is the maximum number of possible points of intersection of a circle and a triangle?
(A) 2 (B) 3 (C) 4 (D) 5 (E) 6

5 How many of the twelve pentominoes pictured below have at least one line of symmetry?
(A) 3 (B) 4 (C) 5 (D) 6 (E) 7

6 Let P (n) and S(n) denote the product and the sum, respectively, of the digits of the integer
n. For example, P (23) = 6 and S(23) = 5. Suppose N is a two-digit number such that N =

© 2019 AoPS Incorporated 1


AoPS Community 2001 AMC 10

P (N ) + S(N ). What is the units digit of N ?


(A) 2 (B) 3 (C) 6 (D) 8 (E) 9

7 When the decimal point of a certain positive decimal number is moved four places to the right,
the new number is four times the reciprocal of the original number. What is the original num-
ber?
(A) 0.0002 (B) 0.002 (C) 0.02 (D) 0.2 (E) 2

8 Wanda, Darren, Beatrice, and Chi are tutors in the school math lab. Their schedule is as follows:
Darren works every third school day, Wanda works every fourth school day, Beatrice works
every sixth school day, and Chi works every seventh school day. Today they are all working in
the math lab. In how many school days from today will they next be together tutoring in the
lab?
(A) 42 (B) 84 (C) 126 (D) 178 (E) 252

9 The state income tax where Kristin lives is levied at the rate of p% of the first $28000 of annual
income plus (p + 2)% of any amount above $28000. Kristin noticed that the state income tax
she paid amounted to (p + 0.25)% of her annual income. What was her annual income?
(A) $28000 (B) $32000 (C) $35000 (D) $42, 000 (E) $56000

10 If x, y, and z are positive with xy = 24, xz = 48, and yz = 72, then x + y + z is


(A) 18 (B) 19 (C) 20 (D) 22 (E) 24

11 Consider the dark square in an array of unit squares, part of which is shown. The rst ring of
squares around this center square contains 8 unit squares. The second ring contains 16 unit
squares. If we continue this process, the number of unit squares in the 100th ring is
(A) 396 (B) 404 (C) 800 (D) 10,000 (E) 10,404

12 Suppose that n is the product of three consecutive integers and that n is divisible by 7. Which
of the following is not necessarily a divisor of n?
(A) 6 (B) 14 (C) 21 (D) 28 (E) 42

13 A telephone number has the form ABC − DEF − GHIJ, where each letter represents a dif-

© 2019 AoPS Incorporated 2


AoPS Community 2001 AMC 10

ferent digit. The digits in each part of the numbers are in decreasing order; that is, A > B > C,
D > E > F , and G > H > I > J. Furthermore, D, E, and F are consecutive even digits; G, H,
I, and J are consecutive odd digits; and A + B + C = 9. Find A.
(A) 4 (B) 5 (C) 6 (D) 7 (E) 8

14 A charity sells 140 benefit tickets for a total of $2001. Some tickets sell for full price (a whole
dollar amount), and the rest sells for half price. How much money is raised by the full-price
tickets?
(A) $782 (B) $986 (C) $1158 (D) $1219 (E) $1449

15 A street has parallel curbs 40 feet apart. A crosswalk bounded by two parallel stripes crosses
the street at an angle. The length of the curb between the stripes is 15 feet and each stripe is
50 feet long. Find the distance, in feet, between the stripes.
(A) 9 (B) 10 (C) 12 (D) 15 (E) 25

16 The mean of three numbers is 10 more than the least of the numbers and 15 less than the
greatest. The median of the three numbers is 5. What is their sum?
(A) 5 (B) 20 (C) 25 (D) 30 (E) 36

17 Which of the cones listed below can be formed from a 252◦ sector of a circle of radius 10 by
aligning the two straight sides?

252◦
10

(A) (B) (C) (D) (E)

18 The plane is tiled by congruent squares and congruent pentagons as indicated. The percent

© 2019 AoPS Incorporated 3


AoPS Community 2001 AMC 10

of the plane that is enclosed by the pentagons is closest to


(A) 50 (B) 52 (C) 54 (D) 56 (E) 58

19 Pat wants to buy four donuts from an ample supply of three types of donuts: glazed, chocolate,
and powdered. How many different selections are possible?
(A) 6 (B) 9 (C) 12 (D) 15 (E) 18

20 A regular octagon is formed by cutting an isosceles right triangle from each of the corners of
a square with sides of length
√ 2000. What is the length
√ of each side of the octagon?

(A) 13 (2000) (B) 2000( 2 − 1) (C) 2000(2 − 2) (D) 1000 (E) 1000 2

21 A right circular cylinder with its diameter equal to its height is inscribed in a right circular cone.
The cone has diameter 10 and altitude 12, and the axes of the cylinder and cone coincide. Find
the radius of the cylinder.
(A) 83 30
(B) 11 (C) 3 (D) 25
8 (E) 27

22 In the magic square shown, the sums of the numbers in each row, column, and diagonal are
the same. Five of these numbers are represented by v, w, x, y, and z. Find y + z.
(A) 43 (B) 44 (C) 45 (D) 46 (E) 47

v 24 w

18 x y

25 z 21

23 A box contains exactly five chips, three red and two white. Chips are randomly removed one at
a time without replacement until all the red chips are drawn or all the white chips are drawn.
What is the probability that the last chip drawn is white?

© 2019 AoPS Incorporated 4


AoPS Community 2001 AMC 10
3 2 1 3 7
(A) (B) (C) (D) (E)
10 5 2 5 10

24 In trapezoid ABCD, AB and CD are perpendicular to AD, with AB + CD = BC, AB < CD,
and AD = 7. What is AB · CD?
(A) 12 (B) 12.25 (C) 12.5 (D) 12.75 (E) 13

25 How many positive integers not exceeding 2001 are multiples of 3 or 4 but not 5?
(A) 768 (B) 801 (C) 934 (D) 1067 (E) 1167


These problems are copyright © Mathematical Association of America (http://maa.org).

© 2019 AoPS Incorporated 5


Art of Problem Solving is an ACS WASC Accredited School.
AoPS Community 2002 AMC 10

AMC 10 2002
www.artofproblemsolving.com/community/c4800
by worthawholebean, redcomet46, djmathman, rrusczyk

– A

102000 + 102002
1 The ratio is closest to which of the following numbers?
102001 + 102001
(A) 0.1 (B) 0.2 (C) 1 (D) 5 (E) 10

2 For the nonzero numbers a, b, c, define


a b c
(a, b, c) = + + .
b c a
Find (2, 12, 9).
(A) 4 (B) 5 (C) 6 (D) 7 (E) 8

3 According to the standard convention for exponentiation,


 
22
22 2( )
22 =2 = 216 = 65,536.
If the order in which the exponentiations are performed is changed, how many other values
are possible?
(A) 0 (B) 1 (C) 2 (D) 3 (E) 4

4 For how many positive integers m does there exist at least one positive integer n such that
m · n ≤ m + n?
(A) 4 (B) 6 (C) 9 (D) 12 (E) infinitely many

5 Each of the small circles in the figure has radius one. The innermost circle is tangent to the
six circles that surround it, and each of those circles is tangent to the large circle and to its
small-circle neighbors. Find the area of the shaded region.

(A) π (B) 1.5π (C) 2π (D) 3π (E) 3.5π

© 2019 AoPS Incorporated 1


AoPS Community 2002 AMC 10

6 Cindy was asked by her teacher to subtract 3 from a certain number and then divide the result
by 9. Instead, she subtracted 9 and then divided the result by 3, giving an answer of 43. What
would her answer have been had she worked the problem correctly?
(A) 15 (B) 34 (C) 43 (D) 51 (E) 138

7 If an arc of 45◦ on circle A has the same length as an arc of 30◦ on circle B, then the ratio of
the area of circle A to the area of circle B is
(A) 49 (B) 32 (C) 56 (D) 32 (E) 94

8 Betsy designed a flag using blue triangles, small white squares, and a red center square, as
shown. Let B be the total area of the blue triangles, W the total area of the white squares, and
R the area of the red square. Which of the following is correct?

(A) B = W (B) W = R (C) B = R (D) 3B = 2R (E) 2R = W

9 Suppose A, B, and C are three numbers for which 1001C −2002A = 4004 and 1001B +3003A =
5005.The average of the three numbers A, B, and C is
(A) 1 (B) 3 (C) 6 (D) 9 (E) not uniquely determined

10 Compute the sum of all the roots of (2x + 3)(x − 4) + (2x + 3)(x − 6) = 0.
(A) 7/2 (B) 4 (C) 5 (D) 7 (E) 13

11 Jamal wants to store 30 computer files on floppy disks, each of which has a capacity of 1.44
megabytes (MB). Three of his files require 0.8 MB of memory each, 12 more require 0.7 MB
each, and the remaining 15 require 0.4 MB each. No file can be split between floppy disks.
What is the minimal number of floppy disks that will hold all the files?
(A) 12 (B) 13 (C) 14 (D) 15 (E) 16

12 Mr. Earl E. Bird leaves his house for work at exactly 8:00 A.M. every morning. When he averages
40 miles per hour, he arrives at his workplace three minutes late. When he averages 60 miles
per hour, he arrives three minutes early. At what average speed, in miles per hour, should Mr.
Bird drive to arrive at his workplace precisely on time?
(A) 45 (B) 48 (C) 50 (D) 55 (E) 58

© 2019 AoPS Incorporated 2


AoPS Community 2002 AMC 10

13 The sides of a triangle have lengths of 15, 20, and 25. Find the length of the shortest altitude.
(A) 6 (B) 12 (C) 12.5 (D) 13 (E) 15

14 Both roots of the quadratic equation x2 − 63x + k = 0 are prime numbers. The number of
possible values of k is
(A) 0 (B) 1 (C) 2 (D) 3 (E) more than four

15 The digits 1, 2, 3, 4, 5, 6, 7, and 9 are used to form four two-digit prime numbers, with each digit
used exactly once. What is the sum of these four primes?
(A) 150 (B) 160 (C) 170 (D) 180 (E) 190

16 If a + 1 = b + 2 = c + 3 = d + 4 = a + b + c + d + 5, then a + b + c + d is
(A) − 5 (B) − 10/3 (C) − 7/3 (D) 5/3 (E) 5

17 Sarah pours four ounces of coffee into an eight-ounce cup and four ounces of cream into a
second cup of the same size. She then transfers half the coffee from the first cup to the second
and, after stirring thoroughly, transfers half the liquid in the second cup back to the first. What
fraction of the liquid in the first cup is now cream?
(A) 1/4 (B) 1/3 (C) 3/8 (D) 2/5 (E) 1/2

18 A 3 × 3 × 3 cube is formed by gluing together 27 standard cubical dice. (On a standard die, the
sum of the numbers on any pair of opposite faces is 7.) The smallest possible sum of all the
numbers showing on the surface of the 3 × 3 × 3 cube is
(A) 60 (B) 72 (C) 84 (D) 90 (E) 96

19 Spot’s doghouse has a regular hexagonal base that measures one yard on each side. He is
tethered to a vertex with a two-yard rope. What is the area, in square yards, of the region outside
of the doghouse that Spot can reach?
(A) 2π/3 (B) 2π (C) 5π/2 (D) 8π/3 (E) 3π

20 Points A, B, C, D, E and F lie, in that order, on AF , dividing it into five segments, each of length
1. Point G is not on line AF . Point H lies on GD, and point J lies on GF . The line segments
HC, JE, and AG are parallel. Find HC/JE.
(A) 5/4 (B) 4/3 (C) 3/2 (D) 5/3 (E) 2

21 The mean, median, unique mode, and range of a collection of eight integers are all equal to 8.
The largest integer that can be an element of this collection is
(A) 11 (B) 12 (C) 13 (D) 14 (E) 15

22 A sit of tiles numbered 1 through 100 is modified repeatedly by the following operation: remove
all tiles numbered with a perfect square, and renumber the remaining tiles consecutively start-

© 2019 AoPS Incorporated 3


AoPS Community 2002 AMC 10

ing with 1. How many times must the operation be performed to reduce the number of tiles in
the set to one?
(A) 10 (B) 11 (C) 18 (D) 19 (E) 20

23 Points A, B, C and D lie on a line, in that order, with AB = CD and BC = 12. Point E is not on
the line, and BE = CE = 10. The perimeter of 4AED is twice the perimeter of 4BEC. Find
AB.
(A) 15/2 (B) 8 (C) 17/2 (D) 9 (E) 19/2

24 Tina randomly selects two distinct numbers from the set {1, 2, 3, 4, 5} and Sergio randomly
selects a number from the set {1, 2, ..., 10}. The probability that Sergio’s number is larger than
the sum of the two numbers chosen by Tina is
(A) 2/5 (B) 9/20 (C) 1/2 (D) 11/20 (E) 24/25

25 In trapezoid ABCD with bases AB and CD, we have AB = 52, BC = 12, CD = 39, and
DA = 5. The area of ABCD is

D 39 C

5 12

A 52 B

(A) 182 (B) 195 (C) 210 (D) 234 (E) 260

– B
2001 2003
1 The ratio 2 62002
·3
is
(A) 61 (B) 31 (C) 1
2 (D) 2
3 (E) 3
2

2 For the nonzero numbers a, b, and c, define


abc
(a, b, c) = .
a+b+c
Find (2, 4, 6).
(A) 1 (B) 2 (C) 4 (D) 6 (E) 24

3 The arithmetic mean of the nine numbers in the set {9, 99, 999, 9999, ..., 999999999} is a 9-digit
number M , all of whose digits are distinct. The number M does not contain the digit
(A) 0 (B) 2 (C) 4 (D) 6 (E) 8

© 2019 AoPS Incorporated 4


AoPS Community 2002 AMC 10

4 What is the value of


(3x − 2)(4x + 1) − (3x − 2)4x + 1
when x = 4?
(A) 0 (B) 1 (C) 10 (D) 11 (E) 12

5 Circles of radius 2 and 3 are externally tangent and are circumscribed by a third circle, as shown
in the figure. Find the area of the shaded region.

3 2

(A) 3π (B) 4π (C) 6π (D) 9π (E) 12π

6 For how many positive integers n is n2 − 3n + 2 a prime number?


(A) none (B) one (C) two (D) more than two, but finitely many
(E) infinitely many

7 Let n be a positive integer such that 12 + 13 + 17 + n1 is an integer. Which of the following statements
is not true?
(A) 2 divides n (B) 3 divides n (C) 6 divides n (D) 7 divides n
(E) n > 84

8 Suppose July of year N has five Mondays. Which of the following must occur five times in
August of year N ? (Note: Both months have 31 days.)
(A) Monday (B) Tuesday (C) Wednesday (D) Thursday (E) Friday

9 Using the letters A, M , O, S, and U , we can form 120 five-letter ”words”. If these ”words” are
arranged in alphabetical order, then the ”word” U SAM O occupies position
(A) 112 (B) 113 (C) 114 (D) 115 (E) 116

10 Suppose that a and b are are nonzero real numbers, and that the equation x2 + ax + b = 0 has
solutions a and b. Then the pair (a, b) is

© 2019 AoPS Incorporated 5


AoPS Community 2002 AMC 10

(A) (−2, 1) (B) (−1, 2) (C) (1, −2) (D) (2, −1) (E) (4, 4)

11 The product of three consecutive positive integers is 8 times their sum. What is the sum of
their squares?
(A) 50 (B) 77 (C) 110 (D) 149 (E) 194

12 For which of the following values of k does the equation x−1


x−2 = x−k
x−6 have no solution for x?
(A) 1 (B) 2 (C) 3 (D) 4 (E) 5

13 Find the value(s) of x such that 8xy − 12y + 2x − 3 = 0 is true for all values of y.
(A) 32 (B) 23 or − 14 (C) − 23 or − 14 (D) 32 (E) − 23 or − 14

14 The number 2564 · 6425 is the square of a positive integer N . In decimal representation, the sum
of the digits of N is
(A) 7 (B) 14 (C) 21 (D) 28 (E) 35

15 The positive integers A, B, A − B, and A + B are all prime numbers. The sum of these four
primes is
(A) even (B) divisible by 3 (C) divisible by 5 (D) divisible by 7
(E) prime

16 For how many integers n is 20−n


n
the square of an integer?
(A) 1 (B) 2 (C) 3 (D) 4 (E) 10

17 A regular√octagon ABCDEF
√ GH has sides
√ of length two.
√ Find the area√of 4ADG.
(A) 4 + 2 2 (B) 6 + 2 (C) 4 + 3 2 (D) 3 + 4 2 (E) 8 + 2

18 Four distinct circles are drawn in a plane. What is the maximum number of points where at
least two of the circles intersect?
(A) 8 (B) 9 (C) 10 (D) 12 (E) 16

19 Suppose that {an } is an arithmetic sequence with

a1 + a2 + · · · + a100 = 100 and a101 + a102 + · · · + a200 = 200.

What is the value of a2 − a1 ?


(A) 0.0001 (B) 0.001 (C) 0.01 (D) 0.1 (E) 1

20 Let a, b, and c be real numbers such that a − 7b + 8c = 4 and 8a + 4b − c = 7. Then a2 − b2 + c2 is


(A) 0 (B) 1 (C) 4 (D) 7 (E) 8

© 2019 AoPS Incorporated 6


AoPS Community 2002 AMC 10

21 Andy’s lawn has twice as much area as Beth’s lawn and three times as much area as Carlos’
lawn. Carlos’ lawn mower cuts half as fast as Beth’s mower and one third as fast as Andy’s
mower. If they all start to mow their lawns at the same time, who will finish first?
(A) Andy (B) Beth (C) Carlos (D) Andy and Carlos tie for first. (E) All three tie.

22 Let 4XOY be a right-angled triangle with m∠XOY = 90◦ . Let M and N be the midpoints of
legs OX and OY , respectively. Given that XN = 19 and Y M = 22, find XY .
(A) 24 (B) 26 (C) 28 (D) 30 (E) 32

23 Let {ak } be a sequence of integers such that a1 = 1 and am+n = am + an + mn, for all positive
integers m and n. Then a12 is
(A) 45 (B) 56 (C) 67 (D) 78 (E) 89

24 Riders on a Ferris wheel travel in a circle in a vertical plane. A particular wheel has radius 20
feet and revolves at the constant rate of one revolution per minute. How many seconds does it
take a rider to travel from the bottom of the wheel to a point 10 vertical feet above the bottom?
(A) 5 (B) 6 (C) 7.5 (D) 10 (E) 15

25 When 15 is appended to a list of integers, the mean is increased by 2. When 1 is appended to


the enlarged list, the mean of the enlarged list is decreased by 1. How many integers were in
the original list?
(A) 4 (B) 5 (C) 6 (D) 7 (E) 8

– P

-1 This test and the matching AMC 12P were developed for the use of a group of Taiwan schools,
in early January of 2002. When Taiwan had taken the contests, the AMC released the questions
here as a set of practice questions for the 2002 AMC 10 and AMC 12 contests.

(24 )8
1 The ratio equals
(48 )2
1 1
(A) (B) (C) 1 (D) 2 (E) 8
4 2

2 The sum of eleven consecutive integers is 2002. What is the smallest of these integers?
(A) 175 (B) 177 (C) 179 (D) 180 (E) 181

3 Mary typed a six-digit number, but the two 1s she typed didn’t show. What appeared was 2002.
How many different six-digit numbers could she have typed?
(A) 4 (B) 8 (C) 10 (D) 15 (E) 20

© 2019 AoPS Incorporated 7


AoPS Community 2002 AMC 10

4 Which of the following numbers is a perfect square?


(A) 44 55 66 (B) 44 56 65 (C) 45 54 66 (D) 46 54 65 (E) 46 55 64

5 Let (an )n≥1 be a sequence such that a1 = 1 and 3an+1 − 3an = 1 for all n ≥ 1. Find a2002 .
(A) 666 (B) 667 (C) 668 (D) 669 (E) 670

6 The perimeter of a rectangle is 100 and its diagonal has length x. What is the area of this
rectangle?
x2 x2 x2
(A) 625 − x2 (B) 625 − (C) 1250 − x2 (D) 1250 − (E) 2500 −
2 2 2

7 The dimensions of a rectangular box in inches are all positive integers and the volume of the
box is 2002 in3 . Find the minimum possible sum in inches of the three dimensions.
(A) 36 (B) 38 (C) 42 (D) 44 (E) 92

8 How many ordered triples of positive integers (x, y, z) satisfy (xy )z = 64?
(A) 5 (B) 6 (C) 7 (D) 8 (E) 9

9 The function f is given by the table

x 1 2 3 4 5
f (x) 4 1 3 5 2

If u0 = 4 and un+1 = f (un ) for n ≥ 0, find u2002 .


(A) 1 (B) 2 (C) 3 (D) 4 (E) 5

10 Let a and b be distinct real numbers for which


a a + 10b
+ = 2.
b b + 10a
a
Find .
b
(A) 0.6 (B) 0.7 (C) 0.8 (D) 0.9 (E) 1

11 Let P (x) = kx3 + 2k 2 x2 + k 3 . Find the sum of all real numbers k for which x − 2 is a factor of
P (x).
(A) − 8 (B) − 4 (C) 0 (D) 5 (E) 8

© 2019 AoPS Incorporated 8


AoPS Community 2002 AMC 10

12 For fn (x) = xn and a 6= 1 consider


I. (f11 (a)f13 (a))14
II. f11 (a)f13 (a)f14 (a)
III. (f11 (f13 (a)))14
IV. f11 (f13 (f14 (a)))
Which of these equal f2002 (a)?
(A) I and II only (B) II and III only (C) III and IV only (D) II, III, and IV only (E) all of them

13 Participation in the local soccer league this year is 10% higher than last year. The number of
males increased by 5% and the number of females increased by 20%. What fraction of the
soccer league is now female?
1 4 2 4 1
(A) (B) (C) (D) (E)
3 11 5 9 2

14 The vertex E of a square EF GH is at the center of square ABCD. The length of a side of
ABCD is 1 and the length of a side of EF GH is 2. Side EF intersects CD at I and EH inter-
sects AD at J. If angle EID = 60◦ , the area of quadrilateral EIDJ is
√ √ √
1 3 1 2 3
(A) (B) (C) (D) (E)
4 6 3 4 2

15 What is the smallest integer n for which any subset of {1, 2, 3, . . . , 20} of size n must contain
two numbers that differ by 8?
(A) 2 (B) 8 (C) 12 (D) 13 (E) 15

16 Two walls and the ceiling of a room meet at right angles at point P . A fly is in the air one meter
from one wall, eight meters from the other wall, and 9 meters from point P . How many meters
is the fly from the ceiling?
√ √ √ √
(A) 13 (B) 14 (C) 15 (D) 4 (E) 17

17 There are 1001 red marbles and 1001 black marbles in a box. Let Ps be the probability that two
marbles drawn at random from the box are the same color, and let Pd be the probability that
they are different colors. Find |Ps − Pd |.
1 1 2 1
(A) 0 (B) (C) (D) (E)
2002 2001 2001 1000

18 For how many positive integers n is n3 − 8n2 + 20n − 13 a prime number?

© 2019 AoPS Incorporated 9


AoPS Community 2002 AMC 10

(A) 1 (B) 2 (C) 3 (D) 4 (E) more than 4

19 If a, b, c are real numbers such that a2 + 2b = 7, b2 + 4c = −7, and c2 + 6a = −14, find a2 + b2 + c2 .


(A) 14 (B) 21 (C) 28 (D) 35 (E) 49

20 How many three-digit numbers have at least one 2 and at least one 3?
(A) 52 (B) 54 (C) 56 (D) 58 (E) 60

21 Let f be a real-valued function such that


 
2002
f (x) + 2f = 3x
x

for all x > 0. Find f (2).


(A) 1000 (B) 2000 (C) 3000 (D) 4000 (E) 6000

2002!
22 In how many zeroes does the number end?
(1001!)2
(A) 0 (B) 1 (C) 2 (D) 200 (E) 400

23 Let
12 22 32 10012
a= + + + ··· +
1 3 5 2001
and
12 22 32 10012
b= + + + ··· + .
3 5 7 2003
Find the integer closest to a − b.
(A) 500 (B) 501 (C) 999 (D) 1000 (E) 1001

24 What is the maximum value of n for which there is a set of distinct positive integers k1 , k2 , . . . , kn
for which
k12 + k22 + . . . + kn2 = 2002?
(A) 14 (B) 15 (C) 16 (D) 17 (E) 18

25 Under the new AMC 10, 12 scoring method, 6 points are given for each correct answer, 2.5
points are given for each unanswered question, and no points are given for an incorrect answer.
Some of the possible scores between 0 and 150 can be obtained in only one way, for example,
the only way to obtain a score of 146.5 is to have 24 correct answers and one unanswered
question. Some scores can be obtained in exactly two ways; for example, a score of 104.5 can
be obtained with 17 correct answers, 1 unanswered question, and 7 incorrect, and also with 12

© 2019 AoPS Incorporated 10


AoPS Community 2002 AMC 10

correct answers and 13 unanswered questions. There are three scores that can be obtained in
exactly three ways. What is their sum?
(A) 175 (B) 179.5 (C) 182 (D) 188.5 (E) 201


These problems are copyright © Mathematical Association of America (http://maa.org).

© 2019 AoPS Incorporated 11


Art of Problem Solving is an ACS WASC Accredited School.
AoPS Community 2003 AMC 10

AMC 10 2003
www.artofproblemsolving.com/community/c4801
by worthawholebean, rrusczyk

– A

– February 11th

1 What is the difference between the sum of the first 2003 even counting numbers and the sum
of the first 2003 odd counting numbers?
(A) 0 (B) 1 (C) 2 (D) 2003 (E) 4006

2 Members of the Rockham Soccer League buy socks and T-shirts. Socks cost $4 per pair and
each T-shirt costs $5 more than a pair of socks. Each member needs one pair of socks and a
shirt for home games and another pair of socks and a shirt for away games. If the total cost
is $2366, how many members are in the League?
(A) 77 (B) 91 (C) 143 (D) 182 (E) 286

3 A solid box is 15 cm by 10 cm by 8 cm. A new solid is formed by removing a cube 3 cm on a


side from each corner of this box. What percent of the original volume is removed?
(A) 4.5 (B) 9 (C) 12 (D) 18 (E) 24

4 It takes Mary 30 minutes to walk uphill 1 km from her home to school, but it takes her only
10 minutes to walk from school to home along the same route. What is her average speed, in
km/hr, for the round trip?
(A) 3 (B) 3.125 (C) 3.5 (D) 4 (E) 4.5

5 Let d and e denote the solutions of 2x2 + 3x − 5 = 0. What is the value of (d − 1)(e − 1)?
(A) − 25 (B) 0 (C) 3 (D) 5 (E) 6

6 Define x♥y to be |x − y| for all real numbers x and y. Which of the following statements is not
true?
(A) x♥y = y♥x for all x and y (B) 2(x♥y) = (2x)♥(2y) for all x and y (C) x♥0 = x for all x
(D) x♥x = 0 for all x (E) x♥y > 0 if x 6= y

7 How many non-congruent triangles with perimeter 7 have integer side lengths?
(A) 1 (B) 2 (C) 3 (D) 4 (E) 5

8 What is the probability that a randomly drawn positive factor of 60 is less than 7?
1 1 1 1 1
(A) 10 (B) 6 (C) 4 (D) 3 (E) 2

© 2019 AoPS Incorporated 1


AoPS Community 2003 AMC 10

9 Simplify s r

3
q
3 3
x x x x
√ √
3

27 √ √
81
(A) x (B) x2 (C) x2 (D) 54
x (E) x80

10 The polygon enclosed by the solid lines in the gure consists of 4 congruent squares joined
edge-to-edge. One more congruent square is attached to an edge at one of the nine positions
indicated. How many of the nine resulting polygons can be folded to form a cube with one face
missing?

7 6 5

8 4

9 2 3

(A) 2 (B) 3 (C) 4 (D) 5 (E) 6

11 The sum of the two 5-digit numbers AM C10 and AM C12 is 123422. What is A + M + C?
(A) 10 (B) 11 (C) 12 (D) 13 (E) 14

12 A point (x, y) is randomly picked from inside the rectangle with vertices (0, 0), (4, 0), (4, 1), and
(0, 1). What is the probability that x < y?
(A) 18 (B) 41 (C) 38 (D) 12 (E) 34

13 The sum of three numbers is 20. The rst is 4 times the sum of the other two. The second is
seven times the third. What is the product of all three?
(A) 28 (B) 40 (C) 100 (D) 400 (E) 800

14 Let n be the largest integer that is the product of exactly 3 distinct prime numbers, d, e, and
10d + e, where d and e are single digits. What is the sum of the digits of n?
(A) 12 (B) 15 (C) 18 (D) 21 (E) 24

© 2019 AoPS Incorporated 2


AoPS Community 2003 AMC 10

15 What is the probability that an integer in the set {1, 2, 3, . . . , 100} is divisible by 2 and not divis-
ible by 3?
(A) 16 33
(B) 100 (C) 17
50 (D) 12 (E) 18
25

16 What is the units digit of 132003 ?


(A) 1 (B) 3 (C) 7 (D) 8 (E) 9

17 The number of inches in the perimeter of an equilateral triangle equals the number of square
inches
√ in the area
√ of its circumscribed
√ circle. What
√ is the radius, in inches, of the circle?
(A) 3 π 2 (B) 3 π 3 (C) 3 (D) π6 (E) 3π

18 What is the sum of the reciprocals of the roots of the equation


2003 1
x + 1 + = 0?
2004 x
2004 2003 2004
(A) − 2003 (B) −1 (C) 2004 (D) 1 (E) 2003

19 A semicircle of diameter 1 sits at the top of a semicircle of diameter 2, as shown. The shaded
area inside the smaller semicircle and outside the larger semicircle is called a lune. Determine
the area of this lune.

√ √ √ √ √
3 3 3 3 3
(A) 16 π − 4 (B) 4 − 1
12 π (C) 4 − 1
24 π (D) 4 + 1
24 π (E) 4 + 1
12 π

20 A base-10 three-digit number n is selected at random. Which of the following is closest to the
probability that the base-9 representation and the base-11 representation of n are both three-
digit numerals?
(A) 0.3 (B) 0.4 (C) 0.5 (D) 0.6 (E) 0.7

21 Pat is to select six cookies from a tray containing only chocolate chip, oatmeal, and peanut
butter cookies. There are at least six of each of these three kinds of cookies on the tray. How
many different assortments of six cookies can be selected?
(A) 22 (B) 25 (C) 27 (D) 28 (E) 29

© 2019 AoPS Incorporated 3


AoPS Community 2003 AMC 10

22 In rectangle ABCD, we have AB = 8, BC = 9, H is on BC with BH = 6, E is on AD with


DE = 4, line EC intersects line AH at G, and F is on line AD with GF ⊥ AF . Find the length
GF .

H 6 B
C

F
D 4 E A

(A) 16 (B) 20 (C) 24 (D) 28 (E) 30

23 A large equilateral triangle is constructed by using toothpicks to create rows of small equilat-
eral triangles. For example, in the gure we have 3 rows of small congruent equilateral triangles,
with 5 small triangles in the base row. How many toothpicks would be needed to construct a
large equilateral triangle if the base row of the triangle consists of 2003 small equilateral tri-
angles?

2 4
1 3 5

(A) 1,004,004 (B) 1,005,006 (C) 1,507,509 (D) 3,015,018 (E) 6,021,018

© 2019 AoPS Incorporated 4


AoPS Community 2003 AMC 10

24 Sally has ve red cards numbered 1 through 5 and four blue cards numbered 3 through 6. She
stacks the cards so that the colors alternate and so that the number on each red card divides
evenly into the number on each neighboring blue card. What is the sum of the numbers on the
middle three cards?
(A) 8 (B) 9 (C) 10 (D) 11 (E) 12

25 Let n be a 5-digit number, and let q and r be the quotient and remainder, respectively, when n
is divided by 100. For how many values of n is q + r divisible by 11?
(A) 8180 (B) 8181 (C) 8182 (D) 9000 (E) 9090

– B

– February 26th

1 Which of the following is the same as


2 − 4 + 6 − 8 + 10 − 12 + 14
?
3 − 6 + 9 − 12 + 15 − 18 + 21
2 2 14
(A) − 1 (B) − 3 (C) 3 (D) 1 (E) 3

2 Al gets the disease algebritis and must take one green pill and one pink pill each day for two
weeks. A green pill costs $1 more than a pink pill, and Als pills cost a total of $546 for the two
weeks. How much does one green pill cost?
(A) $7 (B) $14 (C) $19 (D) $20 (E) $39

3 The sum of 5 consecutive even integers is 4 less than the sum of the rst 8 consecutive odd
counting numbers. What is the smallest of the even integers?
(A) 6 (B) 8 (C) 10 (D) 12 (E) 14

4 Rose fills each of the rectangular regions of her rectangular flower bed with a different type
of flower. The lengths, in feet, of the rectangular regions in her flower bed are as shown in
the gure. She plants one flower per square foot in each region. Asters cost $1 each, begonias
$1.50 each, cannas $2 each, dahlias $2.50 each, and Easter lilies $3 each. What is the least
possible cost, in dollars, for her garden?

4 7

3
5

3
1
6 5

© 2019 AoPS Incorporated 5


AoPS Community 2003 AMC 10

(A) 108 (B) 115 (C) 132 (D) 144 (E) 156

5 Moe uses a mower to cut his rectangular 90-foot by 150-foot lawn. The swath he cuts is 28
inches wide, but he overlaps each cut by 4 inches to make sure that no grass is missed. He
walks at the rate of 5000 feet per hour while pushing the mower. Which of the following is
closest to the number of hours it will take Moe to mow his lawn?
(A) 0.75 (B) 0.8 (C) 1.35 (D) 1.5 (E) 3

6 Many television screens are rectangles that are measured by the length of their diagonals.
The ratio of the horizontal length to the height in a standard television screen is 4 : 3. The
horizontal length of a 27-inch television screen is closest, in inches, to which of the following?

al

Height
gon
i a
D

Length

(A) 20 (B) 20.5 (C) 21 (D) 21.5 (E) 22

7 The symbolism bxc denotes the largest integer not exceeding x. For example. b3c = 3, and
b9/2c = 4. Compute √ √ √ √
b 1c + b 2c + b 3c + · · · + b 16c.
(A) 35 (B) 38 (C) 40 (D) 42 (E) 136

8 The second and fourth terms of a geometric sequence are 2 and 6. Which of the following is a
possible first term? √
√ √ √
(A) − 3 (B) − 2 3 3 (C) − 33 (D) 3 (E) 3

9 Find the value of x that satisfies the equation

548/x
25−2 = .
526/x · 2517/x
(A) 2 (B) 3 (C) 5 (D) 6 (E) 9

10 Nebraska, the home of the AMC, changed its license plate scheme. Each old license plate con-
sisted of a letter followed by four digits. Each new license plate consists of three letters fol-
lowed by three digits. By how many times is the number of possible license plates increased?
2 2 263 263
(A) 26
10 (B) 26
102
(C) 26
10 (D) 103 (E) 102

© 2019 AoPS Incorporated 6


AoPS Community 2003 AMC 10

11 A line with slope 3 intersects a line with slope 5 at the point (10, 15). What is the distance
between the x-intercepts of these two lines?
(A) 2 (B) 5 (C) 7 (D) 12 (E) 20

12 Al, Betty, and Clare split $1000 among them to be invested in different ways. Each begins with
a different amount. At the end of one year they have a total of $1500. Betty and Clare have both
doubled their money, whereas Al has managed to lose $100. What was Als original portion?
(A) $250 (B) $350 (C) $400 (D) $450 (E) $500

13 Let ♣(x) denote the sum of the digits of the positive integer x. For example, ♣(8) = 8 and
♣(123) = 1 + 2 + 3 = 6. For how many two-digit values of x is ♣(♣(x)) = 3?
(A) 3 (B) 4 (C) 6 (D) 9 (E) 10

14 Given that 38 ·52 = ab , where both a and b are positive integers, find the smallest possible value
for a + b.
(A) 25 (B) 34 (C) 351 (D) 407 (E) 900

15 There are 100 players in a singles tennis tournament. The tournament is single elimination,
meaning that a player who loses a match is eliminated. In the first round, the strongest 28
players are given a bye, and the remaining 72 players are paired off to play. After each round,
the remaining players play in the next round. The match continues until only one player re-
mains unbeaten. The total number of matches played is
(A) a prime number (B) divisible by 2 (C) divisible by 5 (D) divisible by 7 (E) divisible by 11

16 A restaurant offers three desserts, and exactly twice as many appetizers as main courses. A
dinner consists of an appetizer, a main course, and a dessert. What is the least number of main
courses that the restaurant should offer so that a customer could have a different dinner each
night in the year 2003?
(A) 4 (B) 5 (C) 6 (D) 7 (E) 8

17 An ice cream cone consists of a sphere of vanilla ice cream and a right circular cone that has
the same diameter as the sphere. If the ice cream melts, it will exactly ll the cone. Assume that
the melted ice cream occupies 75% of the volume of the frozen ice cream. What is the ratio of
the cones height to its radius?
(A) 2 : 1 (B) 3 : 1 (C) 4 : 1 (D) 16 : 3 (E) 6 : 1

18 What is the largest integer that is a divisor of

(n + 1)(n + 3)(n + 5)(n + 7)(n + 9)

for all positive even integers n?


(A) 3 (B) 5 (C) 11 (D) 15 (E) 165

© 2019 AoPS Incorporated 7


AoPS Community 2003 AMC 10

19 Three semicircles of radius 1 are constructed on diameter AB of a semicircle of radius 2. The


centers of the small semicircles divide AB into four line segments of equal length, as shown.
What is the area of the shaded region that lies within the large semicircle but outside the
smaller semicircles?

A B
1 2 1

√ √ √
π+ 2

π+ 3

3
(A) π − 3 (B) π − 2 (C) 2 (D) 2 (E) 76 π − 2

20 In rectangle ABCD, AB = 5 and BC = 3. Points F and G are on CD so that DF = 1 and


GC = 2. Lines AF and BG intersect at E. Find the area of 4AEB.

D 1 2 C
F G
3
3

A 5 B

21 25
(A) 10 (B) 2 (C) 12 (D) 2 (E) 15

21 A bag contains two red beads and two green beads. You reach into the bag and pull out a bead,
replacing it with a red bead regardless of the color you pulled out. What is the probability that
all beads in the bag are red after three such replacements?
(A) 18 5
(B) 32 9
(C) 32 (D) 38 7
(E) 16

22 A clock chimes once at 30 minutes past each hour and chimes on the hour according to the
hour. For example, at 1 PM there is one chime and at noon and midnight there are twelve
chimes. Starting at 11:15 AM on February 26, 2003, on what date will the 2003rd chime occur?
(A) March 8 (B) March 9 (C) March 10 (D) March 20 (E) March 21

© 2019 AoPS Incorporated 8


AoPS Community 2003 AMC 10

23 A regular octagon ABCDEF GH has an area of one square unit. What is the area of the rect-
angle ABEF ?

A B

H C

G D

F E


2

2
√ 1

1+ 2
(A) 1 − 2 (B) 4 (C) 2−1 (D) 2 (E) 4

24 The rst four terms in an arithmetic sequence are x + y, x − y, xy, and x/y, in that order. What
is the fth term?
(A) − 158 (B) − 65 (C) 0 27
(D) 20 (E) 123
40

25 How many distinct four-digit numbers are divisible by 3 and have 23 as their last two digits?
(A) 27 (B) 30 (C) 33 (D) 81 (E) 90


These problems are copyright © Mathematical Association of America (http://maa.org).

© 2019 AoPS Incorporated 9


Art of Problem Solving is an ACS WASC Accredited School.
AoPS Community 2004 AMC 10

AMC 10 2004
www.artofproblemsolving.com/community/c4802
by #H34N1, TachyonPulse, worthawholebean, rrusczyk

– A

1 You and five friends need to raise $1500 in donations for a charity, dividing the fundraising
equally. How many dollars will each of you need to raise?
(A) 250 (B) 300 (C) 1500 (D) 7500 (E) 9000

2 For any three real numbers a, b, and c, with b 6= c, the operation ⊗ is defined by:
a
⊗(a, b, c) =
b−c
What is ⊗(⊗(1, 2, 3), ⊗(2, 3, 1), ⊗(3, 1, 2))?
(A) − 21 (B) − 14 (C) 0 (D) 41 (E) 1
2

3 Alicia earns $20 per hour, of which 1.45% is deducted to pay local taxes. How many cents per
hour of Alicia’s wages are used to pay local taxes?
(A) 0.0029 (B) 0.029 (C) 0.29 (D) 2.9 (E) 29

4 What is the value of x if |x − 1| = |x − 2|?


(A) − 12 (B) 12 (C) 1 (D) 32 (E) 2

5 A set of three points is randomly chosen from the grid shown. Each three point set has the
same probability of being chosen. What is the probability that the points lie on the same
straight line?

1 1 2 1 2
(A) 21 (B) 14 (C) 21 (D) 7 (E) 7

6 Bertha has 6 daughters and no sons. Some of her daughters have 6 daughters and the rest have
none. Bertha has a total of 30 daughters and granddaughters, and no great-grand daughters.
How many of Bertha’s daughters and granddaughters have no daughters?
(A) 22 (B) 23 (C) 24 (D) 25 (E) 26

7 A grocer stacks oranges in a pyramid-like stack whose rectangular base is 5 oranges by 8


oranges. Each orange above the first level rests in a pocket formed by four oranges in the level

© 2019 AoPS Incorporated 1


AoPS Community 2004 AMC 10

below. The stack is completed by a single row of oranges. How many oranges are in the stack?
(A) 96 (B) 98 (C) 100 (D) 101 (E) 134

8 A game is played with tokens according to the following rule. In each round, the player with
the most tokens gives one token to each of the other players and also places one token into a
discard pile. The game ends when some player runs out of tokens. Players A, B, and C start
with 15, 14, and 13 tokens, respectively. How many rounds will there be in the game?
(A) 36 (B) 37 (C) 38 (D) 39 (E) 40

9 In the figure, ∠EAB and ∠ABC are right angles. AB = 4, BC = 6, AE = 8, and AC and BE
intersect at D. What is the difference between the areas of 4ADE and 4BDC?

D
8 6

A 4 B

(A) 2 (B) 4 (C) 5 (D) 8 (E) 9

10 Coin A is flipped three times and coin B is flipped four times. What is the probability that the
number of heads obtained from flipping the two fair coins is the same?
19 23
(A) 128 (B) 128 (C) 14 35
(D) 128 (E) 21

11 A company sells peanut butter in cylindrical jars. Marketing research suggests that using
wider jars will increase sales. If the diameter of the jars is increased by 25% without alter-
ing the volume, by what percent must the height be decreased?
(A) 10 (B) 25 (C) 36 (D) 50 (E) 60

12 Henry’s Hamburger Heaven orders its hamburgers with the following condiments: ketchup,
mustard, mayonnaise, tomato, lettuce, pickles, cheese, and onions. A customer can choose
one, two, or three meat patties, and any collection of condiments. How many different kinds
of hamburgers can be ordered?
(A) 24 (B) 256 (C) 768 (D) 40,320 (E) 120,960

13 At a party, each man danced with exactly three women and each woman danced with exactly
two men. Twelve men attended the party. How many women attended the party?
(A) 8 (B) 12 (C) 16 (D) 18 (E) 24

© 2019 AoPS Incorporated 2


AoPS Community 2004 AMC 10

14 The average value of all the pennies, nickels, dimes, and quarters in Paula’s purse is 20 cents.
If she had one more quarter, the average value would be 21 cents. How many dimes does she
have in her purse?
(A) 0 (B) 1 (C) 2 (D) 3 (E) 4

15 Given that −4 ≤ x ≤ −2 and 2 ≤ y ≤ 4, what is the largest possible value of (x + y)/x?


(A) −1 (B) − 21 (C) 0 (D) 1
2 (E) 1

16 The 5 × 5 grid shown contains a collection of squares with sizes from 1 × 1 to 5 × 5. How many
of these squares contain the black center square?

(A) 12 (B) 15 (C) 17 (D) 19 (E) 20

17 Brenda and Sally run in opposite directions on a circular track, starting at diametrically oppo-
site points. They first meet after Brenda has run 100 meters. They next meet after Sally has
run 150 meters past their first meeting point. Each girl runs at a constant speed. What is the
length of the track in meters?
(A) 250 (B) 300 (C) 350 (D) 400 (E) 500

18 A sequence of three real numbers forms an arithmetic progression with a first term of 9. If 2 is
added to the second term and 20 is added to the third term, the three resulting numbers form a
geometric progression. What is the smallest possible value for the third term of the geometric
progression?
(A) 1 (B) 4 (C) 36 (D) 49 (E) 81

19 A white cylindrical silo has a diameter of 30 feet and a height of 80 feet. A red stripe with a
horizontal width of 3 feet is painted on the silo, as shown, making two complete revolutions
around it. What is the area of the stripe in square feet?

© 2019 AoPS Incorporated 3


AoPS Community 2004 AMC 10

3
80

30

(A) 120 (B) 180 (C) 240 (D) 360 (E) 480

20 Points E and F are located on square ABCD so that ∆BEF is equilateral. What is the ratio
of the area of ∆DEF to that of ∆ABE?

D F C

A B

4 3
√ √
(A) 3 (B) 2 (C) 3 (D) 2 (E) 1 + 3

21 Two distinct lines pass through the center of three concentric circles of radii 3, 2, and 1. The
area of the shaded region in the diagram is 8/13 of the area of the unshaded region. What is the

© 2019 AoPS Incorporated 4


AoPS Community 2004 AMC 10

radian measure of the acute angle formed by the two lines? (Note: π radians is 180 degrees.)

(A) π8 (B) π7 (C) π6 (D) π5 (E) π4

22 Square ABCD has side length 2. A semicircle with diameter AB is constructed inside the
square, and the tangent to the semicircle from C intersects side AD at E. What is the length
of CE?

D C

A B

√ √ √ √
(A) 2+2 5
(B) 5 (C) 6 (D) 52 (E)5 − 5

23 Circles A, B, and C are externally tangent to each other and internally tangent to circle D.
Circles B and C are congruent. Circle A has radius 1 and passes through the center of D.
What is the radius of circle B?

© 2019 AoPS Incorporated 5


AoPS Community 2004 AMC 10
D

√ √
2 3 7 8 1+ 3
(A) 3 (B) 2 (C) 8 (D) 9 (E) 3

24 Let a1 , a2 , · · · , be a sequence with the following properties.


I. a1 = 1, and
II. a2n = n · an for any positive integer n.
What is the value of a2100 ?
(A) 1 (B) 299 (C) 2100 (D) 24950 (E) 29999

25 Three mutually tangent spheres of radius 1 rest on a horizontal plane. A sphere of radius 2
rests on them. What is the distance from the plane to the top of the larger sphere?
√ √ √ √
(A) 3 + 230 (B) 3 + 369 (C) 3 + 123
4 (D) 52
9 (E) 3 + 2 2

– B

1 Each row of the Misty Moon Amphitheater has 33 seats. Rows 12 through 22 are reserved for
a youth club. How many seats are reserved for this club?
(A) 297 (B) 330 (C) 363 (D) 396 (E) 726

2 How many two-digit positive integers have at least one 7 as a digit?


(A) 10 (B) 18 (C) 19 (D) 20 (E) 30

3 At each basketball practice last week, Jenny made twice as many free throws as she made at
the previous practice. At her fifth practice she made 48 free throws. How many free throws did

© 2019 AoPS Incorporated 6


AoPS Community 2004 AMC 10

she make at the first practice?


(A) 3 (B) 6 (C) 9 (D) 12 (E) 15

4 A standard six-sided die is rolled, and P is the product of the fi ve numbers that are visible.
What is the largest number that is certain to divide P ?
(A) 6 (B) 12 (C) 24 (D) 144 (E) 720

5 In the expression c · ab − d, the values of a, b, c, and d are 0, 1, 2, and 3, although not necessarily
in that order. What is the maximum possible value of the result?
(A) 5 (B) 6 (C) 8 (D) 9 (E) 10

6 Which of the following numbers is a perfect square?


(A) 98! · 99! (B) 98! · 100! (C) 99! · 100! (D) 99! · 101! (E) 100! · 101!

7 On a trip from the United States to Canada, Isabella took d U.S. dollars. At the border she
exchanged them all, receiving 10 Canadian dollars for every 7 U.S. dollars. After spending 60
Canadian dollars, she had d Canadian dollars left. What is the sum of the digits of d?
(A) 5 (B) 6 (C) 7 (D) 8 (E) 9

8 Minneapolis-St. Paul International Airport is 8 miles southwest of downtown St. Paul and 10
miles southeast of downtown Minneapolis. Which of the following is closest to the number of
miles between downtown St. Paul and downtown Minneapolis?
(A) 13 (B) 14 (C) 15 (D) 16 (E) 17

9 A square has sides of length 10, and a circle centered at one of its vertices has radius 10. What
is the area of the union of the regions enclosed by the square and the circle?
(A) 200 + 25π (B) 100 + 75π (C) 75 + 100π (D) 100 + 100π (E) 100 + 125π

10 A grocer makes a display of cans in which the top row has one can and each lower row has
two more cans than the row above it. If the display contains 100 cans, how many rows does it
contain?
(A) 5 (B) 8 (C) 9 (D) 10 (E) 11

11 Two eight-sided dice each have faces numbered 1 through 8. When the dice are rolled, each
face has an equal probability of appearing on the top. What is the probability that the product
of the two top numbers is greater than their sum?
(A) 21 47
(B) 64 (C) 34 (D) 55
64 (E) 78

12 An annulus is the region between two concentric circles. The concentric circles in the gure
have radii b and c, with b > c. Let OX be a radius of the larger circle, let XZ be tangent to the
smaller circle at Z, and let OY be the radius of the larger circle that contains Z. Let a = XZ,
d = Y Z, and e = XY . What is the area of the annulus?

© 2019 AoPS Incorporated 7


AoPS Community 2004 AMC 10

(A) πa2 (B) πb2 (C) πc2 (D) πd2 (E) πe2

Y
e
d
a X
Z
c
b

13 In the United States, coins have the following thicknesses: penny, 1.55 mm; nickel, 1.95 mm;
dime, 1.35 mm; quarter, 1.75 mm. If a stack of these coins is exactly 14 mm high, how many
coins are in the stack?
(A) 7 (B) 8 (C) 9 (D) 10 (E) 11

14 A bag initially contains red marbles and blue marbles only, with more blue than red. Red mar-
bles are added to the bag until only 1/3 of the marbles in the bag are blue. Then yellow marbles
are added to the bag until only 1/5 of the marbles in the bag are blue. Finally, the number of
blue marbles in the bag is doubled. What fraction of the marbles now in the bag are blue?
(A) 51 (B) 41 (C) 13 (D) 25 (E) 12

15 Patty has 20 coins consisting of nickels and dimes. If her nickels were dimes and her dimes
were nickels, she would have 70 cents more. How much are her coins worth?
(A) $1.15 (B) $1.20 (C) $1.25 (D) $1.30 (E) $1.35

16 Three circles of radius 1 are externally tangent to each other and internally tangent to a larger
circle. What is the radius of the large circle?

© 2019 AoPS Incorporated 8


AoPS Community 2004 AMC 10

√ √ √ √
2+ 6 2+3 2 3+2 3 3+ 3
(A) 3 (B) 2 (C) 3 (D) 3 (E) 2

17 The two digits in Jack’s age are the same as the digits in Bill’s age, but in reverse order. In five
years Jack will be twice as old as Bill will be then. What is the difference in their current ages?
(A) 9 (B) 18 (C) 27 (D) 36 (E) 45

18 In right triangle 4ACE, we have AC = 12, CE = 16, and EA = 20. Points B, D, and F are
located on AC, CE, and EA, respectively, so that AB = 3, CD = 4, and EF = 5. What is the
ratio of the area of 4DBF to that of 4ACE?

B
15

9
F

C 4 D 12 E

1 9 3 11 7
(A) 4 (B) 25 (C) 8 (D) 25 (E) 16

19 In the sequence 2001, 2002, 2003, . . ., each term after the third is found by subtracting the pre-
vious term from the sum of the two terms that precede that term. For example, the fourth term
is 2001 + 2002 − 2003 = 2000. What is the 2004th term in this sequence?
(A) −2004 (B) −2 (C) 0 (D) 4003 (E) 6007

20 In 4ABC points D and E lie on BC and AC, respectively. If AD and BE intersect at T so that

© 2019 AoPS Incorporated 9


AoPS Community 2004 AMC 10

AT /DT = 3 and BT /ET = 4, what is CD/BD?

T D

A E C

1 2 3 4 5
(A) 8 (B) 9 (C) 10 (D) 11 (E) 12

21 Let 1, 4, · · · and 9, 16, · · · be two arithmetic progressions. The set S is the union of the fi rst
2004 terms of each sequence. How many distinct numbers are in S?
(A) 3722 (B) 3732 (C) 3914 (D) 3924 (E) 4007

22 A triangle with sides of 5, 12, and 13 has both an inscibed and a circumscribed circle. What is
the distance
√ between the√centers of those
√ circles?
(A) 3 2 5 (B) 72 (C) 15 (D) 265 (E) 92

23 Each face of a cube is painted either red or blue, each with probability 1/2. The color of each
face is determined independently. What is the probability that the painted cube can be placed
on a horizontal surface so that the four vertical faces are all the same color?
(A) 14 5
(B) 16 (C) 38 7
(D) 16 (E) 12

24 In 4ABC we have AB = 7, AC = 8, and BC = 9. Point D is on the circumscribed circle of the


triangle so that AD bisects ∠BAC. What is the value of AD/CD?
(A) 98 (B) 35 (C) 2 (D) 17
7 (E) 52

25 A circle of radius 1 is internally tangent to two circles of radius 2 at points A and B, where AB
is a diameter of the smaller circle. What is the area of the region, shaded in the gure, that is
outside the smaller circle and inside each of the two larger circles?

© 2019 AoPS Incorporated 10


AoPS Community 2004 AMC 10

A
2

√ √ √ √ √
(A) 53 π − 3 2 (B) 53 π − 2 3 (C) 38 π − 3 3 (D) 83 π − 3 2 (E) 83 π − 2 3


These problems are copyright © Mathematical Association of America (http://maa.org).

© 2019 AoPS Incorporated 11


Art of Problem Solving is an ACS WASC Accredited School.
AoPS Community 2005 AMC 10

AMC 10 2005
www.artofproblemsolving.com/community/c4803
by TachyonPulse, worthawholebean, Silverfalcon, catcurio, rrusczyk

– A

1 While eating out, Mike and Joe each tipped their server 2 dollars. Mike tipped 10% of his bill
and Joe tipped 20% of his bill. What was the difference, in dollars between their bills?
(A) 2 (B) 4 (C) 5 (D) 10 (E) 20

2 For each pair of real numbers a 6= b, define the operation ? as


a+b
(a ? b) = .
a−b
What is the value of ((1 ? 2) ? 3)?
(A) − 2
3 (B) − 1
5 (C) 0 (D) 1
2 (E) This value is not defined.

3 The equations 2x + 7 = 3 and bx − 10 = −2 have the same solution for x. What is the value of
b?
(A) −8 (B) −4 (C) −2 (D) 4 (E) 8

4 A rectangle with a diagonal of length x is twice as long as it is wide. What is the area of the
rectangle?
(A) 41 x2 (B) 52 x2 (C) 12 x2 (D) x2 (E) 23 x2

5 A store normally sells windows at $100 each. This week the store is offering one free window
for each purchase of four. Dave needs seven windows and Doug needs eight windows. How
many dollars will they save if they purchase the windows together rather than separately?
(A) 100 (B) 200 (C) 300 (D) 400 (E) 500

6 The average (mean) of 20 numbers is 30, and the average of 30 other numbers is 20. What is
the average of all 50 numbers?
(A) 23 (B) 24 (C) 25 (D) 26 (E) 27

7 Josh and Mike live 13 miles apart. Yesterday, Josh started to ride his bicycle toward Mike’s
house. A little later Mike started to ride his bicycle toward Josh’s house. When they met, Josh
had ridden for twice the length of time as Mike and at four-fifths of Mike’s rate. How many
miles had Mike ridden when they met?
(A) 4 (B) 5 (C) 6 (D) 7 (E) 8

© 2019 AoPS Incorporated 1


AoPS Community 2005 AMC 10

8 Square EF GH is inside the square ABCD so that each side of√EF GH can be extended to
pass through a vertex of ABCD. Square ABCD has side length 50 and BE = 1. What is the
area of the inner square EF GH?

A B

H
F

D C

(A) 25 (B) 32 (C) 36 (D) 40 (E) 42

9 Thee tiles are marked X and two other tiles are marked O. The five tiles are randomly arranged
in a row. What is the probability that the arrangement reads XOXOX?
1 1
(A) 12 (B) 10 (C) 16 (D) 14 (E) 13

10 There are two values of a for which the equation 4x2 + ax + 8x + 9 = 0 has only one solution
for x. What is the sum of these values of a?
(A) − 16 (B) − 8 (C) 0 (D) 8 (E) 20

11 A wooden cube n units on a side is painted red on all six faces and then cut into n3 unit cubes.
Exactly one-fourth of the total number of faces of the unit cubes are red. What is n?
(A) 3 (B) 4 (C) 5 (D) 6 (E) 7

12 The gure shown is called a trefoil and is constructed by drawing circular sectors about sides
of the congruent equilateral triangles. What is the area of a trefoil whose horizontal base has
length 2?

© 2019 AoPS Incorporated 2


AoPS Community 2005 AMC 10
√ √ √ √
3 3 3 3
(A) 13 π + 2 (B) 23 π (C) 32 π + 4 (D) 32 π + 3 (E) 23 π + 2

13 How many positive integers n satisfy the following condition:

(130n)50 > n100 > 2200 ?

(A) 0 (B) 7 (C) 12 (D) 65 (E) 125

14 How many three-digit numbers satisfy the property that the middle digit is the average of the
first and the last digits?
(A) 41 (B) 42 (C) 43 (D) 44 (E) 45

15 How many positive integer cubes divide 3! · 5! · 7!?


(A) 2 (B) 3 (C) 4 (D) 5 (E) 6

16 The sum of the digits of a two-digit number is subtracted from the number. The units digit of
the result is 6. How many two-digit numbers have this property?
(A) 5 (B) 7 (C) 9 (D) 10 (E) 19

17 In the five-sided star shown, the letters A, B, C, D, and E are replaced by the numbers 3, 5, 6, 7,
and 9, although not necessarily in this order. The sums of the numbers at the ends of the line
segments AB,BC,CD,DE, and EA form an arithmetic sequence, although not necessarily in
this order. What is the middle term of the arithmetic sequence?

C D

E B

(A) 9 (B) 10 (C) 11 (D) 12 (E) 13

18 Team A and team B play a series. The first team to win three games wins the series. Each
team is equally likely to win each game, there are no ties, and the outcomes of the individual
games are independent. If team B wins the second game and team A wins the series, what is

© 2019 AoPS Incorporated 3


AoPS Community 2005 AMC 10

the probability that team B wins the first game?


(A) 15 (B) 41 (C) 13 (D) 12 (E) 23

19 Three one-inch squares are palced with their bases on a line. The center square is lifted out
and rotated 45◦ , as shown. Then it is centered and lowered into its original location until it
touches both of the adjoining squares. How many inches is the point B from the line on which
the bases of the original squares were placed?

√ 3
√ 1
(A) 1 (B) 2 (C) 2 (D) 2+ 2 (E) 2

20 An equiangular octagon has four sides of length 1 and four sides of length 22 , arranged so
that no two consecutive

sides

have the same

length. What is the area of the octagon?
7 7 2 5+4 2 4+5 2
(A) 2 (B) 2 (C) 2 (D) 2 (E) 7

21 For how many positive integers n does 1 + 2 + · · · + n evenly divide from 6n?
(A) 3 (B) 5 (C) 7 (D) 9 (E) 11

22 Let S be the set of the 2005 smallest multiples of 4, and let T be the set of the 2005 smallest
positive multiples of 6. How many elements are common to S and T ?
(A) 166 (B) 333 (C) 500 (D) 668 (E) 1001

23 Let AB be a diameter of a circle and C be a point on AB with 2 · AC = BC. Let D and E be


points on the circle such that DC ⊥ AB and DE is a second diameter. What is the ratio of the
area of 4DCE to the area of 4ABD?

© 2019 AoPS Incorporated 4


AoPS Community 2005 AMC 10
D

A
C B

1 1 1 1 2
(A) 6 (B) 4 (C) 3 (D) 2 (E) 3

24 For each positive integer m > 1, let P (m) denote the greatest prime√
factor of m. For how many

positive integers n is it true that both P (n) = n and P (n + 48) = n + 48?
(A) 0 (B) 1 (C) 3 (D) 4 (E) 5

25 In ABC we have AB = 25, BC = 39, and AC = 42. Points D and E are on AB and AC
respectively, with AD = 19 and AE = 14. What is the ratio of the area of triangle ADE to the
area of quadrilateral BCED?
266 19
(A) 1521 (B) 75 (C) 13 (D) 19
56 (E) 1

– B

1 A scout troop buys 1000 candy bars at a price of five for $2. They sell all the candy bars at a
price of two for $1. What was their prot, in dollars?
(A) 100 (B) 200 (C) 300 (D) 400 (E) 500

2 A positive number x has the property that x% of x is 4. What is x?


(A) 2 (B) 4 (C) 10 (D) 20 (E) 40

3 A gallon of paint is used to paint a room. One third of the paint is used on the first day. On the
second day, one third of the remaining paint is used. What fraction of the original amount of
paint is available to use on the third day?
1
(A) 10 (B) 19 (C) 13 (D) 49 (E) 59

4 For real numbers a and b, define a  b = a2 + b2 . What is the value of
(5  12)  ((−12)  (−5))?
17

(A) 0 (B) 2 (C) 13 (D) 13 2 (E) 26

© 2019 AoPS Incorporated 5


AoPS Community 2005 AMC 10

5 Brianna is using part of the money she earned on her weekend job to buy several equally-priced
CDs. She used one fifth of her money to buy one third of the CDs. What fraction of her money
will she have left after she buys all the CDs?
1 1 2 2 4
(A) 5 (B) 3 (C) 5 (D) 3 (E) 5

6 At the beginning of the school year, Lisas goal was to earn an A on at least 80% of her 50
quizzes for the year. She earned an A on 22 of the first 30 quizzes. If she is to achieve her goal,
on at most how many of the remaining quizzes can she earn a grade lower than an A?
(A) 1 (B) 2 (C) 3 (D) 4 (E) 5

7 A circle is inscribed in a square, then a square is inscribed in this circle, and finally, a circle is
inscribed in this square. What is the ratio of the area of the smaller circle to the area of the
larger square?
π
(A) 16 (B) π8 (C) 3π
16 (D) π4 (E) π2

8 An 8-foot by 10-foot floor is tiled with square tiles of size 1 foot by 1 foot. Each tile has a pattern
consisting of four white quarter circles of radius 1/2 foot centered at each corner of the tile.
The remaining portion of the tile is shaded. How many square feet of the floor are shaded?

(A) 80 − 20π (B) 60 − 10π (C) 80 − 10π (D) 60 + 10π (E) 80 + 10π

9 One fair die has faces 1, 1, 2, 2, 3, 3 and another has faces 4, 4, 5, 5, 6, 6. The dice are rolled and
the numbers on the top faces are added. What is the probability that the sum will be odd?
(A) 13 (B) 94 (C) 12 (D) 59 (E) 23

10 In 4ABC, we have AC = BC = 7 and AB = 2. Suppose that D is a point on line AB such


that B lies between
√ A and D and CD = 8. What
√ is BD?
(A) 3 (B) 2 3 (C) 4 (D) 5 (E) 4 2

11 The first term of a sequence is 2005. Each succeeding term is the sum of the cubes of the
digits of the previous terms. What is the 2005th term of the sequence?
(A) 29 (B) 55 (C) 85 (D) 133 (E) 250

© 2019 AoPS Incorporated 6


AoPS Community 2005 AMC 10

12 Twelve fair dice are rolled. What is the probability that the product of the numbers on the top
faces is prime?
1 12
12 11 11 10
(B) 16 (C) 2 61 (D) 52 16 (E) 61

(A) 12

13 How many numbers between 1 and 2005 are integer multiples of 3 or 4 but not 12?
(A) 501 (B) 668 (C) 835 (D) 1002 (E) 1169

14 Equilateral 4ABC has side length 2, M is the midpoint of AC, and C is the midpoint of BD.
What is the area of 4CDM ?

B C D


2 3

3

(A) 2 (B) 4 (C) 2 (D) 1 (E) 2

15 An envelope contains eight bills: 2 ones, 2 fives, 2 tens, and 2 twenties. Two bills are drawn at
random without replacement. What is the probability that their sum is $20 or more?
(A) 14 (B) 72 (C) 37 (D) 12 (E) 23

16 The quadratic equation x2 + mx + n = 0 has roots that are twice those of x2 + px + m = 0, and
none of m, n, and p is zero. What is the value of np ?
(A) 1 (B) 2 (C) 4 (D) 8 (E) 16

17 Suppose that 4a = 5, 5b = 6, 6c = 7, and 7d = 8. What is a · b · c · d?


(A) 1 (B) 23 (C) 2 (D) 52 (E) 3

18 All of David’s telephone numbers have the form 555 − abc − def g, where a, b, c, d, e, f , and g are
distinct digits and in increasing order, and none is either 0 or 1. How many different telephone
numbers can David have?
(A) 1 (B) 2 (C) 7 (D) 8 (E) 9

19 On a certain math exam, 10% of the students got 70 points, 25% got 80 points, 20% got 85
points, 15% got 90 points, and the rest got 95 points. What is the difference between the mean
and the median score on this exam?
(A) 0 (B) 1 (C) 2 (D) 4 (E) 5

© 2019 AoPS Incorporated 7


AoPS Community 2005 AMC 10

20 What is the average (mean) of all 5-digit numbers that can be formed by using each of the
digits 1, 3, 5, 7, and 8 exactly once?
(A) 48000 (B) 49999.5 (C) 53332.8 (D) 55555 (E) 56432.8

21 Forty slips are placed into a hat, each bearing a number 1, 2, 3, 4, 5, 6, 7, 8, 9, or 10, with each
number entered on four slips. Four slips are drawn from the hat at random and without replace-
ment. Let p be the probability that all four slips bear the same number. Let q be the probability
that two of the slips bear a number a and the other two bear a number b 6= a. What is the value
of q/p?
(A) 162 (B) 180 (C) 324 (D) 360 (E) 720

22 For how many positive integers n less than or equal to 24 is n! evenly divisible by 1+2+· · ·+n?
(A) 8 (B) 12 (C) 16 (D) 17 (E) 21

23 In trapezoid ABCD we have AB parallel to DC, E as the midpoint of BC, and F as the midpoint
of DA. The area of ABEF is twice the area of F ECD. What is AB/DC?
(A) 2 (B) 3 (C) 5 (D) 6 (E) 8

24 Let x and y be two-digit integers such that y is obtained by reversing the digits of x. The inte-
gers x and y satisfy x2 − y 2 = m2 for some positive integer m. What is x + y + m?
(A) 88 (B) 112 (C) 116 (D) 144 (E) 154

25 A subset B of the set of integers from 1 to 100, inclusive, has the property that no two elements
of B sum to 125. What is the maximum possible number of elements in B?
(A) 50 (B) 51 (C) 62 (D) 65 (E) 68


These problems are copyright © Mathematical Association of America (http://maa.org).

© 2019 AoPS Incorporated 8


Art of Problem Solving is an ACS WASC Accredited School.
AoPS Community 2006 AMC 10

AMC 10 2006
www.artofproblemsolving.com/community/c4804
by joml88, chess64, Mrdavid445, solafidefarms, rrusczyk

– A

1 Sandwiches at Joe’s Fast Food cost $3 each and sodas cost $2 each. How many dollars will it
cost to purchase 5 sandwiches and 8 sodas?
(A) 31 (B) 32 (C) 33 (D) 34 (E) 35

2 Define x ⊗ y = x3 − y. What is h ⊗ (h ⊗ h)?


(A) − h (B) 0 (C) h (D) 2h (E) h3

3 The ratio of Mary’s age to Alice’s age is 3 : 5. Alice is 30 years old. How old is Mary?
(A) 15 (B) 18 (C) 20 (D) 24 (E) 50

4 A digital watch displays hours and minutes with cAM and cP M . What is the largest possible
sum of the digits in the display?
(A) 17 (B) 19 (C) 21 (D) 22 (E) 23

5 Doug and Dave shared a pizza with 8 equally-sized slices. Doug wanted a plain pizza, but Dave
wanted anchovies on half the pizza. The cost of a plain pizza was $8, and there was an ad-
ditional cost of $2 for putting anchovies on one half. Dave ate all the slices of anchovy pizza
and one plain slice. Doug ate the remainder. Each paid for what he had eaten. How many more
dollars did Dave pay than Doug?
(A) 1 (B) 2 (C) 3 (D) 4 (E) 5

6 What non-zero real value for x satisfies (7x)14 = (14x)7 ?


(A) 71 (B) 27 (C) 1 (D) 7 (E) 14

7 The 8 × 18 rectangle ABCD is cut into two congruent hexagons, as shown, in such a way that
the two hexagons can be repositioned without overlap to form a square. What is y?

D C

A B

(A) 6 (B) 7 (C) 8 (D) 9 (E) 10

© 2019 AoPS Incorporated 1


AoPS Community 2006 AMC 10

8 A parabola with equation y = x2 + bx + c passes through the points (2, 3) and (4, 3). What is c?
(A) 2 (B) 5 (C) 7 (D) 10 (E) 11

9 How many sets of two or more consecutive positive integers have a sum of 15?
(A) 1 (B) 2 (C) 3 (D) 4 (E) 5

For how many real values of x is 120 − x an integer?
p
10
(A) 3 (B) 6 (C) 9 (D) 10 (E) 11

11 Which of the following describes the graph of the equation (x + y)2 = x2 + y 2 ?


(A) the empty set (B) one point (C) two lines
(D) a circle (E) the entire plane

12 Rolly wishes to secure his dog with an 8-foot rope to a square shed that is 16 feet on each side.
His preliminary drawings are shown. Which of these arrangements gives the dog the greater
area to roam, and by how many square feet?

8
8
I
Rope Dog
8

4 8
Rope Dog
II

(A) I, by 8π (B) I, by 6π (C) II, by 4π (D) II, by 8π (E) II, by 10π

13 A player pays $5 to play a game. A die is rolled. If the number on the die is odd, the game is
lost. If the number on the die is even, the die is rolled again. In this case the player wins if
the second number matches the first and loses otherwise. How much should the player win if
the game is fair? (In a fair game the probability of winning times the amount won is what the

© 2019 AoPS Incorporated 2


AoPS Community 2006 AMC 10

player should pay.)


(A) $12 (B) $30 (C) $50 (D) $60 (E) $100

14 A number of linked rings, each 1 cm thick, are hanging on a peg. The top ring has an outside
diameter of 20 cm. The outside diameter of each of the outer rings is 1 cm less than that of
the ring above it. The bottom ring has an outside diameter of 3 cm. What is the distance, in
cm, from the top of the top ring to the bottom of the bottom ring?

18 20

(A) 171 (B) 173 (C) 182 (D) 188 (E) 210

15 Odell and Kershaw run for 30 minutes on a circular track. Odell runs clockwise at 250 m/min
and uses the inner lane with a radius of 50 meters. Kershaw runs counterclockwise at 300
m/min and uses the outer lane with a radius of 60 meters, starting on the same radial line as
Odell. How many times after the start do they pass each other?
(A) 29 (B) 42 (C) 45 (D) 47 (E) 50

16 A circle of radius 1 is tangent to a circle of radius 2. The sides of 4ABC are tangent to the
circles as shown, and the sides AB and AC are congruent. What is the area of 4ABC?

© 2019 AoPS Incorporated 3


AoPS Community 2006 AMC 10
A

B C

35
√ 64

(A) 2 (B) 15 2 (C) 3 (D) 16 2 (E) 24

17 In rectangle ADEH, points B and C trisect AD, and points G and F trisect HE. In addition,
AH = AC = 2. What is the area of quadrilateral W XY Z shown in the figure?

A B C D

Z X

Y
H G F E

√ √ √ √
1 2 3 2 2 2 3
(A) 2 (B) 2 (C) 2 (D) 3 (E) 3

18 A license plate in a certain state consists of 4 digits, not necessarily distinct, and 2 letters,
also not necessarily distinct. These six characters may appear in any order, except that the
two letters must appear next to each other. How many distinct license plates are possible?
(A) 104 · 262 (B) 103 · 263 (C) 5 · 104 · 262 (D) 102 · 264
(E) 5 · 103 · 263

19 How many non-similar triangle have angles whose degree measures are distinct positive inte-

© 2019 AoPS Incorporated 4


AoPS Community 2006 AMC 10

gers in arithmetic progression?


(A) 0 (B) 1 (C) 59 (D) 89 (E) 178

20 Six distinct positive integers are randomly chosen between 1 and 2006, inclusive. What is the
probability that some pair of these integers has a difference that is a multiple of 5?
(A) 12 (B) 35 (C) 23 (D) 45 (E) 1

21 How many four-digit positive integers have at least one digit that is a 2 or a 3?
(A) 2439 (B) 4096 (C) 4903 (D) 4904 (E) 5416

22 Two farmers agree that pigs are worth $300 and that goats are worth $210. When one farmer
owes the other money, he pays the debt in pigs or goats, with “change” received in the form
of goats or pigs as necessary. (For example, a $390 debt could be paid with two pigs, with
one goat received in change.) What is the amount of the smallest positive debt that can be
resolved in this way?
(A) $5 (B) $10 (C) $30 (D) $90 (E) $210

23 Circles with centers A and B have radii 3 and 8, respectively. A common internal tangent inter-
sects the circles at C and D, respectively. Lines AB and CD intersect at E, and AE = 5. What
is CD?

A B
E
C

44
√ √ 55
(A) 13 (B) 3 (C) 221 (D) 255 (E) 3

24 Centers of adjacent faces of a unit cube are joined to form a regular octahedron. What is the
volume of this octahedron?
(A) 18 (B) 16 (C) 14 (D) 13 (E) 21

25 A bug starts at one vertex of a cube and moves along the edges of the cube according to
the following rule. At each vertex the bug will choose to travel along one of the three edges
emanating from that vertex. Each edge has equal probability of being chosen, and all choices
are independent. What is the probability that after seven moves the bug will have visited every

© 2019 AoPS Incorporated 5


AoPS Community 2006 AMC 10

vertex exactly once?


1 1 2 1 5
(A) 2187 (B) 729 (C) 243 (D) 81 (E) 243

– B

1 What is (−1)1 + (−1)2 + · · · + (−1)2006 ?


(A) − 2006 (B) − 1 (C) 0 (D) 1 (E) 2006

2 For real numbers x and y, define x♠y = (x + y)(x − y). What is 3♠(4♠5)?
(A) − 72 (B) − 27 (C) − 24 (D) 24 (E) 72

3 A football game was played between two teams, the Cougars and the Panthers. The two teams
scored a total of 34 points, and the Cougars won by a margin of 14 points. How many points
did the Panthers score?
(A) 10 (B) 14 (C) 17 (D) 20 (E) 24

4 Circles of diameter 1 inch and 3 inches have the same center. The smaller circle is painted red,
and the portion outside the smaller circle and inside the larger circle is painted blue. What is
the ratio of the blue-painted area to the red-painted area?
(A) 2 (B) 3 (C) 6 (D) 8 (E) 9

5 A 2 x 3 rectangle and a 3 x 4 rectangle are contained within a square without overlapping at any
interior point, and the sides of the square are parallel to the sides of the two given rectangles.
What is the smallest possible area of the square?
(A) 16 (B) 25 (C) 36 (D) 49 (E) 64

6 A region is bounded by semicircular arcs constructed on the side of a square whose sides
measure 2/π, as shown. What is the perimeter of this region?

4 8 16
(A) π (B) 2 (C) π (D) 4 (E) π

x
r
7 Which of the following is equivalent to when x < 0?
1 − x−1
x

© 2019 AoPS Incorporated 6


AoPS Community 2006 AMC 10
px √
(A) − x (B) x (C) 1 (D) 2 (E) x −1

8 A square of area 40 is inscribed in a semicircle as shown. What is the area of the semicircle?

(A) 20π (B) 25π (C) 30π (D) 40π (E) 50π

9 Francesca uses 100 grams of lemon juice, 100 grams of sugar, and 400 grams of water to make
lemonade. There are 25 calories in 100 grams of lemon juice and 386 calories in 100 grams of
sugar. Water contains no calories. How many calories are in 200 grams of her lemonade?
(A) 129 (B) 137 (C) 174 (D) 223 (E) 411

10 In a triangle with integer side lengths, one side is three times as long as a second side, and
the length of the third side is 15. What is the greatest possible perimeter of the triangle?
(A) 43 (B) 44 (C) 45 (D) 46 (E) 47

11 What is the tens digit in the sum 7! + 8! + 9! + · · · + 2006!?


(A) 1 (B) 3 (C) 4 (D) 6 (E) 9

12 The lines x = 14 y + a and y = 41 x + b intersect at the point (1, 2). What is a + b?


(A) 0 (B) 43 (C) 1 (D) 2 (E) 49

13 Joe and JoAnn each bought 12 ounces of coffee in a 16-ounce cup. Joe drank 2 ounces of
his coffee and then added 2 ounces of cream. JoAnn added 2 ounces of cream, stirred the
coffee well, and then drank 2 ounces. What is the resulting ratio of the amount of cream in
Joe’s coffee to that in JoAnn’s coffee?
(A) 67 (B) 13
14 (C) 1 (D) 14
13 (E) 76

14 Let a and b be the roots of the equation x2 − mx + 2 = 0. Suppose that a + (1/b) and b + (1/a)
are the roots of the equation x2 − px + q = 0. What is q?
(A) 52 (B) 72 (C) 4 (D) 92 (E) 8

15 Rhombus ABCD is similar to rhombus BF DE. The area of rhombus ABCD is 24, and ∠BAD =
60◦ . What is the area of rhombus BF DE?

© 2019 AoPS Incorporated 7


AoPS Community 2006 AMC 10
D C

A B

√ √
(A) 6 (B) 4 3 (C) 8 (D) 9 (E) 6 3

16 Leap Day, February 29, 2004, occurred on a Sunday. On what day of the week will Leap Day,
February 29, 2020, occur?
(A) Tuesday (B) Wednesday (C) Thursday (D) Friday (E) Saturday

17 Bob and Alice each have a bag that contains one ball of each of the colors blue, green, orange,
red, and violet. Alice randomly selects one ball from her bag and puts it into Bob’s bag. Bob
then randomly selects one ball from his bag and puts it into Alice’s bag. What is the probability
that after this process, the contents of the two bags are the same?
1
(A) 10 (B) 16 (C) 51 (D) 13 (E) 12

18 Let a1 , a2 , ... be a sequence for which


an−1
a1 = 2 a2 = 3 and an = for each positive integer n ≥ 3.
an−2

What is a2006 ?
1 2 3
(A) 2 (B) 3 (C) 2 (D) 2 (E) 3

19 A circle of radius 2 is centered at O. Square OABC has side length 1. Sides AB and CB are
extended past b to meet the circle at D and E, respectively. What is the area of the shaded
region in the figure, which is bounded by BD, BE, and the minor arc connecting D and E?

© 2019 AoPS Incorporated 8


AoPS Community 2006 AMC 10
D

C E
B

O A

√ √  √  √
3−1
(A) π3 + 1 − 3 (B) π
2− 3 (C) π 2 − 3 (D) π
+
√ 2 6 2
(E) π3 − 1 + 3

20 In rectangle ABCD, we have A = (6, −22), B = (2006, 178), and D = (8, y), for some integer y.
What is the area of rectangle ABCD?
(A) 4000 (B) 4040 (C) 4400 (D) 40, 000 (E) 40, 400

21 For a particular peculiar pair of dice, the probabilities of rolling 1, 2, 3, 4, 5 and 6 on each die
are in the ratio 1 : 2 : 3 : 4 : 5 : 6. What is the probability of rolling a total of 7 on the two dice?
4
(A) 63 (B) 18 (C) 638
(D) 16 (E) 27

22 Elmo makes N sandwiches for a fundraiser. For each sandwich he uses B globs of peanut
butter at 4 cents per glob and J blobs of jam at 5 cents per glob. The cost of the peanut butter
and jam to make all the sandwiches is $2.53. Assume that B, J, and N are all positive integers
with N > 1. What is the cost of the jam Elmo uses to make the sandwiches?
(A) $1.05 (B) $1.25 (C) $1.45 (D) $1.65 (E) $1.85

23 A triangle is partitioned into three triangles and a quadrilateral by drawing two lines from ver-
tices to their opposite sides. The areas of the three triangles are 3, 7, and 7, as shown. What
is the area of the shaded quadrilateral?

3 7
7

© 2019 AoPS Incorporated 9


AoPS Community 2006 AMC 10
35 55
(A) 15 (B) 17 (C) 2 (D) 18 (E) 3

24 Circles with centers O and P have radii 2 and 4, respectively, and are externally tangent. Points
A and B are on the circle centered at O, and points C and D are on the circle centered at P , such
that AD and BC are common external tangents to the circles. What is the area of hexagon
AOBCP D?

A
4
2
P
O

√ √ √ √
(A) 18 3 (B) 24 2 (C) 36 (D) 24 3 (E) 32 2

25 Mr. Jones has eight children of different ages. On a family trip his oldest child, who is 9, spots
a license plate with a 4-digit number in which each of two digits appears two times. ”Look,
daddy!” she exclaims. ”That number is evenly divisible by the age of each of us kids!” ”That’s
right,” replies Mr. Jones, ”and the last two digits just happen to be my age.” Which of the fol-
lowing is not the age of one of Mr. Jones’s children?
(A) 4 (B) 5 (C) 6 (D) 7 (E) 8


These problems are copyright © Mathematical Association of America (http://maa.org).

© 2019 AoPS Incorporated 10


Art of Problem Solving is an ACS WASC Accredited School.
AoPS Community 2007 AMC 10

AMC 10 2007
www.artofproblemsolving.com/community/c4805
by worthawholebean, Mrdavid445, TachyonPulse, t0rajir0u, junggi, rrusczyk

– A

1 One ticket to a show costs $20 at full price. Susan buys 4 tickets using a coupon that gives her
a 25% discount. Pam buys 5 tickets using a coupon that gives her a 30% discount. How many
more dollars does Pam pay than Susan?
(A) 2 (B) 5 (C) 10 (D) 15 (E) 20

2 Define a@b = ab − b2 and a#b = a + b − ab2 . What is 6#2 ?


6@2

(A) − 12 (B) − 41 (C) 18 (D) 14 (E) 21

3 An aquarium has a rectangular base that measures 100 cm by 40 cm and has a height of 50
cm. It is filled with water to a height of 40 cm. A brick with a rectangular base that measures
40 cm by 20 cm and a height of 10 cm is placed in the aquarium. By how many centimeters
does the water rise?
(A) 0.5 (B) 1 (C) 1.5 (D) 2 (E) 2.5

4 The larger of two consecutive odd integers is three times the smaller. What is their sum?
(A) 4 (B) 8 (C) 12 (D) 16 (E) 20

5 A school store sells 7 pencils and 8 notebooks for $4.15. It also sells 5 pencils and 3 notebooks
for $1.77. How much do 16 pencils and 10 notebooks cost?
(A) $1.76 (B) $5.84 (C) $6.00 (D) $6.16 (E) $6.32

6 At Euclid High School, the number of students taking the AMC10 was 60 in 2002, 66 in 2003,
70 in 2004, 76 in 2005, 78 in 2006, and is 85 in 2007. Between what two consecutive years was
there the largest percentage increase?
(A) 2002 and 2003 (B) 2003 and 2004 (C) 2004 and 2005 (D) 2005 and 2006 (E) 2006 and 2007

7 Last year Mr. John Q. Public received an inheritance. He paid 20% in federal taxes on the in-
heritance, and paid 10% of what he has left in state taxes. He paid a total of $10, 500 for both
taxes. How many dollars was the inheritance?
(A) 30, 000 (B) 32, 500 (C) 35, 000 (D) 37, 500 (E) 40, 000

© 2019 AoPS Incorporated 1


AoPS Community 2007 AMC 10

8 Triangles ABC and ADC are isosceles with AB = BC and AD = DC. Point D is inside 4ABC.
∠ABC = 40◦ , and ∠ADC = 140◦ . What is the degree measure of ∠BAD?
(A) 20 (B) 30 (C) 40 (D) 50 (E) 60

9 Real numbers a and b satisfy the equations 3a = 81b+2 and 125b = 5a−3 . What is ab?
(A) −60 (B) −17 (C) 9 (D) 12 (E) 60

10 The Dunbar family consists of a mother, a father, and some children. The average age of the
members of the family is 20, the father is 48 years old, and the average age of the mother and
children is 16. How many children are in the family?
(A) 2 (B) 3 (C) 4 (D) 5 (E) 6

11 The numbers from 1 to 8 are placed at the vertices of a cube in such a manner that the sum of
the four numbers on each face is the same. What is this common sum?
(A) 14 (B) 16 (C) 18 (D) 20 (E) 24

12 Two tour guides are leading six tourists. The guides decide to split up. Each tourist must
choose one of the guides, but with the stipulation that each guide must take at least one
tourist. How many different groupings of guides and tourists are possible?
(A) 56 (B) 58 (C) 60 (D) 62 (E) 64

13 Yan is somewhere between his home and the stadium. To get to the stadium he can walk
directly to the stadium, or else he can walk home and then ride his bicycle to the stadium. He
rides 7 times as fast as he walks, and both choices require the same amount of time. What is
the ratio of Yan’s distance from his home to his distance from the stadium?
(A) 23 (B) 43 (C) 45 (D) 56 (E) 67

14 A triangle with side lengths in the ratio 3 : 4 : 5 is inscribed in a circle of radius 3. What is the
area of the triangle?
(A) 8.64 (B) 12 (C) 5π (D) 17.28 (E) 18

15 Four circles of radius 1 are each tangent to two sides of a square and externally tangent to a
circle of radius 2, as shown. What is the area of the square?

© 2019 AoPS Incorporated 2


AoPS Community 2007 AMC 10
√ √ √
(A) 32 (B) 22 + 12 2 (C) 16 + 16 3 (D) 48 (E) 36 + 16 2

16 Integers a, b, c, and d, not necessarily distinct, are chosen independently and at random from
0 to 2007, inclusive. What is the probability that ad − bc is even?
(A) 83 (B) 167
(C) 12 9
(D) 16 (E) 58

17 Suppose that m and n are positive integers such that 75m = n3 . What is the minimum possible
value of m + n?
(A) 15 (B) 30 (C) 50 (D) 60 (E) 5700

18 Consider the 12-sided polygon ABCDEF GHIJKL, as shown. Each of its sides has length
4, and each two consecutive sides form a right angle. Suppose that AG and CH meet at M .
What is the area of quadrilateral ABCM ?

A B

K L C D

M
J I F E

H G

44 88 62
(A) 3 (B) 16 (C) 5 (D) 20 (E) 3

19 A paint brush is swept along both diagonals of a square to produce the symmetric painted
area, as shown. Half the area of the square is painted. What is the ratio of the side length of
the square to the brush width?

√ √ √ √ √
(A) 2 2 + 1 (B) 3 2 (C) 2 2 + 2 (D) 3 2 + 1 (E) 3 2 + 2

© 2019 AoPS Incorporated 3


AoPS Community 2007 AMC 10

20 Suppose that the number a satisfies the equation 4 = a + a−1 . What is the value of a4 + a−4 ?
(A) 164 (B) 172 (C) 192 (D) 194 (E) 212

21 A sphere is inscribed in a cube that has a surface area of 24 square meters. A second cube is
then inscribed within the sphere. What is the surface area in square meters of the inner cube?
(A) 3 (B) 6 (C) 8 (D) 9 (E) 12

22 A finite sequence of three-digit integers has the property that the tens and units digits of each
terms are, respectively, the hundreds and tens digits of the next term, and the tens and units
digits of the last term are, respectively, the hundreds and tens digits of the first term. For exam-
ple, such a sequence might begin with terms 247, 475, and 756 and end with the term 824. Let
S be the sum of all the terms in the sequence. What is the largest prime number that always
divides S?
(A) 3 (B) 7 (C) 13 (D) 37 (E) 43

23 How many ordered pairs (m, n) of positive integers, with m > n, have the property that their
squares differ by 96?
(A) 3 (B) 4 (C) 6 (D) 9 (E) 12

24 Circles centered
√ at A and B each have radius 2, as shown. Point O is the midpoint of AB, and
OA = 2 2. Segments OC and OD are tangent to the circles centered at A and B, respectively,
and EF is a common tangent. What is the area of the shaded region ECODF ?

E F
C D

A B
O
2 2


8 2
√ √ √ π
√ π
(A) 3 (B) 8 2 − 4 − π (C) 4 2 (D) 4 2 + 8 (E) 8 2 − 2 − 2

25 For each positive integer n, let S(n) denote the sum of the digits of n. For how many values of
n is n + S(n) + S(S(n)) = 2007?
(A) 1 (B) 2 (C) 3 (D) 4 (E) 5

– B

1 Isabella’s house has 3 bedrooms. Each bedroom is 12 feet long, 10 feet wide, and 8 feet high.
Isabella must paint the walls of all the bedrooms. Doorways and windows, which will not be
painted, occupy 60 square feet in each bedroom. How many square feet of walls must be

© 2019 AoPS Incorporated 4


AoPS Community 2007 AMC 10

painted?
(A) 678 (B) 768 (C) 786 (D) 867 (E) 876

2 Define the operation ? by a ? b = (a + b)b. What is (3 ? 5) − (5 ? 3)?


(A) − 16 (B) − 8 (C) 0 (D) 8 (E) 16

3 A college student drove his compact car 120 miles home for the weekend and averaged 30
miles per gallon. On the return trip the student drove his parents’ SUV and averaged only 20
miles per gallon. What was the average gas mileage, in miles per gallon, for the round trip?
(A) 22 (B) 24 (C) 25 (D) 26 (E) 28

4 The point O is the center of the circle circumscribed about 4ABC, with ∠BOC = 120◦ and
∠AOB = 140◦ , as shown. What is the degree measure of ∠ABC?

140◦ 120◦
O

A C

(A) 35 (B) 40 (C) 45 (D) 50 (E) 60

5 In a certain land, all Arogs are Brafs, all Crups are Brafs, all Dramps are Arogs, and all Crups
are Dramps. Which of the following statements is implied by these facts?
(A) All Dramps are Brafs and are Crups.
(B) All Brafs are Crups and are Dramps.
(C) All Arogs are Crups and are Dramps.
(D) All Crups are Arogs and are Brafs.
(E) All Arogs are Dramps and some Arogs may not be Crups.

6 The 2007 AMC 10 will be scored by awarding 6 points for each correct response, 0 points for
each incorrect response, and 1.5 points for each problem left unanswered. After looking over
the 25 problems, Sarah has decided to attempt the first 22 and leave only the last 3 unanswered.
How many of the first 22 problems must she solve correctly in order to score at least 100
points?
(A) 13 (B) 14 (C) 15 (D) 16 (E) 17

© 2019 AoPS Incorporated 5


AoPS Community 2007 AMC 10

7 All sides of the convex pentagon ABCDE are of equal length, and ∠A = ∠B = 90◦ . What is
the degree measure of ∠E?
(A) 90 (B) 108 (C) 120 (D) 144 (E) 150

8 On the trip home from the meeting where this AMC10 was constructed, the Contest Chair noted
that his airport parking receipt had digits of the form bbcac, where 0 ≤ a < b < c ≤ 9, and b was
the average of a and c. How many different five-digit numbers satisfy all these properties?
(A) 12 (B) 16 (C) 18 (D) 20 (E) 24

9 A cryptographic code is designed as follows. The first time a letter appears in a given message
it is replaced by the letter that is 1 place to its right in the alphabet (assuming that the letter A
is one place to the right of the letter Z). The second time this same letter appears in the given
message, it is replaced by the letter that is 1 + 2 places to the right, the third time it is replaced
by the letter that is 1 + 2 + 3 places to the right, and so on. For example, with this code the word
”banana” becomes ”cbodqg”. What letter will replace the last letter s in the message ”Lee’s sis
is a Mississippi miss, Chriss!”?
(A) g (B) h (C) o (D) s (E) t

10 Two points B and C are in a plane. Let S be the set of all points A in the plane for which 4ABC
has area 1. Which of the following describes S?
(A) two parallel lines (B) a parabola (C) a circle (D) a line segment (E) two points

11 A circle passes through the three vertices of an isosceles triangle that has two sides of length
3 and a base of length 2. What is the area of this circle?
(A) 2π (B) 52 π (C) 81
32 π (D) 3π (E) 72 π

12 Tom’s age is T years, which is also the sum of the ages of his three children. His age N years
ago was twice the sum of their ages then. What is N T
?
(A) 2 (B) 3 (C) 4 (D) 5 (E) 6

13 Two circles of radius 2 are centered at (2, 0) and at (0, 2). What is the area of the intersection
of the interiors of the two circles?

(A) π − 2 (B) π2 (C) π 3 3 (D) 2(π − 2) (E) π

14 Some boys and girls are having a car wash to raise money for a class trip to China. Initially
40% of the group are girls. Shortly thereafter two girls leave and two boys arrive, and then 30%
of the group are girls. How many girls were initially in the group?
(A) 4 (B) 6 (C) 8 (D) 10 (E) 12

15 The angles of quadrilateral ABCD satisfy ∠A = 2∠B = 3∠C = 4∠D. What is the degree
measure of ∠A, rounded to the nearest whole number?
(A) 125 (B) 144 (C) 153 (D) 173 (E) 180

© 2019 AoPS Incorporated 6


AoPS Community 2007 AMC 10

16 A teacher gave a test to a class in which 10% of the students are juniors and 90% are seniors.
The average score on the test was 84. The juniors all received the same score, and the average
score of the seniors was 83. What score did each of the juniors receive on the test?
(A) 85 (B) 88 (C) 93 (D) 94 (E) 98

17 Point P is inside equilateral 4ABC. Points Q, R and S are the feet of the perpendiculars from
P to AB, BC, √ and CA, respectively.√
Given that P Q = 1, P R = 2, and P S = 3, what is AB?
(A) 4 (B) 3 3 (C) 6 (D) 4 3 (E) 9

18 A circle of radius 1 is surrounded by 4 circles of radius r as shown. What is r?

r r

r r

√ √ √ √
(A) 2 (B) 1 + 2 (C) 6 (D) 3 (E) 2 + 2

19 The wheel shown is spun twice, and the randomly determined numbers opposite the pointer
are recorded. The first number is divided by 4, and the second number is divided by 5. The first
remainder designates a column, and the second remainder designates a row on the checker-
board shown. What is the probability that the pair of numbers designates a shaded square?

4
6 1
3
Pointer 3 7
2
9 2 1
1 2 3

© 2019 AoPS Incorporated 7


AoPS Community 2007 AMC 10
1 4 1 5 2
(A) 3 (B) 9 (C) 2 (D) 9 (E) 3

20 A set of 25 square blocks is arranged into a 5 × 5 square. How many different combinations of
3 blocks can be selected from that set so that no two are in the same row or column?
(A) 100 (B) 125 (C) 600 (D) 2300 (E) 3600

21 Right 4ABC has AB = 3, BC = 4, and AC = 5. Square XY ZW is inscribed in 4ABC with X


and Y on AC, W on AB, and Z on BC. What is the side length of the square?

W Z

A X Y C

3 60 12 23
(A) 2 (B) 37 (C) 7 (D) 13 (E) 2

22 A player chooses one of the numbers 1 through 4. After the choice has been made, two regular
four-sided (tetrahedral) dice are rolled, with the sides of the dice numbered 1 through 4. If the
number chosen appears on the bottom of exactly one die after it is rolled, then the player wins
$1. If the number chosen appears on the bottom of both of the dice, then the player wins $2.
If the number chosen does not appear on the bottom of either of the dice, the player loses $1.
What is the expected return to the player, in dollars, for one roll of the dice?
(A) − 18 1
(B) − 16 (C) 0 1
(D) 16 (E) 18

23 A pyramid with a square base is cut by a plane that is parallel to its base and is 2 units from
the base. The surface area of the smaller pyramid that is cut from the top is half the surface
area of the original
√ pyramid. What√ is the altitude of the original
√ pyramid?
(A) 2 (B) 2 + 2 (C) 1 + 2 2 (D) 4 (E) 4 + 2 2

24 Let n denote the smallest positive integer that is divisible by both 4 and 9, and whose base-10
representation consists of only 4’s and 9’s, with at least one of each. What are the last four
digits of n?
(A) 4444 (B) 4494 (C) 4944 (D) 9444 (E) 9944

© 2019 AoPS Incorporated 8


AoPS Community 2007 AMC 10

25 How many pairs of positive integers (a, b) are there such that gcd(a, b) = 1 and

a 14b
+
b 9a
is an integer?
(A) 4 (B) 6 (C) 9 (D) 12 (E) infinitely many


These problems are copyright © Mathematical Association of America (http://maa.org).

© 2019 AoPS Incorporated 9


Art of Problem Solving is an ACS WASC Accredited School.
AoPS Community 2008 AMC 10

AMC 10 2008
www.artofproblemsolving.com/community/c4806
by worthawholebean, mathgeniuseˆln(x), 123456789, Altheman, buzzer11, redcomet46, Eˆ(pi*i)=-1, in-
finity4ever, funmath, azjps, resurrection, archimedes1, andersonw, nickster08, TZF, CatalystOfNostalgia,
cognos599, undefined117, tenniskidperson3, n0vad3m0n, krsattack, rrusczyk

– A

– February 10th

1 A bakery owner turns on his doughnut machine at 8:30 AM. At 11:10 AM the machine has
completed one third of the day’s job. At what time will the doughnut machine complete the
job?
(A) 1:50 PM (B) 3:00 PM (C) 3:30 PM (D) 4:30 PM (E) 5:50 PM

2 A square is drawn inside a rectangle. The ratio of the width of the rectangle to a side of the
square is 2 : 1. The ratio of the rectangle’s length to its width is 2 : 1. What percent of the
rectangle’s area is inside the square?
(A) 12.5 (B) 25 (C) 50 (D) 75 (E) 87.5

3 For the positive integer n, let hni denote the sum of all the positive divisors of n with the ex-
ception of n itself. For example, h4i = 1 + 2 = 3 and h12i = 1 + 2 + 3 + 4 + 6 = 16 What is
hhh6iii?
(A) 6 (B) 12 (C) 24 (D) 32 (E) 36

4 Suppose that 32 of 10 bananas are worth as much as 8 oranges. How many oranges are worth
as much is 21 of 5 bananas?
(A) 2 (B) 52 (C) 3 (D) 72 (E) 4

5 Which of the following is equal to the product


8 12 16 4n + 4 2008
· · ··· ··· ?
4 8 12 4n 2004
(A) 251 (B) 502 (C) 1004 (D) 2008 (E) 4016

6 A triathlete competes in a triathlon in which the swimming, biking, and running segments are
all of the same length. The triathlete swims at a rate of 3 kilometers per hour, bikes at a rate
of 20 kilometers per hour, and runs at a rate of 10 kilometers per hour. Which of the following
is closest to the triathlete’s average speed, in kilometers per hour, for the entire race?
(A) 3 (B) 4 (C) 5 (D) 6 (E) 7

© 2019 AoPS Incorporated 1


AoPS Community 2008 AMC 10

7 The fraction
(32008 )2 − (32006 )2
(32007 )2 − (32005 )2
simplifies to which of the following?
9 9
(A) 1 (B) 4 (C) 3 (D) 2 (E) 9

8 Heather compares the price of a new computer at two different stores. Store A offers 15% off
the sticker price followed by a $90 rebate, and store B offers 25% off the same sticker price
with no rebate. Heather saves $15 by buying the computer at store A instead of store B. What
is the sticker price of the computer, in dollars?
(A) 750 (B) 900 (C) 1000 (D) 1050 (E) 1500

9 Suppose that
2x x

3 6
is an integer. Which of the following statements must be true about x?
(A) It is negative. (B) It is even, but not necessarily a multiple of 3. (C) It is a multiple of 3, but not nece
(D) It is a multiple of 6, but not necessarily a multiple of 12. (E) It is a multiple of 12.

10 Each of the sides of a square S1 with area 16 is bisected, and a smaller square S2 is constructed
using the bisection points as vertices. The same process is carried out on S2 to construct an
even smaller square S3 . What is the area of S3 ?
(A) 21 (B) 1 (C) 2 (D) 3 (E) 4

11 While Steve and LeRoy are fishing 1 mile from shore, their boat springs a leak, and water comes
in at a constant rate of 10 gallons per minute. The boat will sink if it takes in more than 30
gallons of water. Steve starts rowing toward the shore at a constant rate of 4 miles per hour
while LeRoy bails water out of the boat. What is the slowest rate, in gallons per minute, at
which LeRoy can bail if they are to reach the shore without sinking?
(A) 2 (B) 4 (C) 6 (D) 8 (E) 10

12 In a collection of red, blue, and green marbles, there are 25% more red marbles than blue mar-
bles, and there are 60% more green marbles than red marbles. Suppose that there are r red
marbles. What is the total number of marbles in that collection?
(A) 2.85r (B) 3r (C) 3.4r (D) 3.85r (E) 4.25r

13 Doug can paint a room in 5 hours. Dave can paint the same room in 7 hours. Doug and Dave
paint the room together and take a one-hour break for lunch. Let t be the total time, in hours,
required for them to complete the job working together, including lunch. Which of the following
equations is satisfied by t?
(A) 15 + 71  (t + 1) = 1 (B) 15 + 17 t + 1 = 1 (C) 15 + 17 t = 1
 

(D) 15 + 17 (t − 1) = 1 (E) (5 + 7)t = 1

© 2019 AoPS Incorporated 2


AoPS Community 2008 AMC 10

14 Older television screens have an aspect ratio of 4 : 3. That is, the ratio of the width to the
height is 4 : 3. The aspect ratio of many movies is not 4 : 3, so they are sometimes shown on
a television screen by ’letterboxing’ - darkening strips of equal height at the top and bottom of
the screen, as shown. Suppose a movie has an aspect ratio of 2 : 1 and is shown on an older
television screen with a 27-inch diagonal. What is the height, in inches, of each darkened strip?

(A) 2 (B) 2.25 (C) 2.5 (D) 2.7 (E) 3

15 Yesterday Han drove 1 hour longer than Ian at an average speed 5 miles per hour faster than
Ian. Jan drove 2 hours longer than Ian at an average speed 10 miles per hour faster than Ian.
Han drove 70 miles more than Ian. How many more miles did Jan drive than Ian?
(A) 120 (B) 130 (C) 140 (D) 150 (E) 160

16 Points A and B lie on a circle centered at O, and ∠AOB = 60◦ . A second circle is internally
tangent to the first and tangent to both OA and OB. What is the ratio of the area of the smaller
circle to that of the larger circle?
1 1 1 1 1
(A) 16 (B) 9 (C) 8 (D) 6 (E) 4

17 An equilateral triangle has side length 6. What is the area of the region containing all points
that are outside the triangle and not more than 3 units from a point of the triangle?
√ √ √ 2
(A) 36 + 24 3 (B) 54 + 9π (C) 54 + 18 3 + 6π (D) 2 3 + 3 π
√ 2
(E) 9 3 + 1 π

18 A right triangle has perimeter 32 and area 20. What is the length of its hypotenuse?
(A) 57
4 (B) 59
4 (C) 61
4 (D) 63
4 (E) 65
4

19 Rectangle P QRS lies in a plane with P Q = RS = 2 and QR = SP = 6. The rectangle is rotated


90◦ clockwise about R, then rotated 90◦ clockwise about the point that S moved to after the
first rotation.
√ √ What is the length of the path
√ traveled by √
point P√?
(A) (2√ 3 + 5)π (B) 6π (C) (3 + 10)π (D) ( 3 + 2 5)π
(E) 2 10π

20 Trapezoid ABCD has bases AB and CD and diagonals intersecting at K. Suppose that AB =
9, DC = 12, and the area of 4AKD is 24. What is the area of trapezoid ABCD?

© 2019 AoPS Incorporated 3


AoPS Community 2008 AMC 10

(A) 92 (B) 94 (C) 96 (D) 98 (E) 100

21 A cube with side length 1 is sliced by a plane that passes through two diagonally opposite
vertices A and C and the midpoints B and D of two opposite edges not containing A and C,
ac shown. What is the area of quadrilateral ABCD?


6 5
√ 3

(A) 2 (B) 4 (C) 2 (D) 2 (E) 3

22 Jacob uses the following procedure to write down a sequence of numbers. First he chooses
the first term to be 6. To generate each succeeding term, he flips a fair coin. If it comes up
heads, he doubles the previous term and subtracts 1. If it comes up tails, he takes half of the
previous term and subtracts 1. What is the probability that the fourth term in Jacob’s sequence
is an integer?
(A) 61 (B) 31 (C) 12 (D) 58 (E) 34

23 Two subsets of the set S = {a, b, c, d, e} are to be chosen so that their union is S and their
intersection contains exactly two elements. In how many ways can this be done, assuming
that the order in which the subsets are chosen does not matter?
(A) 20 (B) 40 (C) 60 (D) 160 (E) 320

24 Let k = 20082 + 22008 . What is the units digit of k 2 + 2k ?


(A) 0 (B) 2 (C) 4 (D) 6 (E) 8

25 A round table has radius 4. Six rectangular place mats are placed on the table. Each place mat
has width 1 and length x as shown. They are positioned so that each mat has two corners on
the edge of the table, these two corners being end points of the same side of length x. Further,

© 2019 AoPS Incorporated 4


AoPS Community 2008 AMC 10

the mats are positioned so that the inner corners each touch an inner corner of an adjacent
mat. What is x?

1
x

√ √ √ √
3 7− 3
√ √
5+2 3
(A) 2 5 − 3 (B) 3 (C) 2 (D) 2 3 (E) 2

– B

– February 27th

1 A basketball player made 5 baskets during a game. Each basket was worth either 2 or 3 points.
How many different numbers could represent the total points scored by the player?
(A) 2 (B) 3 (C) 4 (D) 5 (E) 6

2 A 4 × 4 block of calendar dates is shown. The order of the numbers in the second row is to
be reversed. Then the order of the numbers in the fourth row is to be reversed. Finally, the
numbers on each diagonal are to be added. What will be the positive difference between the
two diagonal sums?
1 2 3 4
8 9 10 11
15 16 17 18
22 23 24 25

(A) 2 (B) 4 (C) 6 (D) 8 (E) 10



Assume that x is a positive real number. Which is equivalent to
p
3
3 x x?
(A) x1/6 (B) x1/4 (C) x3/8 (D) x1/2 (E) x

4 A semipro baseball league has teams with 21 players each. League rules state that a player
must be paid at least $15, 000, and that the total of all players’ salaries for each team cannot
exceed $700, 000. What is the maximum possiblle salary, in dollars, for a single player?
(A) 270, 000 (B) 385, 000 (C) 400, 000 (D) 430, 000 (E) 700, 000

5 For real numbers a and b, define a$b = (a − b)2 . What is (x − y)2 $(y − x)2 ?
(A) 0 (B) x2 + y 2 (C) 2x2 (D) 2y 2 (E) 4xy

© 2019 AoPS Incorporated 5


AoPS Community 2008 AMC 10

6 Points B and C lie on AD. The length of AB is 4 times the length of BD, and the length of AC
is 9 times the length of CD. The length of BC is what fraction of the length of AD?
1 1 1 5
(A) 36 (B) 13 (C) 10 (D) 36 (E) 15

7 An equilateral triangle of side length 10 is completely filled in by non-overlapping equilateral


triangles of side length 1. How many small triangles are required?
(A) 10 (B) 25 (C) 100 (D) 250 (E) 1000

8 A class collects $50 to buy flowers for a classmate who is in the hospital. Roses cost $3 each,
and carnations cost $2 each. No other flowers are to be used. How many different bouquets
could be purchased for exactly $50?
(A) 1 (B) 7 (C) 9 (D) 16 (E) 17

9 A quadratic equation ax2 − 2ax + b = 0 has two real solutions. What is the average of the
solutions? √
(A) 1 (B) 2 (C) ab (D) 2b
a (E) 2b − a

10 Points A and B are on a circle of radius 5 and AB = 6. Point C is the midpoint of the minor
arc √
AB. What is the length
√ of the line√
segment AC?
(A) 10 (B) 27 (C) 14 (D) 15 (E) 4

11 Suppose that (un ) is a sequence of real numbers satisfying un+2 = 2un+1 + un , and that u3 = 9
and u6 = 128. What is u5 ?
(A) 40 (B) 53 (C) 68 (D) 88 (E) 104

12 Postman Pete has a pedometer to count his steps. The pedometer records up to 99999 steps,
then flips over to 00000 on the next step. Pete plans to determine his mileage for a year. On
January 1 Pete sets the pedometer to 00000. During the year, the pedometer flips from 99999
to 00000 forty-four times. On December 31 the pedometer reads 50000. Pete takes 1800 steps
per mile. Which of the following is closest to the number of miles Pete walked during the year?
(A) 2500 (B) 3000 (C) 3500 (D) 4000 (E) 4500

13 For each positive integer n, the mean of the first n terms of a sequence is n. What is the 2008th
term of the sequence?
(A) 2008 (B) 4015 (C) 4016 (D) 4, 030, 056 (E) 4, 032, 064

14 Triangle OAB has O = (0, 0), B = (5, 0), and A in the first quadrant. In addition, ∠ABO = 90◦
and ∠AOB = 30◦ . Suppose that OA is rotated 90◦ counterclockwise about O. What are the
coordinates
√ ofthe image of5 √
A?  √  √ 
10
(A) − 3 3, 5 (B) − 3 3, 5 (C) 3, 5 (D) 53 3, 5
√ 
(E) 10
3 3, 5

© 2019 AoPS Incorporated 6


AoPS Community 2008 AMC 10

15 How many right triangles have integer leg lengths a and b and a hypotenuse of length b + 1,
where b < 100?
(A) 6 (B) 7 (C) 8 (D) 9 (E) 10

16 Two fair coins are to be tossed once. For each head that results, one fair die is to be rolled.
What is the probability that the sum of the die rolls is odd? (Note that if no die is rolled, their
sum is 0.)
(A) 38 (B) 21 (C) 43
72 (D) 58 (E) 23

17 A poll shows that 70% of all voters approve of the mayor’s work. On three separate occasions
a pollster selects a voter at random. What is the probability that on exactly one of these three
occasions the voter approves of the mayor’s work?
(A) 0.063 (B) 0.189 (C) 0.233 (D) 0.333 (E) 0.441

18 Bricklayer Brenda would take 9 hours to build a chimney alone, and bricklayer Brandon would
take 10 hours to build it alone. When they work together they talk a lot, and their combined
output is decreased by 10 bricks per hour. Working together, they build the chimney in 5 hours.
How many bricks are in the chimney?
(A) 500 (B) 900 (C) 950 (D) 1000 (E) 1900

19 A cylindrical tank with radius 4 feet and height 9 feet is lying on its side. The tank is filled with
water to a depth
√ of 2 feet. What √ is the volume of the√water, in cubic feet?√
(A) 24π − 36√ 2 (B) 24π − 24 3 (C) 36π − 36 3 (D) 36π − 24 2
(E) 48π − 36 3

20 The faces of a cubical die are marked with the numbers 1, 2, 2, 3, 3, and 4. The faces of a second
cubical die are marked with the numbers 1, 3, 4, 5, 6, and 8. Both dice are thrown. What is the
probability that the sum of the two top numbers will be 5, 7, or 9 ?
5 7
(A) 18 (B) 18 (C) 11
18 (D) 34 (E) 89

21 Ten chairs are evenly spaced around a round table and numbered clockwise from 1 through
10. Five married couples are to sit in the chairs with men and women alternating, and no one is
to sit either next to or directly across from his or her spouse. How many seating arrangements
are possible?
(A) 240 (B) 360 (C) 480 (D) 540 (E) 720

22 Three red beads, two white beads, and one blue bead are placed in a line in random order. What
is the probability that no two neighboring beads are the same color?
1 1
(A) 12 (B) 10 (C) 16 (D) 13 (E) 12

23 A rectangular floor measures a by b feet, where a and b are positive integers with b > a. An

© 2019 AoPS Incorporated 7


AoPS Community 2008 AMC 10

artist paints a rectangle on the floor with the sides of the rectangle parallel to the sides of
the floor. The unpainted part of the floor forms a border of width 1 foot around the painted
rectangle and occupies half of the area of the entire floor. How many possibilities are there for
the ordered pair (a, b)?
(A) 1 (B) 2 (C) 3 (D) 4 (E) 5

24 Quadrilateral ABCD has AB = BC = CD, ∠ABC = 70◦ , and ∠BCD = 170◦ . What is the
degree measure of ∠BAD?
(A) 75 (B) 80 (C) 85 (D) 90 (E) 95

25 Michael walks at the rate of 5 feet per second on a long straight path. Trash pails are located
every 200 feet along the path. A garbage truck travels at 10 feet per second in the same direc-
tion as Michael and stops for 30 seconds at each pail. As Michael passes a pail, he notices
the truck ahead of him just leaving the next pail. How many times will Michael and the truck
meet?
(A) 4 (B) 5 (C) 6 (D) 7 (E) 8


These problems are copyright © Mathematical Association of America (http://maa.org).

© 2019 AoPS Incorporated 8


Art of Problem Solving is an ACS WASC Accredited School.
AoPS Community 2009 AMC 10

AMC 10 2009
www.artofproblemsolving.com/community/c4807
by AIME15, worthawholebean, 007math, Eˆ(pi*i)=-1, tenniskidperson3, mgao, abacadaea, rrusczyk

– A

– February 10th

1 One can holds 12 ounces of soda. What is the minimum number of cans to provide a gallon
(128 ounces) of soda?
(A) 7 (B) 8 (C) 9 (D) 10 (E) 11

2 Four coins are picked out of a piggy bank that contains a collection of pennies, nickels, dimes,
and quarters. Which of the following could not be the total value of the four coins, in cents?
(A) 15 (B) 25 (C) 35 (D) 45 (E) 55

3 Which of the following is equal to 1 + 1


1 ?
1+ 1+1
5 3 5
(A) 4 (B) 2 (C) 3 (D) 2 (E) 3

4 Eric plans to compete in a triathlon. He can average 2 miles per hour in the 14 -mile swim and
6 miles per hour in the 3-mile run. His goal is to finish the triathlon in 2 hours. To accomplish
his goal what must his average speed, in miles per hour, be for the 15-mile bicycle ride?
(A) 120
11 (B) 11 (C) 56
5 (D) 45
4 (E) 12

5 What is the sum of the digits of the square of 111, 111, 111?
(A) 18 (B) 27 (C) 45 (D) 63 (E) 81

6 A circle of radius 2 is inscribed in a semicircle, as shown. The area inside the semicircle but
outside the circle is shaded. What fraction of the semicircle’s area is shaded?

1 π 2 2 3
(A) 2 (B) 6 (C) π (D) 3 (E) π

© 2019 AoPS Incorporated 1


AoPS Community 2009 AMC 10

7 A carton contains milk that is 2% fat, and amount that is 40% less fat than the amount con-
tained in a carton of whole milk. What is the percentage of fat in whole milk?
(A) 12
5 (B) 3 (C) 10
3 (D) 38 (E) 42

8 Three generations of the Wen family are going to the movies, two from each generation. The
two members of the youngest generation receive a 50% discount as children. The two mem-
bers of the oldest generation receive a 25% discount as senior citizens. The two members of
the middle generation receive no discount. Grandfather Wen, whose senior ticket costs $6.00,
is paying for everyone. How many dollars must he pay?
(A) 34 (B) 36 (C) 42 (D) 46 (E) 48

9 Positive integers a, b, and 2009, with a < b < 2009, form a geometric sequence with an integer
ratio. What is a?
(A) 7 (B) 41 (C) 49 (D) 289 (E) 2009

10 Triangle ABC has a right angle at B. Point D is the foot of the altitude from B, AD = 3, and
DC = 4. What is the area of 4ABC?

A
3
D

B C

√ √ √
(A) 4 3 (B) 7 3 (C) 21 (D) 14 3 (E) 42

11 One dimension of a cube is increased by 1, another is decreased by 1, and the third is left
unchanged. The volume of the new rectangular solid is 5 less than that of the cube. What was
the volume of the cube?
(A) 8 (B) 27 (C) 64 (D) 125 (E) 216

12 In quadrilateral ABCD, AB = 5, BC = 17, CD = 5, DA = 9, and BD is an integer. What is


BD?

A
D

C B

© 2019 AoPS Incorporated 2


AoPS Community 2009 AMC 10

(A) 11 (B) 12 (C) 13 (D) 14 (E) 15

13 Suppose that P = 2m and Q = 3n . Which of the following is equal to 12mn for every pair of
integers (m, n)?
(A) P 2 Q (B) P n Qm (C) P n Q2m (D) P 2m Qn (E) P 2n Qm

14 Four congruent rectangles are placed as shown. The area of the outer square is 4 times that
of the inner square. What is the ratio of the length of the longer side of each rectangle to the
length of its shorter side?

√ √ √
(A) 3 (B) 10 (C) 2 + 2 (D) 2 3 (E) 4

15 The figures F1 , F2 , F3 , and F4 shown are the first in a sequence of figures. For n ≥ 3, Fn is
constructed from Fn−1 by surrounding it with a square and placing one more diamond on
each side of the new square than Fn−1 had on each side of its outside square. For example,
figure F3 has 13 diamonds. How many diamonds are there in figure F20 ?

F1 F2 F3 F4

(A) 401 (B) 485 (C) 585 (D) 626 (E) 761

16 Let a, b, c, and d be real numbers with |a − b| = 2, |b − c| = 3, and |c − d| = 4. What is the sum


of all possible values of |a − d|?
(A) 9 (B) 12 (C) 15 (D) 18 (E) 24

17 Rectangle ABCD has AB = 4 and BC = 3. Segment EF is constructed through B so that


EF is perpendicular to DB, and A and C lie on DE and DF , respectively. What is EF ?
(A) 9 (B) 10 (C) 125
12 (D) 103
9 (E) 12

© 2019 AoPS Incorporated 3


AoPS Community 2009 AMC 10

18 At Jefferson Summer Camp, 60% of the children play soccer, 30% of the children swim, and 40%
of the soccer players swim. To the nearest whole percent, what percent of the non-swimmers
play soccer?
(A) 30% (B) 40% (C) 49% (D) 51% (E) 70%

19 Circle A has radius 100. Circle B has an integer radius r < 100 and remains internally tangent
to circle A as it rolls once around the circumference of circle A. The two circles have the same
points of tangency at the beginning and end of circle B’s trip. How many possible values can
r have?
(A) 4 (B) 8 (C) 9 (D) 50 (E) 90

20 Andrea and Lauren are 20 kilometers apart. They bike toward one another with Andrea trav-
eling three times as fast as Lauren, and the distance between them decreasing at a rate of 1
kilometer per minute. After 5 minutes, Andrea stops biking because of a flat tire and waits for
Lauren. After how many minutes from the time they started to bike does Lauren reach Andrea?
(A) 20 (B) 30 (C) 55 (D) 65 (E) 80

21 Many Gothic cathedrals have windows with portions containing a ring of congruent circles
that are circumscribed by a larger circle, In the figure shown, the number of smaller circles is
four. What is the ratio of the sum of the areas of the four smaller circles to the area of the
larger circle?

√ √ √ √ √
(A) 3 − 2 2 (B) 2 − 2 (C) 4(3 − 2 2) (D) 12 (3 − 2) (E) 2 2 − 2

22 Two cubical dice each have removable numbers 1 through 6. The twelve numbers on the two
dice are removed, put into a bag, then drawn one at a time and randomly reattached to the
faces of the cubes, one number to each face. The dice are then rolled and the numbers on the
two top faces are added. What is the probability that the sum is 7?
(A) 19 (B) 81 (C) 16 2
(D) 11 (E) 15

23 Convex quadrilateral ABCD has AB = 9 and CD = 12. Diagonals AC and BD intersect at E,


AC = 14, and 4AED and 4BEC have equal areas. What is AE?
(A) 92 50
(B) 11 (C) 21
4 (D) 17
3 (E) 6

© 2019 AoPS Incorporated 4


AoPS Community 2009 AMC 10

24 Three distinct vertices of a cube are chosen at random. What is the probability that the plane
determined by these three vertices contains points inside the cube?
(A) 41 (B) 83 (C) 47 (D) 57 (E) 34

25 For k > 0, let Ik = 10 . . . 064, where there are k zeros between the 1 and the 6. Let N (k) be the
number of factors of 2 in the prime factorization of Ik . What is the maximum value of N (k)?
(A) 6 (B) 7 (C) 8 (D) 9 (E) 10

– B

– February 25th

1 Each morning of her five-day workweek, Jane bought either a 50-cent muffin or a 75-cent bagel.
Her total cost for the week was a whole number of dollars. How many bagels did she buy?
(A) 1 (B) 2 (C) 3 (D) 4 (E) 5

1
− 14
2 Which of the following is equal to 3
1 ?
2 − 13
1 1 1 2
(A) 4 (B) 3 (C) 2 (D) 3 (E) 34

3 Paula the painter had just enough paint for 30 identically sized rooms. Unfortunately, on the
way to work, three cans of paint fell of her truck, so she had only enough paint for 25 rooms.
How many cans of paint did she use for the 25 rooms?
(A) 10 (B) 12 (C) 15 (D) 18 (E) 25

4 A rectangular yard contains two flower beds in the shape of congruent isosceles right trian-
gles. THe remainder of the yard has a trapezoidal shape, as shown. The parallel sides of the
trapezoid have lengths 15 and 25 meters. What fraction of the yard is occupied by the flower
beds?

1 1 1 1 1
(A) 8 (B) 6 (C) 5 (D) 4 (E) 3

5 Twenty percent less than 60 is one-third more than what number?


(A) 16 (B) 30 (C) 32 (D) 36 (E) 48

6 Kiana has two older twin brothers. The product of their ages is 128. What is the sum of their
three ages?
(A) 10 (B) 12 (C) 16 (D) 18 (E) 24

© 2019 AoPS Incorporated 5


AoPS Community 2009 AMC 10

7 By inserting parentheses, it is possible to give the expression

2×3+4×5

several values. How many different values can be obtained?


(A) 2 (B) 3 (C) 4 (D) 5 (E) 6

8 In a certain year the price of gasoline rose by 20% during January, fell by 20% during February,
rose by 25% during March, and fell by x% during April. The price of gasoline at the end of April
was the same as it had been at the beginning of January. To the nearest integer, what is x?
(A) 12 (B) 17 (C) 20 (D) 25 (E) 35

9 Segment BD and AE intersect at C, as shown, AB = BC = CD = CE, and ∠A = 52 ∠B. What


is the degree measure of ∠D?

B
E

A
D

(A) 52.5 (B) 55 (C) 57.5 (D) 60 (E) 62.5

10 A flagpole is originally 5 meters tall. A hurricane snaps the flagpole at a point x meters above
the ground so that the upper part, still attached to the stump, touches the ground 1 meter away
from the base. What is x?
(A) 2.0 (B) 2.1 (C) 2.2 (D) 2.3 (E) 2.4

11 How many 7 digit palindromes (numbers that read the same backward as forward) can be
formed using the digits 2, 2, 3, 3, 5, 5, 5?
(A) 6 (B) 12 (C) 24 (D) 36 (E) 48

12 Distinct points A, B, C, and D lie on a line, with AB = BC = CD = 1. Points E and F lie on a


second line, parallel to the first, with EF = 1. A triangle with positive area has three of the six
points as its vertices. How many possible values are there for the area of the triangle?
(A) 3 (B) 4 (C) 5 (D) 6 (E) 7

13 As shown below, convex pentagon ABCDE has sides AB = 3, BC = 4, CD = 6, DE = 3,


and EA = 7. The pentagon is originally positioned in the plane with vertex A at the origin and

© 2019 AoPS Incorporated 6


AoPS Community 2009 AMC 10

vertex B on the positive x-axis. The pentagon is then rolled clockwise to the right along the
x-axis. Which side will touch the point x = 2009 on the x-axis?

E D
3
6

7
C
4

A 3 B
(0, 0) x

(A) AB (B) BC (C) CD (D) DE (E) EA

14 On Monday, Millie puts a quart of seeds, 25% of which are millet, into a bird feeder. On each
successive day she adds another quart of the same mix of seeds without removing any seeds
that are left. Each day the birds eat only 25% of the millet in the feeder, but they eat all of the
other seeds. On which day, just after Millie has placed the seeds, will the birds find that more
than half the seeds in the feeder are millet?
(A) Tuesday (B) Wednesday (C) Thursday (D) Friday (E) Saturday

15 When a bucket is two-thirds full of water, the bucket and water weigh a kilograms. When the
bucket is one-half full of water the total weight is b kilograms. In terms of a and b, what is the
total weight in kilograms when the bucket is full of water?
(A) 32 a + 13 b (B) 23 a − 12 b (C) 32 a + b (D) 32 a + 2b (E) 3a − 2b

16 Points A and C lie on a circle centered at O, each of BA and BC are tangent to the circle, and
4ABC is equilateral. The circle intersects BO at D. What is BD
BO ?
√ √ √ √
2 1 3 2 3
(A) 3 (B) 2 (C) 3 (D) 2 (E) 2

17 Five unit squares are arranged in the coordinate plane as shown, with the lower left corner at
the origin. The slanted line, extending from (a, 0) to (3, 3), divides the entire region into two
regions of equal area. What is a?

© 2019 AoPS Incorporated 7


AoPS Community 2009 AMC 10
y

(3, 3)

(a, 0) x

1 3 2 3 4
(A) 2 (B) 5 (C) 3 (D) 4 (E) 5

18 Rectangle ABCD has AB = 8 and BC = 6. Point M is the midpoint of diagonal AC, and E is
on AB with M E ⊥ AC. What is the area of 4AM E?
(A) 65
8 (B) 25
3 (C) 9 (D) 75
8 (E) 85
8

19 A particular 12-hour digital clock displays the hour and minute of a day. Unfortunately, when-
ever it is supposed to display a 1, it mistakenly displays a 9. For example, when it is 1:16 PM
the clock incorrectly shows 9:96 PM. What fraction of the day will the clock show the correct
time?
(A) 12 (B) 85 (C) 34 (D) 56 9
(E) 10

20 Triangle ABC has a right angle at B, AB = 1, and BC = 2. The bisector of ∠BAC meets BC
at D. What is BD?

B D C


3−1

5−1

5+1
√ √
6+ 2

(A) 2 (B) 2 (C) 2 (D) 2 (E) 2 3 − 1

© 2019 AoPS Incorporated 8


AoPS Community 2009 AMC 10

21 What is the remainder when 30 + 31 + 32 + . . . + 32009 is divided by 8?


(A) 0 (B) 1 (C) 2 (D) 4 (E) 6

22 A cubical cake with edge length 2 inches is iced on the sides and the top. It is cut vertically
into three pieces as shown in this top view, where M is the midpoint of a top edge. The piece
whose top is triangle B contains c cubic inches of cake and s square inches of icing. What is
c + s?

A
B
M
C

24 32
√ √
16 5

(A) 5 (B) 5 (C) 8 + 5 (D) 5 + 5 (E) 10 + 5 5

23 Rachel and Robert run on a circular track. Rachel runs counterclockwise and completes a lap
every 90 seconds, and Robert runs clockwise and completes a lap every 80 seconds. Both start
from the start line at the same time. At some random time between 10 minutes and 11 minutes
after they begin to run, a photographer standing inside the track takes a picture that shows
one-fourth of the track, centered on the starting line. What is the probability that both Rachel
and Robert are in the picture?
1
(A) 16 (B) 18 3
(C) 16 (D) 14 5
(E) 16

24 The keystone arch is an ancient architectural feature. It is composed of congruent isosceles


trapezoids fitted together along the non-parallel sides, as shown. The bottom sides of the two
end trapezoids are horizontal. In an arch made with 9 trapezoids, let x be the angle measure
in degrees of the larger interior angle of the trapezoid. What is x?

(A) 100 (B) 102 (C) 104 (D) 106 (E) 108

25 Each face of a cube is given a single narrow stripe painted from the center of one edge to the
center of its opposite edge. The choice of the edge pairing is made at random and indepen-
dently for each face. What is the probability that there is a continuous stripe encircling the

© 2019 AoPS Incorporated 9


AoPS Community 2009 AMC 10

cube?
(A) 18 (B) 3
16 (C) 1
4 (D) 3
8 (E) 1
2


These problems are copyright © Mathematical Association of America (http://maa.org).

© 2019 AoPS Incorporated 10


Art of Problem Solving is an ACS WASC Accredited School.
AoPS Community 2010 AMC 10

AMC 10 2010
www.artofproblemsolving.com/community/c4808
by worthawholebean, pythag011, Lord.of.AMC, Smartguy, BarbieRocks, ernie, limac, andersonw, rrusczyk

– A

– February 9th

1 Mary’s top book shelf holds five books with the following widths, in centimeters: 6, 12 , 1, 2.5,
and 10. What is the average book width, in centimeters?
(A) 1 (B) 2 (C) 3 (D) 4 (E) 5

2 Four identical squares and one rectangle are placed together to form one large square as
shown. The length of the rectangle is how many times as large as its width?

5 4 3
(A) 4 (B) 3 (C) 2 (D) 2 (E) 3

3 Tyrone had 97 marbles and Eric had 11 marbles. Tyrone then gave some of his marbles to Eric
so that Tyrone ended with twice as many marbles as Eric. How many marbles did Tyrone give
to Eric?
(A) 3 (B) 13 (C) 18 (D) 25 (E) 29

4 A book that is to be recorded onto compact discs takes 412 minutes to read aloud. Each disc
can hold up to 56 minutes of reading. Assume that the smallest possible number of discs is
used and that each disc contains the same length of reading. How many minutes of reading
will each disc contain?
(A) 50.2 (B) 51.5 (C) 52.4 (D) 53.8 (E) 55.2

5 The area of a circle whose circumference is 24π is kπ. What is the value of k?
(A) 6 (B) 12 (C) 24 (D) 36 (E) 144

© 2019 AoPS Incorporated 1


AoPS Community 2010 AMC 10

6 For positive numbers x and y the operation ♠(x, y) is defined as


1
♠(x, y) = x −
y

What is ♠(2, ♠(2, 2))?


(A) 32 (B) 1 (C) 4
3 (D) 5
3 (E) 2

7 Crystal has a running course marked out for her daily run. She starts this run by heading due
north for one mile. She then runs northeast for one mile, then southeast for one mile. The last
portion of her run takes her on a straight line back to where she started. How far, in miles is
this last portion
√ of her run?
√ √
(A) 1 (B) 2 (C) 3 (D) 2 (E) 2 2

8 Tony works 2 hours a day and is paid $0.50 per hour for each full year of his age. During a six
month period Tony worked 50 days and earned $630. How old was Tony at the end of the six
month period?
(A) 9 (B) 11 (C) 12 (D) 13 (E) 14

9 A palindrome, such as 83438, is a number that remains the same when its digits are reversed.
The numbers x and x + 32 are three-digit and four-digit palindromes, respectively. What is the
sum of the digits of x?
(A) 20 (B) 21 (C) 22 (D) 23 (E) 24

10 Marvin had a birthday on Tuesday, May 27 in the leap year 2008. In what year will his birthday
next fall on a Saturday?
(A) 2011 (B) 2012 (C) 2013 (D) 2015 (E) 2017

11 The length of the interval of solutions of the inequality a ≤ 2x + 3 ≤ b is 10. What is b − a?


(A) 6 (B) 10 (C) 15 (D) 20 (E) 30

12 Logan is constructing a scaled model of his town. The city’s water tower stands 40 meters
high, and the top portion is a sphere that holds 100, 000 liters of water. Logan’s miniature water
tower holds 0.1 liters. How tall, in meters, should Logan make his tower?
0.4 4
(A) 0.04 (B) π (C) 0.4 (D) π (E) 4

13 Angelina drove at an average rate of 80 kph and then stopped 20 minutes for gas. After the
stop, she drove at an average rate of 100 kph. Altogether she drove 250 km in a total trip time
of 3 hours including the stop. Which equation could be used to solve for the time t in hours
that she drove before her stop?
(A) 80t + 100(8/3 − t) = 250 (B) 80t = 250 (C) 100t = 250
(D) 90t = 250 (E) 80(8/3 − t) + 100t = 250

© 2019 AoPS Incorporated 2


AoPS Community 2010 AMC 10

14 Triangle ABC has AB = 2 · AC. Let D and E be on AB and BC, respectively, such that
∠BAE = ∠ACD. Let F be the intersection of segments AE and CD, and suppose that 4CF E
is equilateral. What is ∠ACB?
(A) 60◦ (B) 75◦ (C) 90◦ (D) 105◦ (E) 120◦

15 In a magical swamp there are two species of talking amphibians: toads, whose statements
are always true, and frogs, whose statements are always false. Four amphibians, Brian, Chris,
LeRoy, and Mike live together in the swamp, and they make the following statements:

Brian: ”Mike and I are different species.”


Chris: ”LeRoy is a frog.”
LeRoy: ”Chris is a frog.”
Mike: ”Of the four of us, at least two are toads.”

How many of these amphibians are frogs?


(A) 0 (B) 1 (C) 2 (D) 3 (E) 4

16 Nondegenerate 4ABC has integer side lengths, BD is an angle bisector, AD = 3, and DC = 8.


What is the smallest possible value of the perimeter?
(A) 30 (B) 33 (C) 35 (D) 36 (E) 37

17 A solid cube has side length 3 inches. A 2-inch by 2-inch square hole is cut into the center of
each face. The edges of each cut are parallel to the edges of the cube, and each hole goes all
the way through the cube. What is the volume, in cubic inches, of the remaining solid?
(A) 7 (B) 8 (C) 10 (D) 12 (E) 15

18 Bernardo randomly picks 3 distinct numbers from the set {1, 2, 3, 4, 5, 6, 7, 8, 9} and arranges
them in descending order to form a 3-digit number. Silvia randomly picks 3 distinct numbers
from the set {1, 2, 3, 4, 5, 6, 7, 8} and also arranges them in descending order to form a 3-digit
number. What is the probability that Bernardo’s number is larger than Silvia’s number?
47
(A) 72 (B) 37
56 (C) 23 (D) 49
72 (E) 39
56

19 Equiangular hexagon ABCDEF has side lengths AB = CD = EF = 1 and BC = DE =


F A = r. The area of 4ACE is 70% of the area of the hexagon. What is the sum of all possible
values of r?

4 3 10 17
(A) 3 (B) 3 (C) 4 (D) 4 (E) 6

20 A fly trapped inside a cubical box with side length 1 meter decides to relieve its boredom by
visiting each corner of the box. It will begin and end in the same corner and visit each of the
other corners exactly once. To get from a corner to any other corner, it will either fly or crawl
in a straight
√ line. What is√the maximum
√ possible√length,
√ in meters, √
of its path?

(A) 4 + 4 2 (B) 2 + 4 2 + 2 3 (C) 2 + 3 2 + 3 3 (D) 4 2 + 4 3

© 2019 AoPS Incorporated 3


AoPS Community 2010 AMC 10
√ √
(E) 3 2 + 5 3

21 The polynomial x3 − ax2 + bx − 2010 has three positive integer zeros. What is the smallest
possible value of a?
(A) 78 (B) 88 (C) 98 (D) 108 (E) 118

22 Eight points are chosen on a circle, and chords are drawn connecting every pair of points. No
three chords intersect in a single point inside the circle. How many triangles with all three
vertices in the interior of the circle are created?
(A) 28 (B) 56 (C) 70 (D) 84 (E) 140

23 Each of 2010 boxes in a line contains a single red marble, and for 1 ≤ k ≤ 2010, the box in the
kth position also contains k white marbles. Isabella begins at the first box and successively
draws a single marble at random from each box, in order. She stops when she first draws a red
marble. Let P (n) be the probability that Isabella stops after drawing exactly n marbles. What
is the smallest value of n for which P (n) < 2010
1
?
(A) 45 (B) 63 (C) 64 (D) 201 (E) 1005

24 The number obtained from the last two nonzero digits of 90! is equal to n. What is n?
(A) 12 (B) 32 (C) 48 (D) 52 (E) 68

25 Jim starts with a positive integer n and creates a sequence of numbers. Each successive
number is obtained by subtracting the largest possible integer square less than or equal to
the current number until zero is reached. For example, if Jim starts with n = 55, then his
sequence contains 5 numbers:
55
2
55 − 7 = 6
6 − 22 = 2
2 − 12 = 1
1 − 12 = 0
Let N be the smallest number for which Jim’s sequence has 8 numbers. What is the units digit
of N ?
(A) 1 (B) 3 (C) 5 (D) 7 (E) 9

– B

– February 24th

1 What is 100(100 − 3) − (100 · 100 − 3)?


(A) − 20, 000 (B) − 10, 000 (C) − 297 (D) − 6 (E) 0

© 2019 AoPS Incorporated 4


AoPS Community 2010 AMC 10

2 Makayla attended two meetings during her 9-hour work day. The first meeting took 45 minutes
and the second meeting took twice as long. What percent of her work day was spent attending
meetings?
(A) 15 (B) 20 (C) 25 (D) 30 (E) 35

3 A drawer contains red, green, blue, and white socks with at least 2 of each color. What is the
minimum number of socks that must be pulled from the drawer to guarantee a matching pair?
(A) 3 (B) 4 (C) 5 (D) 8 (E) 9

4 For a real number x, define ♥(x) to be the average of x and x2 . What is ♥(1) + ♥(2) + ♥(3)?
(A) 3 (B) 6 (C) 10 (D) 12 (E) 20

5 A month with 31 days has the same number of Mondays and Wednesdays. How many of the
seven days of the week could be the first day of this month?
(A) 2 (B) 3 (C) 4 (D) 5 (E) 6

6 A circle is centered at O, AB is a diameter and C is a point on the circle with ∠COB = 50◦ .
What is the degree measure of ∠CAB?
(A) 20 (B) 25 (C) 45 (D) 50 (E) 65

7 A triangle has side lengths 10, 10, and 12. A rectangle has width 4 and area equal to the area
of the triangle. What is the perimeter of this rectangle?
(A) 16 (B) 24 (C) 28 (D) 32 (E) 36

8 A ticket to a school play costs x dollars, where x is a whole number. A group of 9th graders
buys tickets costing a total of $48, and a group of 10th graders buys tickets costing a total of
$64. How many values of x are possible?
(A) 1 (B) 2 (C) 3 (D) 4 (E) 5

9 Lucky Larry’s teacher asked him to substitute numbers for a, b, c, d, and e in the expression
a − (b − (c − (d + e))) and evaluate the result. Larry ignored the parentheses but added and
subtracted correctly and obtained the correct result by coincedence. The numbers Larry sub-
stituted for a, b, c, and d were 1, 2, 3, and 4, respectively. What number did Larry substitute for
e?
(A) − 5 (B) − 3 (C) 0 (D) 3 (E) 5

10 Shelby drives her scooter at a speed of 30 miles per hour if it is not raining, and 20 miles per
hour if it is raining. Today she drove in the sun in the morning and in the rain in the evening,
for a total of 16 miles in 40 minutes. How many minutes did she drive in the rain?
(A) 18 (B) 21 (C) 24 (D) 27 (E) 30

© 2019 AoPS Incorporated 5


AoPS Community 2010 AMC 10

11 A shopper plans to purchase an item that has a listed price greater than $100 and can use
any one of the three coupons. Coupon A gives 15% off the listed price, Coupon B gives $30 the
listed price, and Coupon C gives 25% off the amount by which the listed price exceeds $100.
Let x and y be the smallest and largest prices, respectively, for which Coupon A saves at least
as many dollars as Coupon B or C. What is y − x?
(A) 50 (B) 60 (C) 75 (D) 80 (E) 100

12 At the beginning of the school year, 50% of all students in Mr. Well’s math class answered ”Yes”
to the question ”Do you love math”, and 50% answered ”No.” At the end of the school year, 70%
answered ”Yes” and 30% answered ”No.” Altogether, x% of the students gave a different answer
at the beginning and end of the school year. What is the difference between the maximum and
the minimum possible values of x?
(A) 0 (B) 20 (C) 40 (D) 60 (E) 80

13 What is the sum of all the solutions of x = |2x − |60 − 2x||?


(A) 32 (B) 60 (C) 92 (D) 120 (E) 124

14 The average of the numbers 1, 2, 3, ..., 98, 99, and x is 100x. What is x?
49 50
(A) 101 (B) 101 (C) 12 51
(D) 101 50
(E) 99

15 On a 50-question multiple choice math contest, students receive 4 points for a correct answer,
0 points for an answer left blank, and -1 point for an incorrect answer. Jesse’s total score on the
contest was 99. What is the maximum number of questions that Jesse could have answered
correctly?
(A) 25 (B) 27 (C) 29 (D) 31 (E) 33

16 A square of side length 1 and a circle of radius 3/3 share the same center. What is the area
inside the circle, but outside
√ the square?
π 2π 3 π
(A) 3 − 1 (B) 9 − 3 (C) 18 (D) 14 (E) 2π/9

17 Every high school in the city of Euclid sent a team of 3 students to a math contest. Each
participant in the contest received a different score. Andrea’s score was the median among
all students, and hers was the highest score on her team. Andrea’s teammates Beth and Carla
placed 37th and 64th, respectively. How many schools are in the city?
(A) 22 (B) 23 (C) 24 (D) 25 (E) 26

18 Positive integers a, b, and c are randomly and independently selected with replacement from
the set {1, 2, 3, . . . , 2010}. What is the probability that abc + ab + a is divisible by 3?
1 29 31 11 13
(A) (B) (C) (D) (E)
3 81 81 27 27

© 2019 AoPS Incorporated 6


AoPS Community 2010 AMC 10

19 A circle √
with center O has area 156π. Triangle ABC is equilateral, BC is a chord on the circle,
OA = √4 3, and point O is √outside 4ABC. What is the side length of 4ABC?
(A) 2 3 (B) 6 (C) 4 3 (D) 12 (E) 18

20 Two circles lie outside regular hexagon ABCDEF . The first is tangent to AB, and the second
is tangent to DE. Both are tangent to lines BC and F A. What is the ratio of the area of the
second circle to that of the first circle?
(A) 18 (B) 27 (C) 36 (D) 81 (E) 108

21 A palindrome between 1000 and 10, 000 is chosen at random. What is the probability that it is
divisible by 7?
1 1 1 1 1
(A) (B) (C) (D) (E)
10 9 7 6 5

22 Seven distinct pieces of candy are to be distributed among three bags. The red bag and the
blue bag must each receive at least one piece of candy; the white bag may remain empty. How
many arrangements are possible?
(A) 1930 (B) 1931 (C) 1932 (D) 1933 (E) 1934

23 The entries in a 3 × 3 array include all the digits from 1 through 9, arranged so that the entries
in every row and column are in increasing order. How many such arrays are there?
(A) 18 (B) 24 (C) 36 (D) 42 (E) 60

24 A high school basketball game between the Raiders and Wildcats was tied at the end of the
first quarter. The number of points scored by the Raiders in each of the four quarters formed
an increasing geometric sequence, and the number of points scored by the Wildcats in each of
the four quarters formed an increasing arithmetic sequence. At the end of the fourth quarter,
the Raiders had won by one point. Neither team scored more than 100 points. What was the
total number of points scored by the two teams in the first half?
(A) 30 (B) 31 (C) 32 (D) 33 (E) 34

25 Let a > 0, and let P (x) be a polynomial with integer coefficients such that
P (1) = P (3) = P (5) = P (7) = a, and
P (2) = P (4) = P (6) = P (8) = −a.
What is the smallest possible value of a?
(A) 105 (B) 315 (C) 945 (D) 7! (E) 8!


These problems are copyright © Mathematical Association of America (http://maa.org).

© 2019 AoPS Incorporated 7


AoPS Community 2010 AMC 10

© 2019 AoPS Incorporated 8


Art of Problem Solving is an ACS WASC Accredited School.
AoPS Community 2011 AMC 10

AMC 10 2011
www.artofproblemsolving.com/community/c4809
by Binomial-theorem, redcomet46, rrusczyk

– A

1 A cell phone plan costs $20 each month, plus 5 per text message sent, plus 10 for each minute
used over 30 hours. In January Michelle sent 100 text messages and talked for 30.5 hours. How
much did she have to pay?
(A)$24.00 (B)$24.50 (C)$25.50 (D)$28.00 (E)$30.00

2 A small bottle of shampoo can hold 35 milliliters of shampoo, whereas a large bottle can hold
500 milliliters of shampoo. Jasmine wants to buy the minimum number of small bottles nec-
essary to completely fill a large bottle. How many bottles must she buy?
(A) 11 (B) 12 (C) 13 (D) 14 (E) 15

3 Suppose [a b] denotes the average of a and b, and {a b c} denotes the average of a, b, and c.
What is {{1 1 0} [0 1] 0}?
2 5 1 7 2
(A) 9 (B) 18 (C) 3 (D) 18 (E) 3

4 Let X and Y be the following sums of arithmetic sequences:

X = 10 + 12 + 14 + · · · + 100,
Y = 12 + 14 + 16 + · · · + 102.

What is the value of Y − X?


(A) 92 (B) 98 (C) 100 (D) 102 (E) 112

5 At an elementary school, the students in third grade, fourth grade, and fifth grade run an aver-
age of 12, 15, and 10 minutes per day, respectively. There are twice as many third graders as
fourth graders, and twice as many fourth graders as fifth graders. What is the average number
of minutes run per day by these students?
37 88
(A) 12 (B) 3 (C) 7 (D) 13 (E) 14

6 Set A has 20 elements, and set B has 15 elements. What is the smallest possible number of
elements in A ∪ B, the union of A and B?
(A) 5 (B) 15 (C) 20 (D) 35 (E) 300

© 2019 AoPS Incorporated 1


AoPS Community 2011 AMC 10

7 Which of the following equations does NOT have a solution?


√ √
(A) (x + 7)2 = 0 (B) | − 3x| + 5 = 0 (C) −x − 2 = 0 (D) x − 8 = 0 (E) | − 3x| − 4 = 0

8 Last summer 30% of the birds living on Town Lake were geese, 25% were swans, 10% were
herons, and 35% were ducks. What percent of the birds that were not swans were geese?
(A) 20 (B) 30 (C) 40 (D) 50 (E) 60

9 A rectangular region is bounded by the graphs of the equations y = a, y = −b, x = −c, and
x = d, where a, b, c, and d are all positive numbers. Which of the following represents the area
of this region?
(A) ac + ad + bc + bd (B) ac − ad + bc − bd (C) ac + ad − bc − bd (D) − ac − ad +
bc + bd (E) ac − ad − bc + bd

10 A majority of the 30 students in Ms. Deameanor’s class bought pencils at the school book-
store. Each of these students bought the same number of pencils, and this number was greater
than 1. The cost of a pencil in cents was greater than the number of pencils each student
bought, and the total cost of all the pencils was $17.71. What was the cost of a pencil in cents?
(A) 7 (B) 11 (C) 17 (D) 23 (E) 77

11 Square EF GH has one vertex on each side of square ABCD. Point E is on AB with AE =
7 · EB. What is the ratio of the area of EF GH to the area of ABCD?
√ √
49
(A) 64 (B) 25
32 (C) 78 (D) 5 8 2 (E) 14
4

12 The players on a basketball team made some three-point shots, some two-point shots, and
some one-point free throws. They scored as many points with two-point shots as with three-
point shots. Their number of successful free throws was one more than their number of suc-
cessful two-point shots. The team’s total score was 61 points. How many free throws did they
make?
(A) 13 (B) 14 (C) 15 (D) 16 (E) 17

13 How many even integers are there between 200 and 700 whose digits are all different and
come from the set 1,2,5,7,8,9?
(A) 12 (B) 20 (C) 72 (D) 120 (E) 200

14 A pair of standard 6-sided fair dice is rolled once. The sum of the numbers rolled determines
the diameter of a circle. What is the probability that the numerical value of the area of the circle
is less than the numerical value of the circle’s circumference?
1 1
(A) 36 (B) 12 (C) 16 (D) 14 5
(E) 18

© 2019 AoPS Incorporated 2


AoPS Community 2011 AMC 10

15 Roy bought a new battery-gasoline hybrid car. On a trip the car ran exclusively on its battery
for the first 40 miles, then ran exclusively on gasoline for the rest of the trip, using gasoline at
a rate of 0.02 gallons per mile. On the whole trip he averaged 55 miles per gallon. How long
was the trip in miles?
(A) 140 (B) 240 (C) 440 (D) 640 (E) 840
p √ p √
16 Which of the following is equal to 9 − 6 2 + 9 + 6 2?
√ √ √ √
(A) 3 2 (B) 2 6 (C) 7 2 2 (D) 3 3 (E) 6

17 In the eight-term sequence A, B, C, D, E, F, G, H, the value of C is 5 and the sum of any three
consecutive terms is 30. What is A + H?
(A) 17 (B) 18 (C) 25 (D) 26 (E) 43

18 Circles A, B, and C each have radius 1. Circles A and B share one point of tangency. Circle C
has a point of tangency with the midpoint of AB. What is the area inside Circle C but outside
circle A and circle B ?

A B

π π 3π π
(A) 3 − 2 (B) 2 (C) 2 (D) 4 (E) 1 + 2

19 In 1991 the population of a town was a perfect square. Ten years later, after an increase of
150 people, the population was 9 more than a perfect square. Now, in 2011, with an increase
of another 150 people, the population is once again a perfect square. Which of the following is
closest to the percent growth of the town’s population during this twenty-year period?
(A) 42 (B) 47 (C) 52 (D) 57 (E) 62

20 Two points on the circumference of a circle of radius r are selected independently and at ran-
dom. From each point a chord of length r is drawn in a clockwise direction. What is the prob-
ability that the two chords intersect?

© 2019 AoPS Incorporated 3


AoPS Community 2011 AMC 10
1 1 1 1 1
(A) 6 (B) 5 (C) 4 (D) 3 (E) 2

21 Two counterfeit coins of equal weight are mixed with 8 identical genuine coins. The weight
of each of the counterfeit coins is different from the weight of each of the genuine coins. A
pair of coins is selected at random without replacement from the 10 coins. A second pair is
selected at random without replacement from the remaining 8 coins. The combined weight of
the first pair is equal to the combined weight of the second pair. What is the probability that
all 4 selected coins are genuine?
7 9 11 15 15
(A) 11 (B) 13 (C) 15 (D) 19 (E) 16

22 Each vertex of convex pentagon ABCDE is to be assigned a color. There are 6 colors to choose
from, and the ends of each diagonal must have different colors. How many different colorings
are possible?
(A) 2520 (B) 2880 (C) 3120 (D) 3250 (E) 3750

23 Seven students count from 1 to 1000 as follows:


-Alice says all the numbers, except she skips the middle number in each consecutive group of
three numbers. That is, Alice says 1, 3, 4, 6, 7, 9, · · · , 997, 999, 1000.
-Barbara says all of the numbers that Alice doesn’t say, except she also skips the middle num-
ber in each consecutive group of three numbers.
-Candice says all of the numbers that neither Alice nor Barbara says, except she also skips the
middle number in each consecutive group of three numbers.
-Debbie, Eliza, and Fatima say all of the numbers that none of the students with the first names
beginning before theirs in the alphabet say, except each also skips the middle number in each
of her consecutive groups of three numbers.
-Finally, George says the only number that no one else says.
What number does George say?
(A) 37 (B) 242 (C) 365 (D) 728 (E) 998

24 Two distinct regular tetrahedra have all their vertices among the vertices of the same unit
cube. What is the volume of the region formed by the intersection of the tetrahedra?
√ √ √
1 2 3 1 2
(A) 12 (B) 12 (C) 12 (D) 6 (E) 6

25 Let R be a square region and n ≥ 4 an integer. A point X in the interior of R is called n-ray
partitional if there are n rays emanating from X that divide R into n triangles of equal area.
How many points are 100-ray partitional but not 60-ray partitional?
(A) 1500 (B) 1560 (C) 2320 (D) 2480 (E) 2500

© 2019 AoPS Incorporated 4


AoPS Community 2011 AMC 10

– B

1 What is
2+4+6 1+3+5
− ?
1+3+5 2+4+6
5 7 147 43
(A) − 1 (B) 36 (C) 12 (D) 60 (E) 3

2 Josanna’s test scores to date are 90, 80, 70, 60, and 85. Her goal is to raise her test average at
least 3 points with her next test. What is the minimum test score she would need to accomplish
this goal?
(A) 80 (B) 82 (C) 85 (D) 90 (E) 95

3 At a store, when a length is reported as x inches that means the length is at least x − 0.5
inches and at most x + 0.5 inches. Suppose the dimensions of a rectangular tile are reported
as 2 inches by 3 inches. In square inches, what is the minimum area for the rectangle?
(A) 3.75 (B) 4.5 (C) 5 (D) 6 (E) 8.75

4 LeRoy and Bernardo went on a week-long trip together and agreed to share the costs equally.
Over the week, each of them paid for various joint expenses such as gasoline and car rental.
At the end of the trip it turned out that LeRoy had paid A dollars and Bernardo had paid B
dollars, where A < B. How many dollars must LeRoy give to Bernardo so that they share the
costs equally?
A+B A−B B−A
(A) 2 (B) 2 (C) 2 (D) B − A (E) A + B

5 In multiplying two positive integers a and b, Ron reversed the digits of the two-digit number a.
His errorneous product was 161. What is the correct value of the product of a and b?
(A) 116 (B) 161 (C) 204 (D) 214 (E) 224

6 On Halloween Casper ate 13 of his candies and then gave 2 candies to his brother. The next day
he ate 13 of his remaining candies and then gave 4 candies to his sister. On the third day he ate
his final 8 candies. How many candies did Casper have at the beginning?
(A) 30 (B) 39 (C) 48 (D) 57 (E) 66

7 The sum of two angles of a triangle is 56 of a right angle, and one of these two angles is 30◦
larger than the other. What is the degree measure of the largest angle in the triangle?
(A) 69 (B) 72 (C) 90 (D) 102 (E) 108

© 2019 AoPS Incorporated 5


AoPS Community 2011 AMC 10

8 At a certain beach if it is at least 80◦ F and sunny, then the beach will be crowded. On June 10
the beach was not crowded. What can be said about the weather conditions on June 10?
(A) The temperature was cooler than 80◦ F and it was not sunny. (B) The temperature was
cooler than 80◦ F or it was not sunny. (C) If the temperature was at least 80◦ F , then it was
sunny. (D) If the temperature was cooler than 80◦ F , then it was sunny. (E) If the temperature
was cooler than 80◦ F , then it was not sunny.

9 The area of 4EBD is one third of the area of 3 − 4 − 5 4ABC. Segment DE is perpendicular
to segment AB. What is BD?

C
E
3 4

A 5D B

4
√ 9

4 3 5
(A) 3 (B) 5 (C) 4 (D) 3 (E) 2

10 Consider the set of numbers {1, 10, 102 , 103 , ...1010 }. The ratio of the largest element of the set
to the sum of the other ten elements of the set is closest to which integer?
(A) 1 (B) 9 (C) 10 (D) 11 (E) 101

11 There are 52 people in a room. What is the largest value of n such that the statement ”At least
n people in this room have birthdays falling in the same month” is always true?
(A) 2 (B) 3 (C) 4 (D) 5 (E) 12

12 Keiko walks once around a track at exactly the same constant speed every day. The sides of
the track are straight, and the ends are semicircles. The track has width 6 meters, and it takes
her 36 seconds longer to walk around the outside edge of the track than around the inside
edge. What is Keiko’s speed in meters per second?
π 2π 4π 5π
(A) 3 (B) 3 (C) π (D) 3 (E) 3

13 Two real numbers are selected independently at random from the interval [-20, 10]. What is the
probability that the product of those numbers is greater than zero?
1 1 4 5 2
(A) 9 (B) 3 (C) 9 (D) 9 (E) 3

© 2019 AoPS Incorporated 6


AoPS Community 2011 AMC 10

14 A rectangular parking lot has a diagonal of 25 meters and an area of 168 square meters. In
meters, what is the perimeter of the parking lot?
(A) 52 (B) 58 (C) 62 (D) 68 (E) 70

15 Let @ denote the ”averaged with” operation: a @ b = 2 .


a+b
Which of the following distributive
laws hold for all numbers x, y and z?
I. x @ (y+z) = (x @ y) + (x @ z)
II. x + (y @ z) = (x + y) @ (x + z)
III. x @ (y @ z) = (x @ y) @ (x @ z)
(A) I only (B) II only (C) III only (D) I and III only (E) II and III only

16 A dart board is a regular octagon divided into regions as shown. Suppose that a dart thrown
at the board is equally likely to land anywhere on the board. What is probability that the dart
lands within the center square?


2−1 1

2− 2

2

(A) 2 (B) 4 (C) 2 (D) 4 (E) 2 − 2

17 In the given circle, the diameter EB is parallel to DC, and AB is parallel to ED. The angles
AEB and ABE are in the ratio 4 : 5. What is the degree measure of angle BCD?

E B

D C

© 2019 AoPS Incorporated 7


AoPS Community 2011 AMC 10

(A) 120 (B) 125 (C) 130 (D) 135 (E) 140

18 Rectangle ABCD has AB = 6 and BC = 3. Point M is chosen on side AB so that ∠AM D =


∠CM D. What is the degree measure of ∠AM D?
(A) 15 (B) 30 (C) 45 (D) 60 (E) 75

19 What is the product of all the roots of the equation


p p
5|x| + 8 = x2 − 16.

(A) − 64 (B) − 24 (C) − 9 (D) 24 (E) 576

20 Rhombus ABCD has side length 2 and ∠B = 120◦ . Region R consists of all points inside the
rhombus that are closer to vertex B than any of the other three vertices. What is the area of
R?
√ √ √ √
3 3 2 3 3
(A) 3 (B) 2 (C) 3 (D) 1 + 3 (E) 2

21 Brian writes down four integers w > x > y > z whose sum is 44. The pairwise positive dif-
ferences of these numbers are 1, 3, 4, 5, 6, and 9. What is the sum of the possible values for
w?
(A) 16 (B) 31 (C) 48 (D) 62 (E) 93

22 A pyramid has a square base with sides of length 1 and has lateral faces that are equilateral
triangles. A cube is placed within the pyramid so that one face is on the base of the pyramid
and its opposite face has all its edges on the lateral faces of the pyramid. What is the volume
of this cube?
√ √ √ √ √
(A) 5 2 − 7 (B) 7 − 4 3 (C) 2272 (D) 92 (E) 93

23 What is the hundreds digit of 20112011 ?


(A) 1 (B) 4 (C) 5 (D) 6 (E) 9

24 A lattice point in an xy-coordinate system is any point (x, y) where both x and y are integers.
The graph of y = mx + 2 passes through no lattice point with 0 < x ≤ 100 for all m such that
2 < m < a. What is the maximum possible value of a?
1

51 50 51 52 13
(A) 101 (B) 99 (C) 100 (D) 101 (E) 25

25 Let T1 be a triangle with sides 2011, 2012, and 2013. For n ≥ 1, if Tn = 4ABC and D, E, and F
are the points of tangency of the incircle of 4ABC to the sides AB, BC and AC, respectively,
then Tn+1 is a triangle with side lengths AD, BE, and CF , if it exists. What is the perimeter of
the last triangle in the sequence (Tn )?

© 2019 AoPS Incorporated 8


AoPS Community 2011 AMC 10
1509 1509 1509 1509 1509
(A) 8 (B) 32 (C) 64 (D) 128 (E) 256


These problems are copyright © Mathematical Association of America (http://maa.org).

© 2019 AoPS Incorporated 9


Art of Problem Solving is an ACS WASC Accredited School.
AoPS Community 2012 AMC 10

AMC 10 2012
www.artofproblemsolving.com/community/c4810
by bluecarneal, inquisitivity, Draco, Mrdavid445, anonymous0, alex31415, narto928, rrusczyk

– A

1 Cagney can frost a cupcake every 20 seconds and Lacey can frost a cupcake every 30 seconds.
Working together, how many cupcakes can they frost in 5 minutes?
(A) 10 (B) 15 (C) 20 (D) 25 (E) 30

2 A square with side length 8 is cut in half, creating two congruent rectangles. What are the
dimensions of one of these rectangles?
(A) 2 by 4 (B) 2 by 6 (C) 2 by 8 (D) 4 by 4 (E) 4 by 8

3 A bug crawls along a number line, starting at −2. It crawls to −6, then turns around and crawls
to 5. How many units does the bug crawl altogether?
(A) 9 (B) 11 (C) 13 (D) 14 (E) 15

4 Let ∠ABC = 24◦ and ∠ABD = 20◦ . What is the smallest possible degree measure for ∠CBD?
(A) 0 (B) 2 (C) 4 (D) 6 (E) 12

5 Last year 100 adult cats, half of whom were female, were brought into the Smallville Animal
Shelter. Half of the adult female cats were accompanied by a litter of kittens. The average
number of kittens per litter was 4. What was the total number of cats and kittens received by
the shelter last year?
(A) 150 (B) 200 (C) 250 (D) 300 (E) 400

6 The product of two positive numbers is 9. The reciprocal of one of these numbers is 4 times
the reciprocal of the other number. What is the sum of the two numbers?
10 20 15
(A) (B) (C) 7 (D) (E) 8
3 3 2

7 In a bag of marbles, 35 of the marbles are blue and the rest are red. If the number of red marbles
is doubled and the number of blue marbles stays the same, what fraction of the marbles will
be red?
2 3 4 3 4
(A) (B) (C) (D) (E)
5 7 7 5 5

© 2019 AoPS Incorporated 1


AoPS Community 2012 AMC 10

8 The sums of three whole numbers taken in pairs are 12, 17, and 19. What is the middle number?
(A) 4 (B) 5 (C) 6 (D) 7 (E) 8

9 A pair of six-sided fair dice are labeled so that one die has only even numbers (two each of 2,
4, and 6), and the other die has only odd numbers (two each of 1, 3, and 5). The pair of dice is
rolled. What is the probability that the sum of the numbers on top of the two dice is 7?
1 1 1 1 1
(A) (B) (C) (D) (E)
6 5 4 3 2

10 Mary divides a circle into 12 sectors. The central angles of these sectors, measured in degrees,
are all integers and they form an arithmetic sequence. What is the degree measure of the
smallest possible sector angle?
(A) 5 (B) 6 (C) 8 (D) 10 (E) 12

11 Externally tangent circles with centers at points A and B have radii of lengths 5 and 3, respec-
tively. A line externally tangent to both circles intersects ray AB at point C. What is BC?
(A) 4 (B) 4.8 (C) 10.2 (D) 12 (E) 14.4

12 A year is a leap year if and only if the year number is divisible by 400 (such as 2000) or is divisible
by 4 but not by 100 (such as 2012). The 200th anniversary of the birth of novelist Charles Dickens
was celebrated on February 7, 2012, a Tuesday. On what day of the week was Dickens born?
(A) Friday (B) Saturday (C) Sunday (D) Monday (E) Tuesday

13 An iterative average of the numbers 1, 2, 3, 4, and 5 is computed in the following way. Arrange
the five numbers in some order. Find the mean of the first two numbers, then find the mean of
that with the third number, then the mean of that with the fourth number, and finally the mean
of that with the fifth number. What is the difference between the largest and smallest possible
values that can be obtained using this procedure?
31 17 65
(A) 16 (B) 2 (C) 8 (D) 3 (E) 16

14 Chubby makes nonstandard checkerboards that have 31 squares on each side. The checker-
boards have a black square in every corner and alternate red and black squares along every
row and column. How many black squares are there on such a checkerboard?
(A) 480 (B) 481 (C) 482 (D) 483 (E) 484

15 Three unit squares and two line segments connecting two pairs of vertices are shown. What
is the area of 4ABC?

© 2019 AoPS Incorporated 2


AoPS Community 2012 AMC 10

A B


1 1 2 1 2
(A) 6 (B) 5 (C) 9 (D) 3 (E) 4

16 Three runners start running simultaneously from the same point on a 500-meter circular track.
They each run clockwise around the course maintaining constant speeds of 4.4, 4.8, and 5.0
meters per second. The runners stop once they are all together again somewhere on the cir-
cular course. How many seconds do the runners run?
(A) 1, 000 (B) 1, 250 (C) 2, 500 (D) 5, 000 (E) 10, 000

a3 −b3
17 Let a and b be relatively prime integers with a > b > 0 and (a−b)3
= 3 .
73
What is a − b?

(A) 1 (B) 2 (C) 3 (D) 4 (E) 5

18 The closed curve in the figure is made up of 9 congruent circular arcs each of length 2π
3 , where
each of the centers of the corresponding circles is among the vertices of a regular hexagon of
side 2. What is the area enclosed by the curve?

© 2019 AoPS Incorporated 3


AoPS Community 2012 AMC 10

◦ ◦

◦ ◦

√ √ √
(A) 2π + 6 (B) 2π + 4 3 (C) 3π + 4 (D) 2π + 3 3 + 2 (E) π + 6 3

19 Paula the painter and her two helpers each paint at constant, but different, rates. They always
start at 8:00 AM, and all three always take the same amount of time to eat lunch. On Monday
the three of them painted 50% of a house, quitting at 4:00 PM. On Tuesday, when Paula wasn’t
there, the two helpers painted only 24% of the house and quit at 2:12 PM. On Wednesday Paula
worked by herself and finished the house by working until 7:12 PM. How long, in minutes, was
each day’s lunch break?
(A) 30 (B) 36 (C) 42 (D) 48 (E) 60

20 A 3×3 square is partitioned into 9 unit squares. Each unit square is painted either white or black
with each color being equally likely, chosen independently and at random. The square is the
rotated 90◦ clockwise about its center, and every white square in a position formerly occupied
by a black square is painted black. The colors of all other squares are left unchanged. What is
the probability that the grid is now entirely black?
49 7 121 81 9
(A) (B) (C) (D) (E)
512 64 1024 512 32

21 Let points A = (0, 0, 0), B = (1, 0, 0), C = (0, 2, 0), and D = (0, 0, 3). Points E, F, G, and H are
midpoints of line segments BD, AB, AC, and DC respectively. What is the area of EF GH?
√ √ √ √ √
(A) 2 (B) 2 3 5 (C) 3 4 5 (D) 3 (E) 2 3 7

22 The sum of the first m positive odd integers is 212 more than the sum of the first n positive
even integers. What is the sum of all possible values of n?
(A) 255 (B) 256 (C) 257 (D) 258 (E) 259

23 Adam, Benin, Chiang, Deshawn, Esther, and Fiona have internet accounts. Some, but not all, of

© 2019 AoPS Incorporated 4


AoPS Community 2012 AMC 10

them are internet friends with each other, and none of them has an internet friend outside this
group. Each of them has the same number of internet friends. In how many different ways can
this happen?
(A) 60 (B) 170 (C) 290 (D) 320 (E) 660

24 Let a, b, and c be positive integers with a ≥ b ≥ c such that

a2 − b2 − c2 + ab = 2011 and
a2 + 3b2 + 3c2 − 3ab − 2ac − 2bc = −1997

What is a?
(A) 249 (B) 250 (C) 251 (D) 252 (E) 253

25 Real numbers x, y, and z are chosen independently and at random from the interval [0, n] for
some positive integer n. The probability that no two of x, y, and z are within 1 unit of each other
is greater than 21 . What is the smallest possible value of n?
(A) 7 (B) 8 (C) 9 (D) 10 (E) 11

– B

1 Each third-grade classroom at Pearl Creek Elementary has 18 students and 2 pet rabbits. How
many more students than rabbits are there in all 4 of the third-grade classrooms?
(A) 48 (B) 56 (C) 64 (D) 72 (E) 80

2 A circle of radius 5 is inscribed in a rectangle as shown. The ratio of the the length of the
rectangle to its width is 2 : 1. What is the area of the rectangle?

(A) 50 (B) 100 (C) 125 (D) 150 (E) 200

3 The point in the xy-plane with coordinates (1000, 2012) is reflected across line y = 2000. What
are the coordinates of the reflected point?
(A) (998, 2012) (B) (1000, 1988) (C) (1000, 2024) (D) (1000, 4012) (E) (1012, 2012)

© 2019 AoPS Incorporated 5


AoPS Community 2012 AMC 10

4 When Ringo places his marbles into bags with 6 marbles per bag, he has 4 marbles left over.
When Paul does the same with his marbles, he has 3 marbles left over. Ringo and Paul pool
their marbles and place them into as many bags as possible, with 6 marbles per bag. How
many marbles will be left over?
(A) 1 (B) 2 (C) 3 (D) 4 (E) 5

5 Anna enjoys dinner at a restaurant in Washington, D.C., where the sales tax on meals is 10%.
She leaves a 15% tip on the prices of her meal before the sales tax is added, and the tax is
calculated on the pre-tip amount. She spends a total of 27.50 for dinner. What is the cost of
here dinner without tax or tip?
(A) $18 (B) $20 (C) $21 (D) $22 (E) $24

6 In order to estimate the value of x − y where x and y are real numbers with x > y > 0, Xiaoli
rounded x up by a small amount, rounded y down by the same amount, and then subtracted
her values. Which of the following statements is necessarily correct?
(A) Her estimate is larger than x − y
(B) Her estimate is smaller than x − y
(C) Her estimate equals x − y
(D) Her estimate equals y − x
(E) Her estimate is 0

7 For a science project, Sammy observed a chipmunk and a squirrel stashing acorns in holes.
The chipmunk hid 3 acorns in each of the holes it dug. The squirrel hid 4 acorns in each of the
holes it dug. They each hid the same
number of acorns, although the squirrel needed 4 fewer holes. How many acorns did the chip-
munk hide?
(A) 30 (B) 36 (C) 42 (D) 48 (E) 54

8 What is the sum of all integer solutions to 1 < (x − 2)2 < 25?
(A) 10 (B) 12 (C) 15 (D) 19 (E) 25

9 Two integers have a sum of 26. When two more integers are added to the first two integers the
sum is 41. Finally when two more integers are added to the sum of the previous four integers
the sum is 57. What is the minimum number of even integers among the 6 integers?
(A) 1 (B) 2 (C) 3 (D) 4 (E) 5

10 How many ordered pairs of positive integers (M, N ) staisfy the equation M
6 = N?
6

(A) 6 (B) 7 (C) 8 (D) 9 (E) 10

© 2019 AoPS Incorporated 6


AoPS Community 2012 AMC 10

11 A dessert chef prepares the dessert for every day of a week starting with Sunday. The dessert
each day is either cake, pie, ice cream, or pudding. The same dessert may not be served two
days in a row. There must be cake on Friday because of a birthday. How many different dessert
menus for the week are possible?
(A) 729 (B) 972 (C) 1024 (D) 2187 (E) 2304

12 Point B is due√east of point A. Point C is due north of point B. The distance between points
A and C is 10 2 meters, and ∠BAC = 45◦ . Point D is 20 meters due north of point C. The
distance AD is between which two integers?
(A) 30 and 31 (B) 31 and 32 (C) 32 and 33 (D) 33 and 34 (E) 34 and 35

13 It takes Clea 60 seconds to walk down an escalator when it is not operating and only 24 sec-
onds to walk down the escalator when it is operating. How many seconds does it take Clea to
ride down the operating escalator when she just stands on it?
(A) 36 (B) 40 (C) 42 (D) 48 (E) 52

14 Two equilateral triangles are contained in a square whose side length is 2 3. The bases of
these triangles are the opposite sides of the square, and their intersection is a rhombus. What
is the area of the rhombus?
√ √ √ √
(A) 32 (B) 3 (C) 2 2 − 1 (D) 8 3 − 12 (E) 4 3 3

15 In a round-robin tournament with 6 teams, each team plays one game against each other team,
and each game results in one team winning and one team losing. At the end of the tournament,
the teams are ranked by the number of games won. What is the maximum number of teams
that could be tied for the most wins at the end of the tournament?
(A) 2 (B) 3 (C) 4 (D) 5 (E) 6

16 Three circles with radius 2 are mutually tangent. What is the total area of the circles and the
region bounded by them, as shown in the figure?

© 2019 AoPS Incorporated 7


AoPS Community 2012 AMC 10

√ √ √
(A) 10π + 4 3 (B) 13π − 3 (C) 12π + 3 (D) 10π + 9 (E) 13π

17 Jesse cuts a circular paper disk of radius 12 along two radii to form two sectors, the smaller
having a central angle of 120 degrees. He makes two circular cones, using each sector to form
the lateral surface of a cone. What is the ratio of the volume of the smaller cone to that of the
larger?
√ √ √
1 1 10 5 10
(A) 8 (B) 4 (C) 10 (D) 6 (E) 5

18 Suppose that one of every 500 people in a certain population has a particular disease, which
displays no symptoms. A blood test is available for screening for this disease. For a person
who has this disease, the test always turns out positive. For a person who does not have the
disease, however, there is a 2% false positive rate; in other words, for such people, 98% of the
time the test will turn out negative, but 2% of the time the test will turn out positive and will
incorrectly indicate that the person has the disease. Let p be the probability that a person
who is chosen at random from the population and gets a positive test result actually has the
disease. Which of the following is closest to p?
1 1 1 49 98
(A) 98 (B) 9 (C) 11 (D) 99 (E) 99

19 In rectangle ABCD, AB = 6, AD = 30, and G is the midpoint of AD. Segment AB is extended


2 units beyond B to point E, and F is the intersection of ED and BC. What is the area of
BF DG?
133 135 137
(A) 2 (B) 67 (C) 2 (D) 68 (E) 2

20 Bernado and Silvia play the following game. An integer between 0 and 999, inclusive, is se-
lected and given to Bernado. Whenever Bernado receives a number, he doubles it and passes
the result to Silvia. Whenever Silvia receives a number, she adds 50 to it and passes the result
to Bernado. The winner is the last person who produces a number less than 1000. Let N be
the smallest initial number that results in a win for Bernado. What is the sum of the digits of

© 2019 AoPS Incorporated 8


AoPS Community 2012 AMC 10

N?
(A) 7 (B) 8 (C) 9 (D) 10 (E) 11

21 Four distinct points are arranged in a plane so that the segments connecting them has lengths
a, a, a, a, 2a, and b. What is the ratio of b to a?
√ √
(A) 3 (B) 2 (C) 5 (D) 3 (E) π

22 Let (a1 , a2 , . . . , a10 ) be a list of the first 10 positive integers such that for each 2 ≤ i ≤ 10 either
ai + 1 or ai − 1 or both appear somewhere before ai in the list. How many such lists are there?
(A) 120 (B) 512 (C) 1024 (D) 181, 440 (E) 362, 880

23 A solid tetrahedron is sliced off a solid wooden unit cube by a plane passing through two
nonadjacent vertices on one face and one vertex on the opposite face not adjacent to either
of the first two vertices. The tetrahedron is discarded and the remaining portion of the cube is
placed on a table with the cut surface face down. What is the height of this object?
√ √ √
3 2 2 2 3 √
(A) (B) (C) 1 (D) (E) 2
3 3 3

24 Amy, Beth, and Jo listen to four different songs and discuss which ones they like. No song is
liked by all three. Furthermore, for each of the three pairs of the girls, there is at least one song
liked by those two girls but disliked by the third. In how many different ways is this possible?
(A) 108 (B) 132 (C) 671 (D) 846 (E) 1105

25 A bug travels from A to B along the segments in the hexagonal lattice pictured below. The
segments marked with an arrow can be traveled only in the direction of the arrow, and the bug
never travels the same segment more than once. How many different paths are there?

© 2019 AoPS Incorporated 9


AoPS Community 2012 AMC 10

A B

(A) 2112 (B) 2304 (C) 2368 (D) 2384 (E) 2400


These problems are copyright © Mathematical Association of America (http://maa.org).

© 2019 AoPS Incorporated 10


Art of Problem Solving is an ACS WASC Accredited School.
AoPS Community 2013 AMC 10

AMC 10 2013
www.artofproblemsolving.com/community/c4811
by aZpElr68Cb51U51qy9OM, fortenforge, ahaanomegas, countyguy, djmathman, El Ectric, Yoshi, mss-
math, KingSmasher3, program4, electron, tc1729, ProblemSolver1026, Aequilipse, antimonyarsenide, rr-
usczyk

– A

– February 5th

1 A taxi ride costs $1.50 plus $0.25 per mile traveled. How much does a 5-mile taxi ride cost?
(A) $2.25 (B) $2.50 (C) $2.75 (D) $3.00 (E) $3.25

2 Alice is making a batch of cookies and needs 2 12 cups of sugar. Unforunately, her measuring
cup holds only 14 cup of sugar. How many times must she fill that cup to get the correct amount
of sugar?
(A) 8 (B) 10 (C) 12 (D) 16 (E) 20

3 Square ABCD has side length 10. Point E is on BC, and the area of 4ABE is 40. What is BE?
(A) 4 (B) 5 (C) 6 (D) 7 (E) 8

B E C

A D

4 A softball team played ten games, scoring 1, 2, 3, 4, 5, 6, 7, 8, 9, and 10 runs. They lost by one
run in exactly five games. In each of the other games, they scored twice as many runs as their
opponent. How many total runs did their opponents score?
(A) 35 (B) 40 (C) 45 (D) 50 (E) 55

© 2020 AoPS Incorporated 1


AoPS Community 2013 AMC 10

5 Tom, Dorothy, and Sammy went on a vacation and agreed to split the costs evenly. During
their trip Tom paid $105, Dorothy paid $125, and Sammy paid $175. In order to share the costs
equally, Tom gave Sammy t dollars, and Dorothy gave Sammy d dollars. What is t − d?
(A) 15 (B) 20 (C) 25 (D) 30 (E) 35

6 Joey and his five brothers are ages 3, 5, 7, 9, 11, and 13. One afternoon two of his brothers
whose ages sum to 16 went to the movies, two brothers younger than 10 went to play baseball,
and Joey and the 5-year-old stayed home. How old is Joey?
(A) 3 (B) 7 (C) 9 (D) 11 (E) 13

7 A student must choose a program of four courses from a menu of courses consisting of En-
glish, Algebra, Geometry, History, Art, and Latin. This program must contain English and at
least one mathematics course. In how many ways can this program be chosen?
(A) 6 (B) 8 (C) 9 (D) 12 (E) 16

8 What is the value of


22014 + 22012
?
22014 − 22012
5
(A) − 1 (B) 1 (C) 3 (D) 2013 (E) 24024

9 In a recent basketball game, Shenille attempted only three-point shots and two-point shots.
She was successful on 20% of her three-point shots and 30% of her two-point shots. Shenille
attempted 30 shots. How many points did she score?
(A) 12 (B) 18 (C) 24 (D) 30 (E) 36

10 A flower bouquet contains pink roses, red roses, pink carnations, and red carnations. One third
of the pink flowers are roses, three fourths of the red flowers are carnations, and six tenths of
the flowers are pink. What percent of the flowers are carnations?
(A) 15 (B) 30 (C) 40 (D) 60 (E) 70

11 A student council must select a two-person welcoming committee and a three-person plan-
ning committee from among its members. There are exactly 10 ways to select a two-person
team for the welcoming committee. It is possible for students to serve on both committees.
In how many different ways can a three-person planning committee be selected?
(A) 10 (B) 12 (C) 15 (D) 18 (E) 25

© 2020 AoPS Incorporated 2


AoPS Community 2013 AMC 10

12 In 4ABC, AB = AC = 28 and BC = 20. Points D, E, and F are on sides AB, BC, and
AC, respectively, such that DE and EF are parallel to AC and AB, respectively. What is the
perimeter of parallelogram ADEF ?

B E C

(A) 48 (B) 52 (C) 56 (D) 60 (E) 72

13 How many three-digit numbers are not divisible by 5, have digits that sum to less than 20, and
have the first digit equal to the third digit?
(A) 52 (B) 60 (C) 66 (D) 68 (E) 70

14 A solid cube of side length 1 is removed from each corner of a solid cube of side length 3. How
many edges does the remaining solid have?
(A) 36 (B) 60 (C) 72 (D) 84 (E) 108

15 Two sides of a triangle have lengths 10 and 15. The length of the altitude to the third side is
the average of the lengths of the altitudes to the two given sides. How long is the third side?
(A) 6 (B) 8 (C) 9 (D) 12 (E) 18

16 A triangle with vertices (6, 5), (8, −3), and (9, 1) is reflected about the line x = 8 to create a
second triangle. What is the area of the union of the two triangles?
28 31 32
(A) 9 (B) (C) 10 (D) (E)
3 3 3

17 Daphne is visited periodically by her three best friends: Alice, Beatrix, and Claire. Alice visits
every third day, Beatrix visits every fourth day, and Claire visits every fifth day. All three friends

© 2020 AoPS Incorporated 3


AoPS Community 2013 AMC 10

visited Daphne yesterday. How many days of the next 365-day period will exactly two friends
visit her?
(A) 48 (B) 54 (C) 60 (D) 66 (E) 72

18 Let points A = (0, 0), B = (1, 2), C = (3, 3), and D = (4, 0). Quadrilateral ABCD
 iscut into
equal area pieces by a line passing through A. This line intersects CD at point pq , rs , where
these fractions are in lowest terms. What is p + q + r + s?
(A) 54 (B) 58 (C) 62 (D) 70 (E) 75

19 In base 10, the number 2013 ends in the digit 3. In base 9, on the other hand, the same number
is written as (2676)9 and ends in the digit 6. For how many positive integers b does the base-b
representation of 2013 end in the digit 3?
(A) 6 (B) 9 (C) 13 (D) 16 (E) 18

20 A unit square is rotated 45◦ about its center. What is the area of the region swept out by the
interior of the square?
√ √ √ √
(A) 1 − 22 + π4 (B) 12 + π4 (C) 2 − 2 + π4 (D) 22 + π4 (E) 1 + 42 + π8

21 A group of 12 pirates agree to divide a treasure chest of gold coins among themselves as
follows. The k th pirate to take a share takes 12
k
of the coins that remain in the chest. The
number of coins initially in the chest is the smallest number for which this arrangement will
allow each pirate to receive a positive whole number of coins. How many coins does the 12th
pirate receive?
(A) 720 (B) 1296 (C) 1728 (D) 1925 (E) 3850

22 Six spheres of radius 1 are positioned so that their centers are at the vertices of a regular
hexagon of side length 2. The six spheres are internally tangent to a larger sphere whose center
is the center of the hexagon. An eighth sphere is externally tangent to the six smaller spheres
and internally tangent to the larger sphere. What is the radius of this eighth sphere?
√ √
(A) 2 (B) 32 (C) 35 (D) 3 (E) 2

23 In 4ABC, AB = 86, and AC = 97. A circle with center A and radius AB intersects BC at
points B and X. Moreover BX and CX have integer lengths. What is BC?
(A) 11 (B) 28 (C) 33 (D) 61 (E) 72

24 Central High School is competing against Northern High School in a backgammon match.
Each school has three players, and the contest rules require that each player play two games
against each of the other’s school’s players. The match takes place in six rounds, with three

© 2020 AoPS Incorporated 4


AoPS Community 2013 AMC 10

games played simultaneously in each round. In how many different ways can the match be
scheduled?
(A) 540 (B) 600 (C) 720 (D) 810 (E) 900

25 All diagonals are drawn in a regular octagon. At how many distinct points in the interior of the
octagon (not on the boundary) do two or more diagonals intersect?
(A) 49 (B) 65 (C) 70 (D) 96 (E) 128

– B

– February 20th

1 What is 2+4+6
1+3+5 − 2+4+6 ?
1+3+5

5 7 49 43
(A) − 1 (B) 36 (C) 12 (D) 20 (E) 3

2 Mr Green measures his rectangular garden by walking two of the sides and finds that it is 15
steps by 20 steps. Each or Mr Green’s steps is two feet long. Mr Green expect half a pound
of potatoes per square foot from his garden. How many pounds of potatoes does Mr Green
expect from his garden?
(A) 600 (B) 800 (C) 1000 (D) 1200 (E) 1400

3 On a particular January day, the high temperature in Lincoln, Nebraska, was 16 degrees higher
than the low temperature, and the average of the high and low temperatures was 3◦ . In degrees,
what was the low temperature in Lincoln that day?
(A) − 13 (B) − 8 (C) − 5 (D) 3 (E) 11

4 When counting from 3 to 201, 53 is the 51st number counted. When counting backwards from
201 to 3, 53 is the nth number counted. What is n?
(A) 146 (B) 147 (C) 148 (D) 149 (E) 150

5 Positive integers a and b are each less than 6. What is the smallest possible value for 2·a−a·b?
(A) − 20 (B) − 15 (C) − 10 (D) 0 (E) 2

6 The average age of 33 fifth-graders is 11. The average age of 55 of their parents is 33. What is
the average age of all of these parents and fifth-graders?
(A) 22 (B) 23.25 (C) 24.75 (D) 26.25 (E) 28

7 Six points are equally spaced around a circle of radius 1. Three of these points are the vertices
of a triangle that is neither equilateral nor isosceles. What is the area of this triangle?

© 2020 AoPS Incorporated 5


AoPS Community 2013 AMC 10

3

3

(A) 3 (B) 2 (C) 1 (D) 2 (E) 2

8 Ray’s car averages 40 miles per gallon of gasoline, and Tom’s car averages 10 miles per gallon
of gasoline. Ray and Tom each drive the same number of miles. What is the cars’ combined
rate of miles per gallon of gasoline?
(A) 10 (B) 16 (C) 25 (D) 30 (E) 40

9 Three positive integers are each greater than 1, have a product of 27000, and are pairwise rel-
atively prime. What is their sum?
(A) 100 (B) 137 (C) 156 (D) 160 (E) 165

10 A basketball team’s players were successful on 50% of their two-point shots and 40% of their
three-point shots, which resulted in 54 points. They attempted 50% more two-point shots than
three-point shots. How many three-point shots did they attempt?
(A) 10 (B) 15 (C) 20 (D) 25 (E) 30

11 Real numbers x and y satisfy the equation x2 + y 2 = 10x − 6y − 34. What is x + y?


(A) 1 (B) 2 (C) 3 (D) 6 (E) 8

12 Let S be the set of sides and diagonals of a regular pentagon. A pair of elements of S are se-
lected at random without replacement. What is the probability that the two chosen segments
have the same length?
2 4 1 5 4
(A) 5 (B) 9 (C) 2 (D) 9 (E) 5

13 Jo and Blair take turns counting from 1 to one more than the last number said by the other
person. Jo starts by saying ”1”, so Blair follows by saying ”1, 2”. Jo then says ”1, 2, 3”, and so
on. What is the 53rd number said?
(A) 2 (B) 3 (C) 5 (D) 6 (E) 8

14 Define a♣b = a2 b − ab2 . Which of the following describes the set of points (x, y) for which
x♣y = y♣x?
(A) a finite set of points (B) one line (C) two parallel lines (D two intersecting lines
(E) three lines

15 A wire is cut into two pieces, one of length a and the other of length b. The piece of length a is
bent to form an equilateral triangle, and the piece of length b is bent to form a regular hexagon.
The triangle and the hexagon have equal area. What is ab ?
√ √ √
(A) 1 (B) 26 (C) 3 (D 2 (E) 3 2 2

© 2020 AoPS Incorporated 6


AoPS Community 2013 AMC 10

16 In 4ABC, medians AD and CE intersect at P , P E = 1.5, P D = 2, and DE = 2.5. What is the


area of AEDC?

D
P

A E B

(A) 13 (B) 13.5 (C) 14 (D) 14.5 (E) 15

17 Alex has 75 red tokens and 75 blue tokens. There is a booth where Alex can give two red to-
kens and receive in return a silver token and a blue token, and another booth where Alex can
give three blue tokens and receive in return a silver token and a red token. Alex continues to
exchange tokens until no more exchanges are possible. How many silver tokens will Alex have
at the end?
(A) 62 (B) 82 (C) 83 (D 102 (E) 103

18 The number 2013 has the property that its units digit is the sum of its other digits, that is
2 + 0 + 1 = 3. How many integers less than 2013 but greater than 1000 share this property?
(A) 33 (B) 34 (C) 45 (D) 46 (E) 58

19 The real numbers c, b, a form an arithmetic sequence with a ≥ b ≥ c ≥ 0. The quadratic


ax2 + bx + c has exactly one root. What is this root?
√ √ √ √
(A) − 7 − 4 3 (B) − 2 − 3 (C) − 1 (D) − 2 + 3 (E) − 7 + 4 3

20 The number 2013 is expressed in the form


a1 !a2 ! · · · am !
2013 = ,
b1 !b2 ! · · · bn !
where a1 ≥ a2 ≥ · · · ≥ am and b1 ≥ b2 ≥ · · · ≥ bn are positive integers and a1 + b1 is as small
as possible. What is |a1 − b1 |?
(A) 1 (B) 2 (C) 3 (D 4 (E) 5

21 Two non-decreasing sequences of nonnegative integers have different first terms. Each se-
quence has the property that each term beginning with the third is the sum of the previous

© 2020 AoPS Incorporated 7


AoPS Community 2013 AMC 10

two terms, and the seventh term of each sequence is N . What is the smallest possible value
of N ?
(A) 55 (B) 89 (C) 104 (D 144 (E) 273

22 The regular octagon ABCDEF GH has its center at J. Each of the vertices and the center are
to be associated with one of the digits 1 through 9, with each digit used once, in such a way
that the sums of the numbers on the lines AJE, BJF , CJG, and DJH are equal. In how many
ways can this be done?
(A) 384 (B) 576 (C) 1152 (D) 1680 (E) 3546

A B

H C

G D

F E

23 In triangle ABC, AB = 13, BC = 14, and CA = 15. Distinct points D, E, and F lie on segments
BC, CA, and DE, respectively, such that AD ⊥ BC, DE ⊥ AC, and AF ⊥ BF . The length of
segment DF can be written as m n , where m and n are relatively prime positive integers. What
is m + n?
(A) 18 (B) 21 (C) 24 (D 27 (E) 30

24 A positive integer n is nice if there is a positive integer m with exactly four positive divisors
(including 1 and m) such that the sum of the four divisors is equal to n. How many numers in
the set {2010, 2011, 2012, . . . , 2019} are nice?
(A) 1 (B) 2 (C) 3 (D 4 (E) 5

25 Bernardo chooses a three-digit positive integer N and writes both its base-5 and base-6 rep-
resentations on a blackboard. Later LeRoy sees the two numbers Bernardo has written. Treat-
ing the two numbers as base-10 integers, he adds them to obtain an integer S. For exam-
ple, if N = 749, Bernardo writes the numbers 10,444 and 3,245, and LeRoy obtains the sum

© 2020 AoPS Incorporated 8


AoPS Community 2013 AMC 10

S = 13, 689. For how many choices of N are the two rightmost digits of S, in order, the same
as those of 2N ?
(A) 5 (B) 10 (C) 15 (D 20 (E) 25


These problems are copyright © Mathematical Association of America (http://maa.org).

© 2020 AoPS Incorporated 9


Art of Problem Solving is an ACS WASC Accredited School.
AoPS Community 2014 AMC 10

AMC 10 2014
www.artofproblemsolving.com/community/c4812
by AlcumusGuy, El Ectric, djmathman, bestwillcui1, alex31415, niraekjs, brandbest1, Royalreter1, bobthes-
martypants, sunny2000, flamefoxx99, professordad, Quadratic64, mathman523, rrusczyk

– A

– February 4th

1 −1
What is 10 · 1 1

1 2 + 5 + 10 ?
25 170
(A) 3 (B) 8 (C) 2 (D) 3 (E) 170

2 Roy’s cat eats 13 of a can of cat food every morning and 41 of a can of cat food every evening.
Before feeding his cat on Monday morning, Roy opened a box containing 6 cans of cat food.
On what day of the week did the cat finish eating all the cat food in the box?
(A) Tuesday (B) Wednesday (C) Thursday (D) Friday (E) Saturday

3 Bridget bakes 48 loaves of bread for her bakery. She sells half of them in the morning for $2.50
each. In the afternoon she sells two thirds of what she has left, and because they are not
fresh, she charges only half price. In the late afternoon she sells the remaining loaves at a
dollar each. Each loaf costs $0.75 for her to make. In dollars, what is her profit for the day?
(A) 24 (B) 36 (C) 44 (D) 48 (E) 52

4 Walking down Jane Street, Ralph passed four houses in a row, each painted a different color.
He passed the orange house before the red house, and he passed the blue house before the
yellow house. The blue house was not next to the yellow house. How many orderings of the
colored houses are possible?
(A) 2 (B) 3 (C) 4 (D) 5 (E) 6

5 On an algebra quiz, 10% of the students scored 70 points, 35% scored 80 points, 30% scored 90
points, and the rest scored 100 points. What is the difference between the mean and median
score of the students’ scores on this quiz?
(A) 1 (B) 2 (C) 3 (D) 4 (E) 5

6 Suppose that a cows give b gallons of milk in c days. At this rate, how many gallons of milk
will d cows give in e days?
bde ac abde bcde abc
(A) ac (B) bde (C) c (D) a (E) de

© 2019 AoPS Incorporated 1


AoPS Community 2014 AMC 10

7 Nonzero real numbers x, y, a, and b satisfy x < a and y < b. How many of the following
inequalities must be true?
(I) x + y < a + b
(II) x − y < a − b
(III) xy < ab
(IV) xy < ab
(A) 0 (B) 1 (C) 2 (D) 3 (E) 4

8 Which of the following numbers is a perfect square?


14!15! 15!16! 16!17! 17!18! 18!19!
(A) (B) (C) (D) (E)
2 2 2 2 2

9 The two legs of a right triangle, which are altitudes, have lengths 2 3 and 6. How long is the
third altitude of the triangle?
(A) 1 (B) 2 (C) 3 (D) 4 (E) 5

10 Five positive consecutive integers starting with a have average b. What is the average of 5
consecutive integers that start with b?
(A) a + 3 (B) a + 4 (C) a + 5 (D) a + 6 (E) a + 7

11 A customer who intends to purchase an appliance has three coupons, only one of which may
be used:
Coupon 1: 10% off the listed price if the listed price is at least $50
Coupon 2: $20 off the listed price if the listed price is at least $100
Coupon 3: 18% off the amount by which the listed price exceeds $100
For which of the following listed prices will coupon 1 offer a greater price reduction than either
coupon 2 or coupon 3?
(A) $179.95 (B) $199.95 (C) $219.95 (D) $239.95 (E) $259.95

12 A regular hexagon has side length 6. Congruent arcs with radius 3 are drawn with the center
at each of the vertices, creating circular sectors as shown. The region inside the hexagon but
outside the sectors is shaded as shown What is the area of the shaded region?

© 2019 AoPS Incorporated 2


AoPS Community 2014 AMC 10

√ √ √ √ √
(A) 27 3 − 9π (B) 27 3 − 6π (C) 54 3 − 18π (D) 54 3 − 12π (E) 108 3 − 9π

13 Equilateral 4ABC has side length 1, and squares ABDE, BCHI, CAF G lie outside the trian-
gle. What is the area of hexagon DEF GHI?

F
E

G
D

B C

I H

√ √
12 + 3 3 9 √ 6+3 3
(A) (B) (C) 3 + 3 (D) (E) 6
4 2 2

14 The y-intercepts, P and Q, of two perpendicular lines intersecting at the point A(6, 8) have a
sum of zero. What is the area of 4AP Q?
(A) 45 (B) 48 (C) 54 (D) 60 (E) 72

15 David drives from his home to the airport to catch a flight. He drives 35 miles in the first hour,
but realizes that he will be 1 hour late if he continues at this speed. He increases his speed by
15 miles per hour for the rest of the way to the airport and arrives 30 minutes early. How many
miles is the airport from his home?
(A) 140 (B) 175 (C) 210 (D) 245 (E) 280

© 2019 AoPS Incorporated 3


AoPS Community 2014 AMC 10

16 In rectangle ABCD, AB = 1, BC = 2, and points E, F , and G are midpoints of BC, CD, and
AD, respectively. Point H is the midpoint of GE. What is the area of the shaded region?

A B

H E
G

D 1 F 1 C
2 2

√ √ √
1 3 2 3 1
(A) (B) (C) (D) (E)
12 18 12 12 6

17 Three fair six-sided dice are rolled. What is the probability that the values shown on two of the
dice sum to the value shown on the remaining die?
1 13 7 5 2
(A) (B) (C) (D) (E)
6 72 36 24 9

18 A square in the coordinate plane has vertices whose y-coordinates are 0, 1, 4, and 5. What is
the area of the square?
(A) 16 (B) 17 (C) 25 (D) 26 (E) 27

19 Four cubes with edge lengths 1, 2, 3, and 4 are stacked as shown. What is the length of the
portion of XY contained in the cube with edge length 3?
√ √
3 33 √ 2 33 √
(A) (B) 2 3 (C) (D) 4 (E) 3 2
5 3

© 2019 AoPS Incorporated 4


AoPS Community 2014 AMC 10

X
1

4 Y

20 The product (8)(888 . . . 8), where the second factor has k digits, is an integer whose digits have
a sum of 1000. What is k?
(A) 901 (B) 911 (C) 919 (D) 991 (E) 999

21 Positive integers a and b are such that the graphs of y = ax + 5 and y = 3x + b intersect
the x-axis at the same point. What is the sum of all possible x-coordinates of these points of
intersection?
(A) −20 (B) −18 (C) −15 (D) −12 (E) −8

22 In rectangle ABCD, AB = 20 and BC = 10. Let E be a point on CD such that ∠CBE = 15◦ .
What is AE?

20 3 √ √
(A) (B) 10 3 (C) 18 (D) 11 3 (E) 20
3

23 A rectangular piece of paper whose length is 3 times the width has area A. The paper is
divided into equal sections along the opposite lengths, and then a dotted line is drawn from
the first divider to the second divider on the opposite side as shown. The paper is then folded
flat along this dotted line to create a new shape with area B. What is the ratio B : A?

© 2019 AoPS Incorporated 5


AoPS Community 2014 AMC 10

(A) 1 : 2 (B) 3 : 5 (C) 2 : 3 (D) 3 : 4 (E) 4 : 5

24 A sequence of natural numbers is constructed by listing the first 4, then skipping one, listing
the next 5, skipping 2, listing 6, skipping 3, and, on the nth iteration, listing n + 3 and skipping n.
The sequence begins 1, 2, 3, 4, 6, 7, 8, 9, 10, 13. What is the 500, 000th number in the sequence?
(A) 996, 506 (B) 996507 (C) 996508 (D) 996509 (E) 996510

25 The number 5867 is between 22013 and 22014 . How many pairs of integers (m, n) are there such
that 1 ≤ m ≤ 2012 and
5n < 2m < 2m+2 < 5n+1 ?
(A) 278 (B) 279 (C) 280 (D) 281 (E) 282

– B

1 1. Leah has 13 coins, all of which are pennies and nickels. If she had one more nickel than she
has now, then she would have the same number of pennies and nickels. In cents, how much
are Leah’s coins worth?
(A) 33 (B) 35 (C) 37 (D) 39 (E) 41

23 +23
2 What is 2−3 +2−3
?
(A) 16 (B) 24 (C) 32 (D) 48 (E) 64

3 Randy drove the first third of his trip on a gravel road, the next 20 miles on pavement, and the
remaining one-fifth on a dirt road. In miles, how long was Randy’s trip?
400 75 300
(A) 30 (B) 11 (C) 2 (D) 40 (E) 7

4 Susie pays for 4 muffins and 3 bananas. Calvin spends twice as much paying for 2 muffins and
16 bananas. A muffin is how many times as expensive as a banana? (A) 23 (B) 53 (C) 74 (D) 2 (

© 2019 AoPS Incorporated 6


AoPS Community 2014 AMC 10

5 Doug constructs a square window using 8 equal-size panes of glass, as shown. The ratio of
the height to width for each pane is 5 : 2, and the borders around and between the panes are
2 inches wide. In inches, what is the side length o the square window?

(A) 26 (B) 28 (C) 30 (D) 32 (E) 34

6 Orvin went to the store with just enough money to buy 30 balloons. When he arrived, he dis-
covered that the store had a special sale on balloons: buy 1 balloon at the regular price and
get a second at 31 off the regular price. What is the greatest number of balloons Orvin could
buy?
(A) 33 (B) 34 (C) 36 (D) 38 (E) 39

7 Suppose A > B > 0 and A is x% greater than B. What is x?


(A) 100 A−B (B) 100 A+B (C) 100 A+B A−B A
    
B B A (D) 100 A (E) 100 B

8 A truck travels 6b feet every t seconds. There are 3 feet in a yard. How many yards does the
truck travel in 3 minutes?
b 30t 30b 10t 10b
(A) 1080t (B) b (C) t (D) b (E) t

9 For real numbers w and z,


1 1
w + z
1 1 = 2014.
w − z

What is w+z
w−z ?
−1 1
(A) − 2014 (B) 2014 (C) 2014 (D) 1 (E) 2014

© 2019 AoPS Incorporated 7


AoPS Community 2014 AMC 10

10 In the addition shown below A, B, C, and D are distinct digits. How many different values are
possible for D?

ABBCB
+ BCADA
DBDDD
(A) 2 (B) 4 (C) 7 (D) 8 (E) 9

11 For the consumer, a single discount of n% is more advantageous than any of the following
discounts:
(1) two successive 15% discounts (2) three successive 10% discounts (3) a 25% discount
followed by a 5% discount
What is the smallest possible positive integer value of n?
(A) 27 (B) 28 (C) 29 (D) 31 (E) 33

12 The largest divisor of 2, 014, 000, 000 is itself. What is its fifth largest divisor?
(A) 125, 875, 000 (B) 201, 400, 000 (C) 251, 750, 000 (D) 402, 800, 000 (E) 503, 500, 000

13 Six regular hexagons surround a regular hexagon of side length 1 as shown. What is the area
of 4ABC?

√ √ √ √ √
(A) 2 3 (B) 3 3 (C) 1 + 3 2 (D) 2 + 2 3 (E) 3 + 2 3

14 Danica drove her new car on a trip for a whole number of hours, averaging 55 miles per hour.
At the beginning of the trip, abc miles were displayed on the odometer, where abc is a 3-digit

© 2019 AoPS Incorporated 8


AoPS Community 2014 AMC 10

number with a ≥ 1 and a + b + c ≤ 7. At the end of the trip, where the odometer showed cba
miles. What is a2 + b2 + c2 ?
(A) 26 (B) 27 (C) 36 (D) 37 (E) 41

15 In rectangle ABCD, DC = 2CB and points E and F lie on AB so that ED and F D trisect
∠ADC as shown. What is the ratio of the area of 4DEF to the area of rectangle ABCD?

A E F B

D C

√ √ √ √
3 6 3 3 1 2
(A) 6 (B) 8 (C) 16 (D) 3 (E) 4

16 Four fair six-sided dice are rolled. What is the probability that at least three of the four dice
show the same value?
1 7 1 5 1
(A) 36 (B) 72 (C) 9 (D) 36 (E) 6

17 What is the greatest power of 2 that is a factor of 101002 − 4501 ?


(A) 21002 (B) 21003 (C) 21004 (D) 21005 (E) 21006

18 A list of 11 positive integers has a mean of 10, a median of 9, and a unique mode of 8. What is
the largest possible value of an integer in the list?
(A) 24 (B) 30 (C) 31 (D) 33 (E) 35

19 Two concentric circles have radii 1 and 2. Two points on the outer circle are chosen indepen-
dently and uniformly at random. What is the probability that the chord joining the two points
intersects the inner circle?

1 1 2− 2 1 1
(A) 6 (B) 4 (C) 2 (D) 3 (E) 2

20 For how many integers is the number x4 − 51x2 + 50 negative?


(A) 8 (B) 10 (C) 12 (D) 14 (E) 16

© 2019 AoPS Incorporated 9


AoPS Community 2014 AMC 10

21 Trapezoid ABCD has parallel sides AB or length 33 and CD of length 21. The other two sides
are of lengths 10 and 14. The angles at A and B are acute. What is the length of the shorter
diagonal of ABCD?
√ √ √
(A) 10 6 (B) 25 (C) 8 10 (D) 18 2 (E) 26

22 Eight semicircles line the inside of a square with side length 2 as shown. What is the radius of
the circle tangent to all of these semicircles?

√ √ √ √ √
1+ 2 5−1 3+1 2 3 5
(A) (B) (C) (D) (E)
4 2 4 5 3

23 A sphere is inscribed in a truncated right circular cone as shown. The volume of the truncated
cone is twice that of the sphere. What is the ratio of the radius of the bottom base of the
truncated cone to the radius of the top base of the truncated cone?

√ √
3 1+ 5 √ 3+ 5
(A) (B) (C) 3 (D) 2 (E)
2 2 2

© 2019 AoPS Incorporated 10


AoPS Community 2014 AMC 10

24 The numbers 1, 2, 3, 4, 5 are to be arranged in a circle. An arrangement is bad if it is not true that
for every n from 1 to 15 one can find a subset of the numbers that appear consecutively on the
circle that sum to n. Arrangements that differ only by a rotation or a reflection are considered
the same. How many different bad arrangements are there?
(A) 1 (B) 2 (C) 3 (D) 4 (E) 5

25 In a small pond there are eleven lily pads in a row labeled 0 through 10. A frog is sitting on pad
1. When the frog is on pad N , 0 < N < 10, it will jump to pad N − 1 with probability 10 N
and to
pad N + 1 with probability 1 − 10 . Each jump is independent of the previous jumps. If the frog
N

reaches pad 0 it will be eaten by a patiently waiting snake. If the frog reaches pad 10 it will exit
the pond, never to return. What is the probability that the frog will escape being eaten by the
snake?
32 161 63 7 1
(A) 79 (B) 384 (C) 146 (D) 16 (E) 2


These problems are copyright © Mathematical Association of America (http://maa.org).

© 2019 AoPS Incorporated 11


Art of Problem Solving is an ACS WASC Accredited School.
AoPS Community 2015 AMC 10

AMC 10 2015
www.artofproblemsolving.com/community/c4813
by djmathman, mathman523, stan23456, ABCDE, happiface, mota60ceng, Googolplex, mdlu, chezbgone,
va2010, droid347, nosaj, zacchro, rrusczyk

– A

– February 3rd

1 What is the value of (20 − 1 + 52 + 0)−1 × 5?


1 5
(A) − 125 (B) − 120 (C) (D) (E) 25
5 24

2 A box contains a collection of triangular and square tiles. There are 25 tiles in the box, con-
taining 84 edges total. How many square tiles are there in the box?
(A) 3 (B) 5 (C) 7 (D) 9 (E) 11

3 Ann made a 3-step staircase using 18 toothpicks as shown in the figure. How many toothpicks
does she need to add to complete a 5-step staircase?
(A) 9 (B) 18 (C) 20 (D) 22 (E) 24

4 Pablo, Sofia, and Mia got some candy eggs at a party. Pablo had three times as many eggs as
Sofia, and Sofia had twice as many eggs as Mia. Pablo decides to give some of his eggs to
Sofia and Mia so that all three will have the same number of eggs. What fraction of his eggs
should Pablo give to Sofia?

© 2019 AoPS Incorporated 1


AoPS Community 2015 AMC 10
1 1 1 1 1
(A) (B) (C) (D) (E)
12 6 4 3 2

5 Mr. Patrick teaches math to 15 students. He was grading tests and found that when he graded
everyone’s test except Payton’s, the average grade for the class was 80. After he graded Pay-
ton’s test, the class average became 81. What was Payton’s score on the test?
(A) 81 (B) 85 (C) 91 (D) 94 (E) 95

6 The sum of two positive numbers is 5 times their difference. What is the ratio of the larger
number to the smaller?
5 3 9 5
(A) (B) (C) (D) 2 (E)
4 2 5 2

7 How many terms are there in the arithmetic sequence 13, 16, 19, . . . , 70, 73?
(A) 20 (B) 21 (C) 24 (D) 60 (E) 61

8 Two years ago Pete was three times as old as his cousin Claire. Two years before that, Pete
was four times as old as Claire. In how many years will the ratio of their ages be 2 : 1?
(A) 2 (B) 4 (C) 5 (D) 6 (E) 8

9 Two right circular cylinders have the same volume. The radius of the second cylinder is 10%
more than the radius of the first. What is the relationship between the heights of the two cylin-
ders?
(A) The second height is 10% less than the first.
(B) The first height is 10% more than the second.
(C) The second height is 21% less than the first.
(D) The first height is 21% more than the second.
(E) The second height is 80% of the first.

10 How many rearrangements of abcd are there in which no two adjacent letters are also adjacent
letters in the alphabet? For example, no such rearrangements could include either ab or ba.
(A) 0 (B) 1 (C) 2 (D) 3 (E) 4

11 The ratio of the length to the width of a rectangle is 4 : 3. If the rectangle has diagonal of length
d, then the area may be expressed as kd2 for some constant k. What is k?
2 3 12 16 3
(A) (B) (C) (D) (E)
7 7 25 25 4

© 2019 AoPS Incorporated 2


AoPS Community 2015 AMC 10
√ √
12 Points ( π, a) and ( π, b) are distinct points on the graph of y 2 +x4 = 2x2 y +1. What is |a−b|?
π √ √
(A) 1 (B) (C) 2 (D) 1 + π (E) 1 + π
2

13 Claudia has 12 coins, each of which is a 5-cent coin or a 10-cent coin. There are exactly 17
different values that can be obtained as combinations of one or more of her coins. How many
10-cent coins does Claudia have?
(A) 3 (B) 4 (C) 5 (D) 6 (E) 7

14 The diagram below shows the circular face of a clock with radius 20 cm and a circular disk with
radius 10 cm externally tangent to the clock face at 12 o’clock. The disk has an arrow painted
on it, initially pointing in the upward vertical direction. Let the disk roll clockwise around the
clock face. At what point on the clock face will the disk be tangent when the arrow is next
pointing in the upward vertical direction?

11 12 1
10 2
9 3
8 4
7 5
6

(A) 2 o’clock (B) 3 o’clock (C) 4 o’clock (D) 6 o’clock (E) 8 o’clock

15 Consider the set of all fractions xy , where x and y are relatively prime positive integers. How
many of these fractions have the property that if both numerator and denominator are in-
creased by 1, the value of the fraction is increased by 10%?
(A) 0 (B) 1 (C) 2 (D) 3 (E) infinitely many

16 If y + 4 = (x − 2)2 , x + 4 = (y − 2)2 , and x 6= y, what is the value of x2 + y 2 ?


(A) 10 (B) 15 (C) 20 (D) 25 (E) 30

© 2019 AoPS Incorporated 3


AoPS Community 2015 AMC 10

17 A line that passes through the origin intersects both the line x = 1 and the line y = 1 + 3
3 x.
The three lines create an equilateral triangle. What is the perimeter of the triangle?
√ √ √ √
(A) 2 6 (B) 2 + 2 3 (C) 6 (D) 3 + 2 3 (E) 6 + 33

18 Hexadecimal (base-16) numbers are written using numeric digits 0 through 9 as well as the
letters A through F to represent 10 through 15. Among the first 1000 positive integers, there
are n whose hexadecimal representation contains only numeric digits. What is the sum of the
digits of n?
(A) 17 (B) 18 (C) 19 (D) 20 (E) 21

19 The isosceles right triangle ABC has right angle at C and area 12.5. The rays trisecting ∠ACB
intersect AB at D and E. What is the area of 4CDE?
√ √ √ √
5 2 50 3−75 15 3 50−25 3 25
(A) 3 (B) 4 (C) 8 (D) 2 (E) 6

20 A rectangle has area A cm2 and perimeter P cm, where A and P are positive integers. Which
of the following numbers cannot equal A + P ?
(A) 100 (B) 102 (C) 104 (D) 106 (E) 108

21 Tetrahedron ABCD has AB = 5, AC = 3, BC = 4, BD = 4, AD = 3, and CD = 12
5 2. What is
the volume of the tetrahedron?
√ √ 24 √ 24 √
(A) 3 2 (B) 2 5 (C) (D) 3 3 (E) 2
5 5

22 Eight people are sitting around a circular table, each holding a fair coin. All eight people flip
their coins and those who flip heads stand while those who flip tails remain seated. What is
the probability that no two adjacent people will stand?
47 3 49 25 51
(A) (B) (C) (D) (E)
256 16 256 128 256

23 The zeroes of the function f (x) = x2 − ax + 2a are integers. What is the sum of all possible
values of a?
(A) 7 (B) 8 (C) 16 (D) 17 (E) 18

24 For some positive integers p, there is a quadrilateral ABCD with positive integer side lengths,
perimeter p, right angles at B and C, AB = 2, and CD = AD. How many different values of
p < 2015 are possible?
(A) 30 (B) 31 (C) 61 (D) 62 (E) 63

© 2019 AoPS Incorporated 4


AoPS Community 2015 AMC 10

25 Let S be a square of side length 1. Two points are chosen independently at random on the
sides of S. The probability that the straight-line distance between the points is at least 12 is
c , where a, b, and c are positive integers and gcd(a, b, c) = 1. What is a + b + c?
a−bπ

(A) 59 (B) 60 (C) 61 (D) 62 (E) 63

– B

– February 25th

1 What is the value of 2 − (−2)−2 ?


1 7 9
(A) − 2 (B) (C) (D) (E) 6
16 4 4

2 Marie does three equally time-consuming tasks in a row without taking breaks. She begins the
first task at 1:00 PM and finishes the second task at 2:40 PM. When does she finish the third
task?
(A) 3:10 PM (B) 3:30 PM (C) 4:00 PM (D) 4:10 PM (E) 4:30 PM

3 Isaac has written down one integer two times and another integer three times. The sum of the
five numbers is 100, and one of the numbers is 28. What is the other number?
(A) 8 (B) 11 (C) 14 (D) 15 (E) 18

4 Four siblings ordered an extra large pizza. Alex ate 15 , Beth 13 , and Cyril 14 of the pizza. Dan got
the leftovers. What is the sequence of the siblings in decreasing order of the part of pizza they
consumed?
(A) Alex, Beth, Cyril, Dan (B) Beth, Cyril, Alex, Dan (C) Beth, Cyril, Dan, Alex (D) Beth, Dan, Cyril, Alex
(E) Dan, Beth, Cyril, Alex

5 David, Hikmet, Jack, Marta, Rand, and Todd were in a 12-person race with 6 other people. Rand
finished 6 places ahead of Hikmet. Marta finished 1 place behind Jack. David finished 2 places
behind Hikmet. Jack finished 2 places behind Todd. Todd finished 1 place behind Rand. Marta
finished in 6th place. Who finished in 8th place?
(A) David (B) Hikmet (C) Jack (D) Rand (E) Todd

6 Marley practices exactly one sport each day of the week. She runs three days a week but never
on two consecutive days. On Monday she plays basketball and two days later golf. She swims
and plays tennis, but she never plays tennis the day after running or swimming. Which day of
the week does Marley swim?
(A) Sunday (B) Tuesday (C) Thursday (D) Friday (E) Saturday

7 Consider the operation ”minus the reciprocal of,” defined by a  b = a − 1b . What is ((1  2)  3) −
(1  (2  3))?

© 2019 AoPS Incorporated 5


AoPS Community 2015 AMC 10
7 1 1 7
(A) − (B) − (C) 0 (D) (E)
30 6 6 30

8 The letter F shown below is rotated 90◦ clockwise around the origin, then reflected in the y-axis,
and then rotated a half turn around the origin. What is the final image?

y y y

(A) x (B) x (C) x

y y

(D) x (E) x

9 The shaded region below is called a shark’s fin falcata, a figure studied by Leonardo da Vinci. It
is bounded by the portion of the circle of radius 3 and center (0, 0) that lies in the first quadrant,
the portion of the circle with radius 23 and center (0, 32 ) that lies in the first quadrant, and the
line segment from (0, 0) to (3, 0). What is the area of the shark’s fin falcata?

y
3

3
2

3 x

© 2019 AoPS Incorporated 6


AoPS Community 2015 AMC 10

4π 9π 4π 7π 3π
(A) (B) (C) (D) (E)
5 8 3 5 2

10 What are the sign and units digit of the product of all the odd negative integers strictly greater
than −2015?
(A) It is a negative number ending with a 1. (B) It is a positive number ending with a 1. (C) It is a negative
(D) It is a positive number ending with a 5. (E) It is a negative number ending with a 0.

11 Among the positive integers less than 100, each of whose digits is a prime number, one is
selected at random. What is the probablility that the selected number is prime?
8 2 9 1 9
(A) (B) (C) (D) (E)
99 5 20 2 16

12 For how many integers x is the point (x, −x) inside or on the circle of radius 10 centered at
(5, 5)?
(A) 11 (B) 12 (C) 13 (D) 14 (E) 15

13 The line 12x+5y = 60 forms a triangle with the coordinate axes. What is the sum of the lengths
of the altitudes of this triangle?
360 107 43 281
(A) 20 (B) (C) (D) (E)
17 5 2 13

14 Let a, b, and c be three distinct one-digit numbers. What is the maximum value of the sum of
the roots of the equation (x − a)(x − b) + (x − b)(x − c) = 0?
(A) 15 (B) 15.5 (C) 16 (D) 16.5 (E) 17

15 The town of Hamlet has 3 people for each horse, 4 sheep for each cow, and 3 ducks for each
person. Which of the following could not possibly be the total number of people, horses, sheep,
cows, and ducks in Hamlet?
(A) 41 (B) 47 (C) 59 (D) 61 (E) 66

16 Al, Bill, and Cal will each randomly be assigned a whole number from 1 to 10, inclusive, with no
two of them getting the same number. What is the probability that Al’s number will be a whole
number multiple of Bill’s and Bill’s number will be a whole number multiple of Cal’s?
9 1 1 1 2
(A) (B) (C) (D) (E)
1000 90 80 72 121

17 The centers of the faces of the right rectangular prism shown below are joined to create an
octahedron, What is the volume of the octahedron?

© 2019 AoPS Incorporated 7


AoPS Community 2015 AMC 10

75 √
(A) (B) 10 (C) 12 (D) 10 2 (E) 15
12

18 Johann has 64 fair coins. He flips all the coins. Any coin that lands on tails is tossed again.
Coins that land on tails on the second toss are tossed a third time. What is the expected
number of coins that are now heads?
(A) 32 (B) 40 (C) 48 (D) 56 (E) 64

19 In 4ABC, ∠C = 90◦ and AB = 12. Squares ABXY and ACW Z are constructed outside of
the triangle. The points X, Y, Z, and W lie on a circle. What is the perimeter of the triangle?
√ √ √
(A) 12 + 9 3 (B) 18 + 6 3 (C) 12 + 12 2 (D) 30 (E) 32

20 Erin the ant starts at a given corner of a cube and crawls along exactly 7 edges in such a way
that she visits every corner exactly once and then finds that she is unable to return along an
edge to her starting point. How many paths are there meeting these conditions?
(A) 6 (B) 9 (C) 12 (D) 18 (E) 24

21 Cozy the Cat and Dash the Dog are going up a staircase with a certain number of steps. How-
ever, instead of walking up the steps one at a time, both Cozy and Dash jump. Cozy goes two
steps up with each jump (though if necessary, he will just jump the last step). Dash goes five
steps up with each jump (though if necessary, he will just jump the last steps if there are fewer
than 5 steps left). Suppose the Dash takes 19 fewer jumps than Cozy to reach the top of the
staircase. Let s denote the sum of all possible numbers of steps this staircase can have. What
is the sum of the digits of s?
(A) 9 (B) 11 (C) 12 (D) 13 (E) 15

22 In the figure shown below, ABCDE is a regular pentagon and AG = 1. What is F G+JH +CD?

© 2019 AoPS Incorporated 8


AoPS Community 2015 AMC 10
A

E F G B

J H
I

D C

√ √
√ 5+2 5 √ 11 + 11 5
(A) 3 (B) 12 − 4 5 (C) (D) 1 + 5 (E)
3 10

23 Let n be a positive integer greater than 4 such that the decimal representation of n! ends in
k zeros and the decimal representation of (2n)! ends in 3k zeros. Let s denote the sum of the
four least possible values of n. What is the sum of the digits of s?
(A) 7 (B) 8 (C) 9 (D) 10 (E) 11

24 Aaron the ant walks on the coordinate plane according to the following rules. He starts at the
origin p0 = (0, 0) facing to the east and walks one unit, arriving at p1 = (1, 0). For n = 1, 2, 3, . . . ,
right after arriving at the point pn , if Aaron can turn 90◦ left and walk one unit to an unvisited
point pn+1 , he does that. Otherwise, he walks one unit straight ahead to reach pn+1 . Thus the
sequence of points continues p2 = (1, 1), p3 = (0, 1), p4 = (−1, 1), p5 = (−1, 0), and so on in a
counterclockwise spiral pattern. What is p2015 ?
(A) (−22, −13) (B) (−13, −22) (C) (−13, 22) (D) (13, −22) (E) (22, −13)

25 A rectangular box measures a × b × c, where a, b, and c are integers and 1 ≤ a ≤ b ≤ c. The


volume and surface area of the box are numerically equal. How many ordered triples (a, b, c)
are possible?
(A) 4 (B) 10 (C) 12 (D) 21 (E) 26


These problems are copyright © Mathematical Association of America (http://maa.org).

© 2019 AoPS Incorporated 9


Art of Problem Solving is an ACS WASC Accredited School.
AoPS Community 2016 AMC 10

AMC 10 2016
www.artofproblemsolving.com/community/c222490
by adihaya, Royalreter1, atmchallenge, mathmaster2012, DanielL2000, Th3Numb3rThr33, shiningsunny-
day, DeathLlama9, quinna nyc, thatindiankid55, ythomashu, rrusczyk

– A

– February 2nd

11! − 10!
1 What is the value of ?
9!
(A) 99 (B) 100 (C) 110 (D) 121 (E) 132

2 For what value of x does 10x · 1002x = 10005 ?


(A) 1 (B) 2 (C) 3 (D) 4 (E) 5

3 For every dollar Ben spent on bagels, David spent 25 cents less. Ben paid $12.50 more than
David. How much did they spend in the bagel store together?
(A) $37.50 (B) $50.00 (C) $87.50 (D) $90.00 (E) $92.50

4 The remainder can be defined for all real numbers x and y with y 6= 0 by
 
x
rem(x, y) = x − y
y
j k
where xy denotes the greatest integer less than or equal to xy . What is the value of rem( 83 , − 25 )?
3 1 3 31
(A) − 8 (B) − 40 (C) 0 (D) 8 (E) 40

5 A rectangular box has integer side lengths in the ratio 1 : 3 : 4. Which of the following could
be the volume of the box?
(A) 48 (B) 56 (C) 64 (D) 96 (E) 144

6 Ximena lists the whole numbers 1 through 30 once. Emilio copies Ximena’s numbers, replacing
each occurrence of the digit 2 by the digit 1. Ximena adds her numbers and Emilio adds his
numbers. How much larger is Ximena’s sum than Emilio’s?
(A) 13 (B) 26 (C) 102 (D) 103 (E) 110

© 2019 AoPS Incorporated 1


AoPS Community 2016 AMC 10

7 The mean, median, and mode of the 7 data values 60, 100, x, 40, 50, 200, 90 are all equal to x.
What is the value of x?
(A) 50 (B) 60 (C) 75 (D) 90 (E) 100

8 Trickster Rabbit agrees with Foolish Fox to double Fox’s money every time Fox crosses the
bridge by Rabbit’s house, as long as Fox pays 40 coins in toll to Rabbit after each crossing. The
payment is made after the doubling, Fox is excited about his good fortune until he discovers
that all his money is gone after crossing the bridge three times. How many coins did Fox have
at the beginning?
(A) 20 (B) 30 (C) 35 (D) 40 (E) 45

9 A triangular array of 2016 coins has 1 coin in the first row, 2 coins in the second row, 3 coins in
the third row, and so on up to N coins in the N th row. What is the sum of the digits of N ?
(A) 6 (B) 7 (C) 8 (D) 9 (E) 10

10 A rug is made with three different colors as shown. The areas of the three differently colored
regions form an arithmetic progression. The inner rectangle is one foot wide, and each of the
two shaded regions is 1 foot wide on all four sides. What is the length in feet of the inner
rectangle?

1 1 1

(A) 1 (B) 2 (C) 4 (D) 6 (E) 8

11 What is the area of the shaded region of the given 8 × 5 rectangle?

© 2019 AoPS Incorporated 2


AoPS Community 2016 AMC 10
1 7
1

1
7 1

3 1 1
(A) 4 (B) 5 (C) 5 (D) 6 (E) 8
5 4 2

12 Three distinct integers are selected at random between 1 and 2016, inclusive. Which of the
following is a correct statement about the probability p that the product of the three integers
is odd?
1 1 1 1 1 1
(A) p < (B) p = (C) < p < (D) p = (E) p >
8 8 8 3 3 3

13 Five friends sat in a movie theater in a row containing 5 seats, numbered 1 to 5 from left to
right. (The directions ”left” and ”right” are from the point of view of the people as they sit in the
seats.) During the movie Ada went to the lobby to get some popcorn. When she returned, she
found that Bea had moved two seats to the right, Ceci had moved one seat to the left, and Dee
and Edie had switched seats, leaving an end seat for Ada. In which seat had Ada been sitting
before she got up?
(A) 1 (B) 2 (C) 3 (D) 4 (E) 5

14 How many ways are there to write 2016 as the sum of twos and threes, ignoring order? (For
example, 1008 · 2 + 0 · 3 and 402 · 2 + 404 · 3 are two such ways.)
(A) 236 (B) 336 (C) 337 (D) 403 (E) 672

15 Seven cookies of radius 1 inch are cut from a circle of cookie dough, as shown. Neighboring
cookies are tangent, and all except the center cookie are tangent to the edge of the dough. The
leftover scrap is reshaped to form another cookie of the same thickness. What is the radius in
inches of the scrap cookie?

© 2019 AoPS Incorporated 3


AoPS Community 2016 AMC 10

√ √ √
(A) 2 (B) 1.5 (C) π (D) 2π (E) π

16 A triangle with vertices A(0, 2), B(−3, 2), and C(−3, 0) is reflected about the x-axis, then the
image 4A0 B 0 C 0 is rotated counterclockwise about the origin by 90◦ to produce 4A00 B 00 C 00 .
Which of the following transformations will return 4A00 B 00 C 00 to 4ABC?
(A) counterclockwise rotation about the origin by 90◦ . (B) clockwise rotation about the origin
by 90◦ . (C) reflection about the x-axis (D) reflection about the line y = x (E) reflection about
the y-axis.

17 Let N be a positive multiple of 5. One red ball and N green balls are arranged in a line in random
order. Let P (N ) be the probability that at least 35 of the green balls are on the same side of the
red ball. Observe that P (5) = 1 and that P (N ) approaches 45 as N grows large. What is the
sum of the digits of the least value of N such that P (N ) < 321 400 ?

(A) 12 (B) 14 (C) 16 (D) 18 (E) 20

18 Each vertex of a cube is to be labeled with an integer 1 through 8, with each integer being used
once, in such a way that the sum of the four numbers on the vertices of a face is the same for
each face. Arrangements that can be obtained from each other through rotations of the cube
are considered to be the same. How many different arrangements are possible?
(A) 1 (B) 3 (C) 6 (D) 12 (E) 24

19 In rectangle ABCD, AB = 6 and BC = 3. Point E between B and C, and point F between


E and C are such that BE = EF = F C. Segments AE and AF intersect BD at P and Q,
respectively. The ratio BP : P Q : QD can be written as r : s : t, where the greatest common
factor of r, s and t is 1. What is r + s + t?
(A) 7 (B) 9 (C) 12 (D) 15 (E) 20

© 2019 AoPS Incorporated 4


AoPS Community 2016 AMC 10

20 For some particular value of N , when (a + b + c + d + 1)N is expanded and like terms are
combined, the resulting expression contains exactly 1001 terms that include all four variables
a, b, c, and d, each to some positive power. What is N ?
(A) 9 (B) 14 (C) 16 (D) 17 (E) 19

21 Circles with centers P, Q and R, having radii 1, 2 and 3, respectively, lie on the same side of
line l and are tangent to l at P 0 , Q0 and R0 , respectively, with Q0 between P 0 and R0 . The circle
with center Q is externally tangent to each of the other two circles. What is the area of triangle
P QR?
q √ √ q
(A) 0 (B) 23 (C) 1 (D) 6 − 2 (E) 32

22 For some positive integer n, the number 110n3 has 110 positive integer divisors, including 1
and the number 110n3 . How many positive integer divisors does the number 81n4 have?
(A) 110 (B) 191 (C) 261 (D) 325 (E) 425

23 A binary operation ♦ has the properties that a ♦ (b ♦ c) = (a ♦ b) · c and that a ♦ a = 1 for


all nonzero real numbers a, b, and c. (Here · represents multiplication). The solution to the
equation 2016 ♦ (6 ♦ x) = 100 can be written as pq , where p and q are relatively prime positive
integers. What is p + q?
(A) 109 (B) 201 (C) 301 (D) 3049 (E) 33, 601

24 A quadrilateral is inscribed in a circle of radius 200 2. Three of the sides of this quadrilateral
have length 200. What is the length of the fourth side?
√ √ √
(A) 200 (B) 200 2 (C) 200 3 (D) 300 2 (E) 500

25 How many ordered triples (x, y, z) of positive integers satisfy lcm(x, y) = 72, lcm(x, z) = 600
and lcm(y, z) = 900?
(A) 15 (B) 16 (C) 24 (D) 27 (E) 64

– B

– February 17th

−1
2a−1 + a 2
1 What is the value of a when a = 12 ?
5
(A) 1 (B) 2 (C) 2 (D) 10 (E) 20

2 If n♥m = n3 m2 , what is 4♥2 ?


2♥4

© 2019 AoPS Incorporated 5


AoPS Community 2016 AMC 10
1 1
(A) 4 (B) 2 (C) 1 (D) 2 (E) 4

3 Let x = −2016. What is the value of |x| − x − |x| − x?


(A) − 2016 (B) 0 (C) 2016 (D) 4032 (E) 6048

4 Zoey read 15 books, one at a time. The first book took her 1 day to read, the second book took
her 2 days to read, the third book took her 3 days to read, and so on, with each book taking her
1 more day to read than the previous book. Zoey finished the first book on a monday, and the
second on a Wednesday. On what day the week did she finish her 15th book?
(A) Sunday (B) Monday (C) Wednesday (D) Friday (E) Saturday

5 The mean age of Amanda’s 4 cousins is 8, and their median age is 5. What is the sum of the
ages of Amanda’s youngest and oldest cousins?
(A) 13 (B) 16 (C) 19 (D) 22 (E) 25

6 Laura added two three-digit positive integers. All six digits in these numbers are different.
Laura’s sum is a three-digit number S. What is the smallest possible value for the sum of the
digits of S?
(A) 1 (B) 4 (C) 5 (D) 15 (E) 21

7 The ratio of the measures of two acute angles is 5 : 4, and the complement of one of these
two angles is twice as large as the complement of the other. What is the sum of the degree
measures of the two angles?
(A) 75 (B) 90 (C) 135 (D) 150 (E) 270

8 What is the tens digit of 20152016 − 2017?


(A) 0 (B) 1 (C) 3 (D) 5 (E) 8

9 All three vertices of 4ABC lie on the parabola defined by y = x2 , with A at the origin and BC
parallel to the x-axis. The area of the triangle is 64. What is the length of BC?
(A) 4 (B) 6 (C) 8 (D) 10 (E) 16

10 A thin piece of wood of uniform density in the shape of an equilateral triangle with side length
3 inches weighs 12 ounces. A second piece of the same type of wood, with the same thickness,
also in the shape of an equilateral triangle, has side length of 5 inches. Which of the following
is closest to the weight, in ounces, of the second piece?
(A) 14.0 (B) 16.0 (C) 20.0 (D) 33.3 (E) 55.6

© 2019 AoPS Incorporated 6


AoPS Community 2016 AMC 10

11 Carl decided to fence in his rectangular garden. He bought 20 fence posts, placed one on each
of the four corners, and spaced out the rest evenly along the edges of the garden, leaving ex-
actly 4 yards between neighboring posts. The longer side of his garden, including the corners,
has twice as many posts as the shorter side, including the corners. What is the area, in square
yards, of Carls garden?
(A) 256 (B) 336 (C) 384 (D) 448 (E) 512

12 Two different numbers are selected at random from (1, 2, 3, 4, 5) and multiplied together. What
is the probability that the product is even?
(A) 0.2 (B) 0.4 (C) 0.5 (D) 0.7 (E) 0.8

13 At Megapolis Hospital one year, multiple-birth statistics were as follows: Sets of twins, triplets,
and quadruplets accounted for 1000 of the babies born. There were four times as many sets
of triplets as sets of quadruplets, and there was three times as many sets of twins as sets of
triplets. How many of these 1000 babies were in sets of quadruplets?
(A) 25 (B) 40 (C) 64 (D) 100 (E) 160

14 How many squares whose sides are parallel to the axes and whose vertices have coordinates
that are integers lie entirely within the region bounded by the line y = πx, the line y = −0.1
and the line x = 5.1?
(A) 30 (B) 41 (C) 45 (D) 50 (E) 57

15 All the numbers 1, 2, 3, 4, 5, 6, 7, 8, 9 are written in a 3 × 3 array of squares, one number in each
square, in such a way that if two numbers are consecutive then they occupy squares that share
an edge. The numbers in the four corners add up to 18. What is the number in the center?
(A) 5 (B) 6 (C) 7 (D) 8 (E) 9

16 The sum of an infinite geometric series is a positive number S, and the second term in the
series is 1. What is the smallest possible value of S?
√ √
(A) 1+2 5 (B) 2 (C) 5 (D) 3 (E) 4

17 All the numbers 2, 3, 4, 5, 6, 7 are assigned to the six faces of a cube, one number to each face.
For each of the eight vertices of the cube, a product of three numbers is computed, where the
three numbers are the numbers assigned to the three faces that include that vertex. What is
the greatest possible value of the sum of these eight products?
(A) 312 (B) 343 (C) 625 (D) 729 (E) 1680

18 In how many ways can 345 be written as the sum of an increasing sequence of two or more
consecutive positive integers?

© 2019 AoPS Incorporated 7


AoPS Community 2016 AMC 10

(A) 1 (B) 3 (C) 5 (D) 6 (E) 7

19 Rectangle ABCD has AB = 5 and BC = 4. Point E lies on AB so that EB = 1, point G lies on


BC so that CG = 1. and point F lies on CD so that DF = 2. Segments AG and AC intersect
EF at Q and P , respectively. What is the value of PEF
Q
?

A E B

Q
P
G

D F C

√ √
13 2 9 10 1
(A) 16 (B) 13 (C) 82 (D) 91 (E) 9

20 A dilation of the planethat is, a size transformation with a positive scale factorsends the circle
of radius 2 centered at A(2, 2) to the circle of radius 3 centered at A(5, 6). What distance does
the origin O(0, 0), move under this transformation?

(A) 0 (B) 3 (C) 13 (D) 4 (E) 5

21 What is the area of the region enclosed by the graph of the equation x2 + y 2 = |x| + |y|?
√ √ √ √
(A) π + 2 (B) π + 2 (C) π + 2 2 (D) 2π + 2 (E) 2π + 2 2

22 A set of teams held a round-robin tournament in which every team played every other team
exactly once. Every team won 10 games and lost 10 games; there were no ties. How many sets
of three teams {A, B, C} were there in which A beat B, B beat C, and C beat A?
(A) 385 (B) 665 (C) 945 (D) 1140 (E) 1330

23 In regular hexagon ABCDEF , points W , X, Y , and Z are chosen on sides BC, CD, EF , and
F A respectively, so lines AB, ZW , Y X, and ED are parallel and equally spaced. What is the
ratio of the area of hexagon W CXY F Z to the area of hexagon ABCDEF ?
1 10 11 4 13
(A) 3 (B) 27 (C) 27 (D) 9 (E) 27

24 How many four-digit integers abcd, with a 6= 0, have the property that the three two-digit inte-
gers ab < bc < cd form an increasing arithmetic sequence? One such number is 4692, where

© 2019 AoPS Incorporated 8


AoPS Community 2016 AMC 10

a = 4, b = 6, c = 9, and d = 2.
(A) 9 (B) 15 (C) 16 (D) 17 (E) 20

Let f (x) = 10
k=2 (bkxc − kbxc), where brc denotes the greatest integer less than or equal to r.
P
25
How many distinct values does f (x) assume for x ≥ 0?
(A) 32 (B) 36 (C) 45 (D) 46 (E) infinitely many


These problems are copyright © Mathematical Association of America (http://maa.org).

© 2019 AoPS Incorporated 9


Art of Problem Solving is an ACS WASC Accredited School.
AoPS Community 2017 AMC 10

AMC 10 2017
www.artofproblemsolving.com/community/c408752
by techguy2, MathArt4, thatindiankid55, WhaleVomit, CountofMC, happiface, DeathLlama9, Mudkipswims42,
ythomashu, always correct, PiDude314, LauraZed, pi Plus 45x23, Superwiz, Benq, AstrapiGnosis, Can-
tonMathGuy, rrusczyk

– A

– February 7th, 2017

1 What is the value of 2(2(2(2(2(2 + 1) + 1) + 1) + 1) + 1) + 1?


(A) 70 (B) 97 (C) 127 (D) 159 (E) 729

2 Pablo buys popsicles for his friends. The store sells single popsicles for $1 each, 3-popsicle
boxes for $2, and 5-popsicle boxes for $3. What is the greatest number of popsicles that Pablo
can buy with $8?
(A) 8 (B) 11 (C) 12 (D) 13 (E) 15

3 Tamara has three rows of two 6-feet by 2-feet flower beds in her garden. The beds are sep-
arated and also surrounded by 1-foot-wide walkways, as shown on the diagram. What is the
total area of the walkways, in square feet?

1 1 1
1

2
6
1

(A) 72 (B) 78 (C) 90 (D) 120 (E) 150

© 2019 AoPS Incorporated 1


AoPS Community 2017 AMC 10

4 Mia is helping her mom pick up 30 toys that are strewn on the floor. Mias mom manages to put
3 toys into the toy box every 30 seconds, but each time immediately after those 30 seconds
have elapsed, Mia takes 2 toys out of the box. How much time, in minutes, will it take Mia and
her mom to put all 30 toys into the box for the first time?
(A) 13.5 (B) 14 (C) 14.5 (D) 15 (E) 15.5

5 The sum of two nonzero real numbers is 4 times their product. What is the sum of the recipro-
cals of the two numbers?
(A) 1 (B) 2 (C) 4 (D) 8 (E) 12

6 Ms. Carroll promised that anyone who got all the multiple choice questions right on the up-
coming exam would receive an A on the exam. Which of these statements necessarily follows
logically?
(A) If Lewis did not receive an A, then he got all of the multiple choice questions wrong.
(B) If Lewis did not receive an A, then he got at least one of the multiple choice questions
wrong.
(C) If Lewis got at least one of the multiple choice questions wrong, then he did not receive an
A.
(D) If Lewis received an A, then he got all of the multiple choice questions right.
(E) If Lewis received an A, then he got at least one of the multiple choice questions right.

7 Jerry and Silvia wanted to go from the southwest corner of a square field to the northeast
corner. Jerry walked due east and then due north to reach the goal, but Silvia headed northeast
and reached the goal walking in a straight line. Which of the following is closest to how much
shorter Silvia’s trip was, compared to Jerry’s trip?
(A) 30% (B) 40% (C) 50% (D) 60% (E) 70%

8 At a gathering of 30 people, there are 20 people who all know each other and 10 people who
know no one. People who know each other hug, and people who do not know each other shake
hands. How many handshakes occur?
(A) 240 (B) 245 (C) 290 (D) 480 (E) 490

9 Minnie rides on a flat road at 20 kilometers per hour (kph), downhill at 30 kph, and uphill at 5
kph. Penny rides on a flat road at 30 kph, downhill at 40 kph, and uphill at 10 kph. Minnie goes
from town A to town B, a distance of 10 km all uphill, then from town B, a distance of 15 km
all downhill, and then back to town A, a distance of 20 km on the flat. Penny goes the other
way around using the same route. How many more minutes does it take Minnie to complete
the 45-km ride than it takes Penny?

© 2019 AoPS Incorporated 2


AoPS Community 2017 AMC 10

(A) 45 (B) 60 (C) 65 (D) 90 (E) 95

10 Joy has 30 thin rods, one each of every integer length from 1 cm through 30 cm. She places
the rods with lengths 3 cm, 7 cm, and 15 cm on a table. She then wants to choose a fourth rod
that she can put with these three to form a quadrilateral with positive area. How many of the
remaining rods can she choose as the fourth rod?
(A) 16 (B) 17 (C) 18 (D) 19 (E) 20

11 The region consisting of all points in three-dimensional space within 3 units of line segment
AB has volume 216π. What is the length AB?
(A) 6 (B) 12 (C) 18 (D) 20 (E) 24

12 Let S be the set of points (x, y) in the coordinate plane such that two of the three quantities 3,
x + 2, and y − 4 are equal and the third of the three quantities is no greater than this common
value. Which of the following is a correct description of S?
(A) a single point (B) two intersecting lines

(C) three lines whose pairwise intersections are three distinct points

(D) a triangle (E) three rays with a common endpoint

13 Define a sequence recursively by F0 = 0, F1 = 1, and Fn = the remainder when Fn−1 + Fn−2


is divided by 3, for all n ≥ 2. Thus the sequence starts 0, 1, 1, 2, 0, 2 . . .. What is F2017 + F2018 +
F2019 + F2020 + F2021 + F2022 + F2023 + F2024 ?
(A) 6 (B) 7 (C) 8 (D) 9 (E) 10

14 Every week Roger pays for a movie ticket and a soda out of his allowance. Last week, Roger’s
allowance was A dollars. The cost of his movie ticket was 20% of the difference between A
and the cost of his soda, while the cost of his soda was 5% of the difference between A and
the cost of his movie ticket. To the nearest whole percent, what fraction of A did Roger pay
for his movie ticket and soda?
(A) 9% (B) 19% (C) 22% (D) 23% (E) 25%

15 Chlo chooses a real number uniformly at random from the interval [0, 2017]. Independently,
Laurent chooses a real number uniformly at random from the interval [0, 4034]. What is the
probability that Laurent’s number is greater than Chlo’s number?
1 2 3 5 7
(A) 2 (B) 3 (C) 4 (D) 6 (E) 8

© 2019 AoPS Incorporated 3


AoPS Community 2017 AMC 10

16 There are 10 horses, named Horse 1, Horse 2, . . ., Horse 10. They get their names from how
many minutes it takes them to run one lap around a circular race track: Horse k runs one lap
in exactly k minutes. At time 0 all the horses are together at the starting point on the track.
The horses start running in the same direction, and they keep running around the circular track
at their constant speeds. The least time S > 0, in minutes, at which all 10 horses will again
simultaneously be at the starting point is S = 2520. Let T > 0 be the least time, in minutes,
such that at least 5 of the horses are again at the starting point. What is the sum of the digits
of T ?
(A) 2 (B) 3 (C) 4 (D) 5 (E) 6

17 Distinct points P , Q, R, S lie on the circle x2 + y 2 = 25 and have integer coordinates. The
distances P Q and RS are irrational numbers. What is the greatest possible value of the ratio
PQ
RS ?
√ √
(A) 3 (B) 5 (C) 3 5 (D) 7 (E) 5 2

18 Amelia has a coin that lands heads with probability 13 , and Blaine has a coin that lands on
heads with probability 52 . Amelia and Blaine alternately toss their coins until someone gets a
head; the first one to get a head wins. All coin tosses are independent. Amelia goes first. The
probability that Amelia wins is pq , where p and q are relatively prime positive integers. What is
q − p?
(A) 1 (B) 2 (C) 3 (D) 4 (E) 5

19 Alice refuses to sit next to either Bob or Carla. Derek refuses to sit next to Eric. How many
ways are there for the five of them to sit in a row of 5 chairs under these conditions?
(A) 12 (B) 16 (C) 28 (D) 32 (E) 40

20 Let S(n) equal the sum of the digits of positive integer n. For example, S(1507) = 13. For a
particular positive integer n, S(n) = 1274. Which of the following could be the value of S(n+1)?
(A) 1 (B) 3 (C) 12 (D) 1239 (E) 1265

21 A square with side length x is inscribed in a right triangle with sides of length 3, 4, and 5 so
that one vertex of the square coincides with the right-angle vertex of the triangle. A square
with side length y is inscribed so that one side of the square lies on the hypotenuse of the
triangle. What is xy ?
12 35 37 13
(A) 13 (B) 37 (C) 1 (D) 35 (E) 12

22 Sides AB and AC of equilateral triangle ABC are tangent to a circle at points B and C respec-
tively. What fraction of the area of 4ABC lies outside the circle?

© 2019 AoPS Incorporated 4


AoPS Community 2017 AMC 10
√ √
4 3π 1

3 π 1
√ √
2 3π 4 4 3π
(A) − 3 (B) 2 − 8 (C) 2 (D) 3− 9 (E) 3 −
27 27

23 How many triangles with positive area have all their vertices at points (i, j) in the coordinate
plane, where i and j are integers between 1 and 5, inclusive?
(A) 2128 (B) 2148 (C) 2160 (D) 2200 (E) 2300

24 For certain real numbers a, b, and c, the polynomial

g(x) = x3 + ax2 + x + 10

has three distinct roots, and each root of g(x) is also a root of the polynomial

f (x) = x4 + x3 + bx2 + 100x + c.

What is f (1)?
(A) − 9009 (B) − 8008 (C) − 7007 (D) − 6006 (E) − 5005

25 How many integers between 100 and 999, inclusive, have the property that some permutation
of its digits is a multiple of 11 between 100 and 999? For example, both 121 and 211 have this
property.
(A) 226 (B) 243 (C) 270 (D) 469 (E) 486

– B

– February 15th, 2017

1 Mary thought of a positive two-digit number. She multiplied it by 3 and added 11. Then she
switched the digits of the result, obtaining a number between 71 and 75, inclusive. What was
Mary’s number?
(A) 11 (B) 12 (C) 13 (D) 14 (E) 15

2 Sofia ran 5 laps around the 400-meter track at her school. For each lap, she ran the first 100
meters at an average speed of 4 meters per second and the remaining 300 meters at an aver-
age speed of 5 meters per second. How much time did Sofia take running the 5 laps?
(A) 5 minutes and 35 seconds (B) 6 minutes and 40 seconds (C) 7 minutes and 5 seconds (D) 7 minutes
(E) 8 minutes and 10 seconds

3 Real numbers x, y, and z satisfy the inequalities

0 < x < 1, −1 < y < 0, and 1 < z < 2.

Which of the following numbers is nessecarily positive?

© 2019 AoPS Incorporated 5


AoPS Community 2017 AMC 10

(A) y + x2 (B) y + xz (C) y + y 2 (D) y + 2y 2


(E) y + z

4 Suppose that x and y are nonzero real numbers such that


3x + y
= −2.
x − 3y
What is the value of
x + 3y
?
3x − y
(A) −3 (B) −1 (C) 1 (D) 2 (E) 3

5 Camilla had twice as many blueberry jelly beans as cherry jelly beans. After eating 10 pieces of
each kind, she now has three times as many blueberry jelly beans as cherry jelly beans. How
many blueberry jelly beans did she originally have?
(A) 10 (B) 20 (C) 30 (D) 40 (E) 50

6 What is the largest number of solid 2-in × 2-in × 1-in blocks that can fit in a 3-in × 2-in × 3-in
box?
(A) 3 (B) 4 (C) 5 (D) 6 (E) 7

7 Samia set off on her bicycle to visit her friend, traveling at an average speed of 17 kilometers
per hour. When she had gone half the distance to her friend’s house, a tire went flat, and she
walked the rest of the way at 5 kilometers per hour. In all it took her 44 minutes to reach her
friend’s house. In kilometers rounded to the nearest tenth, how far did Samia walk?
(A) 2.0 (B) 2.2 (C) 2.8 (D) 3.4 (E) 4.4

8 Points A(11, 9) and B(2, −3) are vertices of 4ABC with AB = AC. The altitude from A meets
the opposite side at D(−1, 3). What are the coordinates of point C?
(A) (−8, 9) (B) (−4, 8) (C) (−4, 9) (D) (−2, 3) (E) (−1, 0)

9 A radio program has a quiz consisting of 3 multiple-choice questions, each with 3 choices. A
contestant wins if he or she gets 2 or more of the questions right. The contestant answers
randomly to each question. What is the probability of winning?
1 1 2 7 1
(A) 27 (B) 9 (C) 9 (D) 27 (E) 2

10 The lines with equations ax − 2y = c and 2x + by = −c are perpendicular and intersect at


(1, −5). What is c?
(A) − 13 (B) − 8 (C) 2 (D) 8 (E) 13

© 2019 AoPS Incorporated 6


AoPS Community 2017 AMC 10

11 At Typico High School, 60% of the students like dancing, and the rest dislike it. Of those who
like dancing, 80% say that they like it, and the rest say that they dislike it. Of those who dislike
dancing, 90% say that they dislike it, and the rest say that they like it. What fraction of students
who say they dislike dancing actually like it?
(A) 10% (B) 12% (C) 20% (D) 25% (E) 33 13 %

12 Elmer’s new car gives 50% percent better fuel efficiency, measured in kilometers per liter, than
his old car. However, his new car uses diesel fuel, which is 20% more expensive per liter than
the gasoline his old car used. By what percent will Elmer save money if he uses his new car
instead of his old car for a long trip?
(A) 20% (B) 26 23 % (C) 27 97 % (D) 33 13 % (E) 66 23 %

13 There are 20 students participating in an after-school program offering classes in yoga, bridge,
and painting. Each student must take at least one of these three classes, but may take two or
all three. There are 10 students taking yoga, 13 taking bridge, and 9 taking painting. There are
9 students taking at least two classes. How many students are taking all three classes?
(A) 1 (B) 2 (C) 3 (D) 4 (E) 5

14 An integer N is selected at random in the range 1 ≤ N ≤ 2020. What is the probability that the
remainder when N 16 is divided by 5 is 1?
(A) 1
5 (B) 2
5 (C) 3
5 (D) 4
5 (E) 1

15 Rectangle ABCD has AB = 3 and BC = 4. Point E is the foot of the perpendicular from B to
diagonal AC. What is the area of 4ADE?
(A) 1 (B) 42
25 (C) 28
15 (D) 2 (E) 54
25

16 How many of the base-ten numerals for the positive integers less than or equal to 2017 contain
the digit 0?
(A) 469 (B) 471 (C) 475 (D) 478 (E) 481

17 Call a positive integer monotonous if it is a one-digit number or its digits, when read from left
to right, form either a strictly increasing or a strictly decreasing sequence. For example, 3,
23578, and 987620 are monotonous, but 88, 7434, and 23557 are not. How many monotonous
positive integers are there?
(A) 1024 (B) 1524 (C) 1533 (D) 1536 (E) 2048

18 In the figure below, 3 of the 6 disks are to be painted blue, 2 are to be painted red, and 1 is
to be painted green. Two paintings that can be obtained from one another by a rotation or

© 2019 AoPS Incorporated 7


AoPS Community 2017 AMC 10

a reflection of the entire figure are considered the same. How many different paintings are
possible?

(A) 6 (B) 8 (C) 9 (D) 12 (E) 15

19 Let ABC be an equilateral triangle. Extend side AB beyond B to a point B 0 so that BB 0 = 3AB.
Similarly, extend side BC beyond C to a point C 0 so that CC 0 = 3BC, and extend side CA
beyond A to a point A0 so that AA0 = 3CA. What is the ratio of the area of 4A0 B 0 C 0 to the area
of 4ABC?
(A) 9 : 1 (B) 16 : 1 (C) 25 : 1 (D) 36 : 1 (E) 37 : 1

20 The number 21! = 51, 090, 942, 171, 709, 440, 000 has over 60, 000 positive integer divisors. One
of them is chosen at random. What is the probability that it is odd?
1 1 1
(A) 21 (B) 19 (C) 18 (D) 12 (E) 11
21

21 In 4ABC, AB = 6, AC = 8, BC = 10, and D is the midpoint of BC. What is the sum of the
radii of the circles inscribed in 4ADB and 4ADC?
√ √
(A) 5 (B) 11
4 (C)2 2 (D) 17
6 (E)3

22 The diameter AB of a circle of radius 2 is extended to a point D outside the circle so that
BD = 3. Point E is chosen so that ED = 5 and the line ED is perpendicular to the line AD.
Segment AE intersects the circle at point C between A and E. What is the area of 4ABC?
120 140 145 140 120
(A) 37 (B) 39 (C) 39 (D) 37 (E) 31

23 Let N = 123456789101112 . . . 4344 be the 79-digit number obtained that is formed by writing
the integers from 1 to 44 in order, one after the other. What is the remainder when N is divided
by 45?
(A) 1 (B) 4 (C) 9 (D) 18 (E) 44

© 2019 AoPS Incorporated 8


AoPS Community 2017 AMC 10

24 The vertices of an equilateral triangle lie on the hyperbola xy = 1, and a vertex of this hyperbola
is the centroid of the triangle. What is the square of the area of the triangle?
(A) 48 (B) 60 (C) 108 (D) 120 (E) 169

25 Last year Isabella took 7 math tests and received 7 different scores, each an integer between
91 and 100, inclusive. After each test she noticed that the average of her test scores was an
integer. Her score on the seventh test was 95. What was her score on the sixth test?
(A) 92 (B) 94 (C) 96 (D) 98 (E) 100


These problems are copyright © Mathematical Association of America (http://maa.org).

– - Compiled by techguy2 -

© 2019 AoPS Incorporated 9


Art of Problem Solving is an ACS WASC Accredited School.
AoPS Community 2018 AMC 10

AMC 10 2018
www.artofproblemsolving.com/community/c588207
by AlcumusGuy, Vfire, proshi, thatmath, bluecarneal, djmathman, laegolas, AOPS12142015, avn, ythomashu,
Dhman2727, mathmaster2012, Mudkipswims42, claserken, reedmj, lifeisgood03, kevinmathz, Canton-
MathGuy, rrusczyk, brainiac1, AIME12345, BIGBUBBLE, CobbleHead, ccx09, FunkyPants1263, Math Magicians,
itised, Dr4gon39, geogirl08, GeronimoStilton

– A

– February 7th, 2018

1 What is the value of  −1


−1
−1
(2 + 1) +1 +1 + 1?

5 11 8 18 15
(A) 8 (B) 7 (C) 5 (D) 11 (E) 8

2 Liliane has 50% more soda than Jacqueline, and Alice has 25% more soda than Jacqueline.
What is the relationship between the amounts of soda that Liliane and Alica have?
(A) Liliane has 20% more soda than Alice. (B) Liliane has 25% more soda than Alice. (C) Liliane has 45%
(D) Liliane has 75% more soda than Alice. (E) Liliane has 100% more soda than Alice.

3 A unit of blood expires after 10! = 10 · 9 · 8 · · · 1 seconds. Yasin donates a unit of blood at noon
of January 1. On what day does his unit of blood expire?
(A) January 2 (B) January 12 (C) January 22 (D) February 11 (E) February 12

4 How many ways can a student schedule 3 mathematics courses – algebra, geometry, and
number theory – in a 6-period day if no two mathematics courses can be taken in consecutive
periods? (What courses the student takes during the other 3 periods is of no concern here.)
(A) 3 (B) 6 (C) 12 (D) 18 (E) 24

5 Alice, Bob, and Charlie were on a hike and were wondering how far away the nearest town was.
When Alice said, ”We are at least 6 miles away,” Bob replied, ”We are at most 5 miles away.”
Charlie then remarked, ”Actually the nearest town is at most 4 miles away.” It turned out that
none of the three statements were true. Let d be the distance in miles to the nearest town.
Which of the following intervals is the set of all possible values of d?
(A) (0, 4) (B) (4, 5) (C) (4, 6) (D) (5, 6) (E) (5, ∞)

© 2019 AoPS Incorporated 1


AoPS Community 2018 AMC 10

6 Sangho uploaded a video to a website where viewers can vote that they like or dislike a video.
Each video begins with a score of 0, and the score increases by 1 for each like vote and de-
creases by 1 for each dislike vote. At one point Sangho saw that his video had a score of 90,
and that 65% of the votes cast on his video were like votes. How many votes had been cast on
Sangho’s video at that point?
(A) 200 (B) 300 (C) 400 (D) 500 (E) 600

2 n
For how many (not necessarily positive) integer values of n is the value of 4000· an integer?

7 5

(A) 3 (B) 4 (C) 6 (D) 8 (E) 9

8 Joe has a collection of 23 coins, consisting of 5-cent coins, 10-cent coins, and 25-cent coins.
He has 3 more 10-cent coins than 5-cent coins, and the total value of his collection is 320
cents. How many more 25-cent coins does Joe have than 5-cent coins?
(A) 0 (B) 1 (C) 2 (D) 3 (E) 4

9 All of the triangles in the diagram below are similar to iscoceles triangle ABC, in which AB =
AC. Each of the 7 smallest triangles has area 1, and 4ABC has area 40. What is the area of
trapezoid DBCE?

D E

B C

(A) 16 (B) 18 (C) 20 (D) 22 (E) 24

10 Suppose that real number x satisfies


p p
49 − x2 − 25 − x2 = 3.
√ √
What is the value of 49 − x2 + 25 − x2 ?

© 2019 AoPS Incorporated 2


AoPS Community 2018 AMC 10
√ √
(A) 8 (B) 33 + 8 (C) 9 (D) 2 10 + 4 (E) 12

11 When 7 fair standard 6-sided dice are thrown, the probability that the sum of the numbers on
the top faces is 10 can be written as
n
,
67
where n is a positive integer. What is n?
(A) 42 (B) 49 (C) 56 (D) 63 (E) 84

12 How many ordered pairs of real numbers (x, y) satisfy the following system of equations?

x + 3y = 3
|x| − |y| = 1

(A) 1 (B) 2 (C) 3 (D) 4 (E) 8

13 A paper triangle with sides of lengths 3, 4, and 5 inches, as shown, is folded so that point A
falls on point B. What is the length in inches of the crease?

5
3

A 4 C

1
√ √ 7 15
(A) 1 + 2 2 (B) 3 (C) 4 (D) 8 (E) 2

14 What is the greatest integer less than or equal to

3100 + 2100
?
396 + 296
(A) 80 (B) 81 (C) 96 (D) 97 (E) 625

15 Two circles of radius 5 are externally tangent to each other and are internally tangent to a circle
of radius 13 at points A and B, as shown in the diagram. The distance AB can be written in
the form mn , where m and n are relatively prime positive integers. What is m + n?

© 2019 AoPS Incorporated 3


AoPS Community 2018 AMC 10

A B

(A) 21 (B) 29 (C) 58 (D) 69 (E) 93

16 Right triangle ABC has leg lengths AB = 20 and BC = 21. Including AB and BC, how many
line segments with integer length can be drawn from vertex B to a point on hypotenuse AC?
(A) 5 (B) 8 (C) 12 (D) 13 (E) 15

17 Let S be a set of 6 integers taken from {1, 2, . . . , 12} with the property that if a and b are el-
ements of S with a < b, then b is not a multiple of a. What is the least possible value of an
element in S?
(A) 2 (B) 3 (C) 4 (D) 5 (E) 7

18 How many nonnegative integers can be written in the form


a7 · 37 + a6 · 36 + a5 · 35 + a4 · 34 + a3 · 33 + a2 · 32 + a1 · 31 + a0 · 30 ,
where ai ∈ {−1, 0, 1} for 0 ≤ i ≤ 7?
(A) 512 (B) 729 (C) 1094 (D) 3281 (E) 59, 048

19 A number m is randomly selected from the set {11, 13, 15, 17, 19}, and a number n is randomly
selected from {1999, 2000, 2001, . . . , 2018}. What is the probability that mn has a units digit of
1?
1 1 3 7 2
(A) 5 (B) 4 (C) 10 (D) 20 (E) 5

20 A scanning code consists of a 7 × 7 grid of squares, with some of its squares colored black
and the rest colored white. There must be at least one square of each color in this grid of
49 squares. A scanning code is called symmetric if its look does not change when the entire
square is rotated by a multiple of 90◦ counterclockwise around its center, nor when it is re-
flected across a line joining opposite corners or a line joining midpoints of opposite sides.
What is the total number of possible symmetric scanning codes?

© 2019 AoPS Incorporated 4


AoPS Community 2018 AMC 10

(A) 510 (B) 1022 (C) 8190 (D) 8192 (E) 65,534

21 Which of the following describes the set of values of a for which the curves x2 + y 2 = a2 and
y = x2 − a in the real xy-plane intersect at exactly 3 points?
1 1 1 1 1 1
(A) a = 4 (B) 4 <a< 2 (C) a > 4 (D) a = 2 (E) a > 2

22 Let a, b, c, and d be positive integers such that gcd(a, b) = 24, gcd(b, c) = 36, gcd(c, d) = 54, and
70 < gcd(d, a) < 100. Which of the following must be a divisor of a?
(A) 5 (B) 7 (C) 11 (D) 13 (E) 17

23 Farmer Pythagoras has a field in the shape of a right triangle. The right triangle’s legs have
lengths 3 and 4 units. In the corner where those sides meet at a right angle, he leaves a small
unplanted square S so that from the air it looks like the right angle symbol. The rest of the
field is planted. The shortest distance from S to the hypotenuse is 2 units. What fraction of
the field is planted?

3
2

S 4

25 26 73 145 74
(A) 27 (B) 27 (C) 75 (D) 147 (E) 75

24 Triangle ABC with AB = 50 and AC = 10 has area 120. Let D be the midpoint of AB, and
let E be the midpoint of AC. The angle bisector of ∠BAC intersects DE and BC at F and G,
respectively. What is the area of quadrilateral F DBG?
(A) 60 (B) 65 (C) 70 (D) 75 (E) 80

25 For a positive integer n and nonzero digits a, b, and c, let An be the n-digit integer each of whose
digits is equal to a; let Bn be the n-digit integer each of whose digits is equal to b, and let Cn
be the 2n-digit (not n-digit) integer each of whose digits is equal to c. What is the greatest
possible value of a + b + c for which there are at least two values of n such that Cn − Bn = A2n ?
(A) 12 (B) 14 (C) 16 (D) 18 (E) 20

© 2019 AoPS Incorporated 5


AoPS Community 2018 AMC 10


These problems are copyright © Mathematical Association of America (http://maa.org).

– B

– February 15th, 2018

1 Kate bakes a 20-inch by 18-inch pan of cornbread. The cornbread is cut into pieces that mea-
sure 2 inches by 2 inches. How many pieces of cornbread does the pan contain?
(A) 90 (B) 100 (C) 180 (D) 200 (E) 360

2 Sam drove 96 miles in 90 minutes. His average speed during the first 30 minutes was 60 mph
(miles per hour), and his average speed during the second 30 minutes was 65 mph. What was
his average speed, in mph, during the last 30 minutes?
(A) 64 (B) 65 (C) 66 (D) 67 (E) 68

3 In the expression ( × )+( × ) each blank is to be filled in with one of the digits
1, 2, 3, or 4, with each digit being used once. How many different values can be obtained?
(A) 2 (B) 3 (C) 4 (D) 6 (E) 24

4 4. A three-dimensional rectangular box with dimensions X, Y , and Z has faces whose surface
areas are 24, 24, 48, 48, 72, and 72 square units. What is X + Y + Z?
(A) 18 (B) 22 (C) 24 (D) 30 (E) 36

5 How many subsets of {2, 3, 4, 5, 6, 7, 8, 9} contain at least one prime number?


(A) 128 (B) 192 (C) 224 (D) 240 (E) 256

6 A box contains 5 chips, numbered 1, 2, 3, 4, and 5. Chips are drawn randomly one at a time
without replacement until the sum of the values drawn exceeds 4. What is the probability that
3 draws are required?
1 1
(A) 15 (B) 10 (C) 16 (D) 15 (E) 14

7 In the figure below, N congruent semicircles lie on the diameter of a large semicircle, with their
diameters covering the diameter of the large semicircle with no overlap. Let A be the combined
area of the small semicircles and B be the area of the region inside the large semicircle but
outside the semicircles. The ratio A : B is 1 : 18. What is N ?

© 2019 AoPS Incorporated 6


AoPS Community 2018 AMC 10

...

(A) 16 (B) 17 (C) 18 (D) 19 (E) 36

8 Sara makes a staircase out of toothpicks as shown:

This is a 3-step staircase and uses 18 toothpicks. How many steps would be in a staircase
that used 180 toothpicks?
(A) 10 (B) 11 (C) 12 (D) 24 (E) 30

9 The faces of each of 7 standard dice are labeled with the integers from 1 to 6. Let p be the
probability that when all 7 dice are rolled, the sum of the numbers on the top faces is 10. What
other sum occurs with the same probability as p?
(A) 13 (B) 26 (C) 32 (D) 39 (E) 42

10 In the rectangular parallelpiped shown, AB = 3, BC = 1, and CG = 2. Point M is the midpoint


of F G. What is the volume of the rectangular pyramid with base BCHE and apex M ?

© 2019 AoPS Incorporated 7


AoPS Community 2018 AMC 10
H G

E
F
2

D C

A 3 B

4 3 5
(A) 1 (B) 3 (C) 2 (D) 3 (E) 2

11 Which of the following expressions is never a prime number when p is a prime number?
(A) p2 + 16 (B) p2 + 24 (C) p2 + 26 (D) p2 + 46 (E) p2 + 96

12 Line segment AB is a diameter of a circle with AB = 24. Point C, not equal to A or B, lies on
the circle. As point C moves around the circle, the centroid (center of mass) of 4ABC traces
out a closed curve missing two points. To the nearest positive integer, what is the area of the
region bounded by this curve?
(A) 25 (B) 38 (C) 50 (D) 63 (E) 75

13 How many of the first 2018 numbers in the sequence 101, 1001, 10001, 100001, . . . are divisible
by 101?
(A) 253 (B) 504 (C) 505 (D) 506 (E) 1009

14 A list of 2018 positive integers has a unique mode, which occurs exactly 10 times. What is the
least number of distinct values that can occur in the list?
(A) 202 (B) 223 (C) 224 (D) 225 (E) 234

15 A closed box with a square base is to be wrapped with a square sheet of wrapping paper. The
box is centered on the wrapping paper with the vertices of the base lying on the midlines of
the square sheet of paper, as shown in the figure on the left. The four corners of the wrapping
paper are to be folded up over the sides and brought together to meet at the center of the top
of the box, point A in the figure on the right. The box has base length w and height h. What is
the area of the sheet of wrapping paper?

© 2019 AoPS Incorporated 8


AoPS Community 2018 AMC 10
A

w w

(w+h)2
(A) 2(w + h)2 (B) 2 (C) 2w2 + 4wh (D) 2w2 (E) w2 h

16 Let a1 , a2 , . . . , a2018 be a strictly increasing sequence of positive integers such that

a1 + a2 + · · · + a2018 = 20182018 .

What is the remainder when a31 + a32 + · · · + a32018 is divided by 6?


(A) 0 (B) 1 (C) 2 (D) 3 (E) 4

17 In rectangle P QRS, P Q = 8 and QR = 6. Points A and B lie on P Q, points C and D lie on QR,
points E and F lie on RS, and points G and H lie on SP so that AP = BQ < 4 and the convex
octagon ABCDEF GH is equilateral. The length of a side of this octagon can be expressed

in the form k + m n, where k, m, and n are integers and n is not divisible by the square of any
prime. What is k + m + n?
(A) 1 (B) 7 (C) 21 (D) 92 (E) 106

18 Three young brother-sister pairs from different families need to take a trip in a van. These six
children will occupy the second and third rows in the van, each of which has three seats. To
avoid disruptions, siblings may not sit right next to each other in the same row, and no child
may sit directly in front of his or her sibling. How many seating arrangements are possible for
this trip?
(A) 60 (B) 72 (C) 92 (D) 96 (E) 120

19 Joey and Chloe and their daughter Zoe all have the same birthday. Joey is 1 year older than
Chloe, and Zoe is exactly 1 year old today. Today is the first of the 9 birthdays on which Chloe’s
age will be an integral multiple of Zoe’s age. What will be the sum of the two digits of Joey’s
age the next time his age is a multiple of Zoe’s age?
(A) 7 (B) 8 (C) 9 (D) 10 (E) 11

© 2019 AoPS Incorporated 9


AoPS Community 2018 AMC 10

20 A function f is defined recursively by f (1) = f (2) = 1 and

f (n) = f (n − 1) − f (n − 2) + n

for all integers n ≥ 3. What is f (2018)?


(A) 2016 (B) 2017 (C) 2018 (D) 2019 (E) 2020

21 Mary chose an even 4-digit number n. She wrote down all the divisors of n in increasing order
from left to right: 1, 2, ..., n2 , n. At some moment Mary wrote 323 as a divisor of n. What is the
smallest possible value of the next divisor written to the right of 323?
(A) 324 (B) 330 (C) 340 (D) 361 (E) 646

22 Real numbers x and y are chosen independently and uniformly at random from the interval
[0, 1]. Which of the following numbers is closest to the probability that x, y, and 1 are the side
lengths of an obtuse triangle?
(A) 0.21 (B) 0.25 (C) 0.29 (D) 0.50 (E) 0.79

23 How many ordered pairs (a, b) of positive integers satisfy the equation

a · b + 63 = 20 · lcm(a, b) + 12 · gcd(a, b),

where gcd(a, b) denotes the greatest common divisor of a and b, and lcm(a, b) denotes their
least common multiple?
(A) 0 (B) 2 (C) 4 (D) 6 (E) 8

24 Let ABCDEF be a regular hexagon with side length 1. Denote by X, Y, and Z the midpoints
of AB, CD, and EF , respectively. What is the area of the convex hexagon whose interior is the
intersection of the interiors of 4ACE and 4XY Z?
3√ 7√ 15 √ 1√ 9√
(A) 3 (B) 3 (C) 3 (D) 3 (E) 3
8 16 32 2 16

25 Let bxc denote the greatest integer less than or equal to x. How many real numbers x satisfy
the equation x2 + 10,000bxc = 10,000x?
(A) 197 (B) 198 (C) 199 (D) 200 (E) 201

© 2019 AoPS Incorporated 10


Art of Problem Solving is an ACS WASC Accredited School.
AoPS Community 2019 AMC 10

AMC 10 2019
www.artofproblemsolving.com/community/c448656
by LauraZed, VoidTV, kevinmathz, OmicronGamma, AOPS12142015, P Groudon, ilikepie2003, djmath-
man, eisirrational, phi ftw1618, avn, ItsAmeYushi, fidgetboss 4000, popcorn1, worldwidenoah1, Generic Username,
ev3commander, EZmath2006, Ultroid999OCPN, rrusczyk

– A

– February 7th, 2019

1 What is the value of  


19
0( )   1 9
2 + 20 ?

(A) 0 (B) 1 (C) 2 (D) 3 (E) 4

2 What is the hundreds digit of (20! − 15!)?


(A) 0 (B) 1 (C) 2 (D) 4 (E) 5

3 Ana and Bonita were born on the same date in different years, n years apart. Last year Ana
was 5 times as old as Bonita. This year Ana’s age is the square of Bonita’s age. What is n?
(A) 3 (B) 5 (C) 9 (D) 12 (E) 15

4 A box contains 28 red balls, 20 green balls, 19 yellow balls, 13 blue balls, 11 white balls, and 9
black balls. What is the minimum number of balls that must be drawn from the box without
replacement to guarantee that at least 15 balls of a single color will be drawn?
(A) 75 (B) 76 (C) 79 (D) 84 (E) 91

5 What is the greatest number of consecutive integers whose sum is 45?


(A) 9 (B) 25 (C) 45 (D) 90 (E) 120

6 For how many of the following types of quadrilaterals does there exist a point in the plane of
the quadrilateral that is equidistant from all four vertices of the quadrilateral?
- a square
-a rectangle that is not a square
- a rhombus that is not a square
- a parallelogram that is not a rectangle or a rhombus
- an isosceles trapezoid that is not a parallelogram
(A) 1 (B) 2 (C) 3 (D) 4 (E) 5

© 2020 AoPS Incorporated 1


AoPS Community 2019 AMC 10

1
7 Two lines with slopes and 2 intersect at (2, 2). What is the area of the triangle enclosed by
2
these two lines and the line x + y = 10?
√ √
(A) 4 (B) 4 2 (C) 6 (D) 8 (E) 6 2

8 The figure below shows line ` with a regular, infinite, recurring pattern of squares and line
segments.

How many of the following four kinds of rigid motion transformations of the plane in which
this figure is drawn, other than the identity transformation, will transform this figure into itself?

- some rotation around a point of line `


- some translation in the direction parallel to line `
- the reflection across line `
- some reflection across a line perpendicular to line `
(A) 0 (B) 1 (C) 2 (D) 3 (E) 4

9 What is the greatest three-digit positive integer n for which the sum of the first n positive
integers is not a divisor of the product of the first n positive integers?
(A) 995 (B) 996 (C) 997 (D) 998 (E) 999

10 A rectangular floor that is 10 feet wide and 17 feet long is tiled with 170 one-foot square tiles.
A bug walks from one corner to the opposite corner in a straight line. Including the first and
the last tile, how many tiles does the bug visit?
(A) 17 (B) 25 (C) 26 (D) 27 (E) 28

11 How many positive integer divisors of 2019 are perfect squares or perfect cubes (or both)?
(A) 32 (B) 36 (C) 37 (D) 39 (E) 41

12 Melanie computes the mean µ, the median M , and the modes of the 365 values that are the
dates in the months of 2019. Thus her data consist of 12 1s, 12 2s, . . . , 12 28s, 11 29s, 11 30s,
and 7 31s. Let d be the median of the modes. Which of the following statements is true?

© 2020 AoPS Incorporated 2


AoPS Community 2019 AMC 10

(A) µ < d < M (B) M < d < µ (C) d = M = µ (D) d < M < µ (E) d < µ < M

13 Let ∆ABC be an isosceles triangle with BC = AC and ∠ACB = 40◦ . Contruct the circle with
diameter BC, and let D and E be the other intersection points of the circle with the sides AC
and AB, respectively. Let F be the intersection of the diagonals of the quadrilateral BCDE.
What is the degree measure of ∠BF C?
(A) 90 (B) 100 (C) 105 (D) 110 (E) 120

14 For a set of four distinct lines in a plane, there are exactly N distinct points that lie on two or
more of the lines. What is the sum of all possible values of N ?
(A) 14 (B) 16 (C) 18 (D) 19 (E) 21

15 A sequence of numbers is defined recursively by a1 = 1, a2 = 73 , and


an−2 · an−1
an =
2an−2 − an−1
for all n ≥ 3 Then a2019 can be written as pq , where p and q are relatively prime positive inegers.
What is p + q?
(A) 2020 (B) 4039 (C) 6057 (D) 6061 (E) 8078

16 The figure below shows 13 circles of radius 1 within a larger circle. All the intersections occur
at points of tangency. What is the area of the region, shaded in the figure, inside the larger
circle but outside all the circles of radius 1?

√ √ √ √
(A) 4π 3 (B) 7π (C) π(3 3 + 2) (D) 10π( 3 − 1) (E) π( 3 + 6)

© 2020 AoPS Incorporated 3


AoPS Community 2019 AMC 10

17 A child builds towers using identically shaped cubes of different color. How many different
towers with a height 8 cubes can the child build with 2 red cubes, 3 blue cubes, and 4 green
cubes? (One cube will be left out.)
(A) 24 (B) 288 (C) 312 (D) 1, 260 (E) 40, 320

18 For some positive integer k, the repeating base-k representation of the (base-ten) fraction 7
51
is 0.23k = 0.232323...k . What is k?
(A) 13 (B) 14 (C) 15 (D) 16 (E) 17

19 What is the least possible value of

(x + 1)(x + 2)(x + 3)(x + 4) + 2019

where x is a real number?


(A) 2017 (B) 2018 (C) 2019 (D) 2020 (E) 2021

20 The numbers 1, 2, . . . , 9 are randomly placed into the 9 squares of a 3 × 3 grid. Each square
gets one number, and each of the numbers is used once. What is the probability that the sum
of the numbers in each row and each column is odd?
(A) 1/21 (B) 1/14 (C) 5/63 (D) 2/21 (E) 1/7

21 A sphere with center O has radius 6. A triangle with sides of length 15, 15, and 24 is situated in
space so that each of its sides is tangent to the sphere. What is the distance between O and
the plane determined by the triangle?
√ √ √
(A) 2 3 (B) 4 (C) 3 2 (D) 2 5 (E) 5

22 Real numbers between 0 and 1, inclusive, are chosen in the following manner. A fair coin is
flipped. If it lands heads, then it is flipped again and the chosen number is 0 if the second flip
is heads and 1 if the second flip is tails. On the other hand, if the first coin flip is tails, then the
number is chosen uniformly at random from the closed interval [0, 1]. Two random numbers x
and y are chosen independently in this manner. What is the probability that |x − y| > 21 ?
(A) 13 7
(B) 16 (C) 12 9
(D) 16 (E) 23

23 Travis has to babysit the terrible Thompson triplets. Knowing that they love big numbers,
Travis devises a counting game for them. First Tadd will say the number 1, then Todd must
say the next two numbers (2 and 3), then Tucker must say the next three numbers (4, 5, 6), then
Tadd must say the next four numbers (7, 8, 9, 10), and the process continues to rotate through
the three children in order, each saying one more number than the previous child did, until the
number 10, 000 is reached. What is the 2019th number said by Tadd?
(A) 5743 (B) 5885 (C) 5979 (D) 6001 (E) 6011

© 2020 AoPS Incorporated 4


AoPS Community 2019 AMC 10

24 Let p, q, and r be the distinct roots of the polynomial x3 − 22x2 + 80x − 67. It is given that there
exist real numbers A, B, and C such that
1 A B C
= + +
s3 − 22s2 + 80s − 67 s−p s−q s−r

for all s 6∈ {p, q, r}. What is 1


A + 1
B + C?
1

(A) 243 (B) 244 (C) 245 (D) 246 (E) 247

25 For how many integers n between 1 and 50, inclusive, is

(n2 − 1)!
(n!)n

an integer? (Recall that 0! = 1.)


(A) 31 (B) 32 (C) 33 (D) 34 (E) 35

– B

– February 13th, 2019

1 Alicia had two containers. The first was 65 full of water and the second was empty. She poured
all the water from the first container into the second container, at which point the second
container was 34 full of water. What is the ratio of the volume of the first container to the volume
of the second container?
5 4 7 9 11
(A) 8 (B) 5 (C) 8 (D) 10 (E) 12

2 Consider the statement, ”If n is not prime, then n − 2 is prime.” Which of the following values
of n is a counterexample to this statement?
(A) 11 (B) 15 (C) 19 (D) 21 (E) 27

3 In a high school with 500 students, 40% of the seniors play a musical instrument, while 30% of
the non-seniors do not play a musical instrument. In all, 46.8% of the students do not play a
musical instrument. How many non-seniors play a musical instrument?
(A) 66 (B) 154 (C) 186 (D) 220 (E) 266

4 All lines with equation ax+by = c such that a, b, c form an arithmetic progression pass through
a common point. What are the coordinates of that point?
(A) (−1, 2) (B) (0, 1) (C) (1, −2) (D) (1, 0) (E) (1, 2)

© 2020 AoPS Incorporated 5


AoPS Community 2019 AMC 10

5 Triangle ABC lies in the first quadrant. Points A, B, and C are reflected across the line y = x
to points A0 , B 0 , and C 0 , respectively. Assume that none of the vertices of the triangle lie on
the line y = x. Which of the following statements is not always true? (A) Triangle A0 B 0 C 0 lies
in the first quadrant. (B) Triangles ABC and A0 B 0 C 0 have the same area. (C) The slope of
line AA0 is −1. (D) The slopes of lines AA0 and CC 0 are the same. (E) Lines AB and A0 B 0 are
perpendicular to each other.

6 A positive integer n satisfies the equation (n + 1)! + (n + 2)! = n! · 440. What is the sum of the
digits of n?
(A) 2 (B) 5 (C) 10 (D) 12 (E) 15

7 Each piece of candy in a store costs a whole number of cents. Casper has exactly enough
money to buy either 12 pieces of red candy, 14 pieces of green candy, 15 pieces of blue candy,
or n pieces of purple candy. A piece of purple candy costs 20 cents. What is the smallest
possible value of n?
(A) 18 (B) 21 (C) 24 (D) 25 (E) 28

8 The figure below shows a square and four equilateral triangles, with each triangle having a side
lying on a side of the square, such that each triangle has side length 2 and the third vertices
of the triangles meet at the center of the square. The region inside the square but outside the
triangles is shaded. What is the area of the shaded region?

√ √ √ √
(A) 4 (B) 12 − 4 3 (C) 3 3 (D) 4 3 (E) 16 − 3

9 The function f is defined by j k


f (x) = |x| − bxc

for all real numbers x, where brc denotes the greatest integer less than or equal to the real

© 2020 AoPS Incorporated 6


AoPS Community 2019 AMC 10

number r. What is the range of f ?


(A) {−1, 0} (B) The set of nonpositive integers (C) {−1, 0, 1}
(D) {0} (E) The set of nonnegative integers

10 In a given plane, points A and B are 10 units apart. How many points C are there in the plane
such that the perimeter of 4ABC is 50 units and the area of 4ABC is 100 square units?
(A) 0 (B) 2 (C) 4 (D) 8 (E) infinitely many

11 Two jars each contain the same number of marbles, and every marble is either blue or green.
In Jar 1 the ratio of blue to green marbles is 9:1, and the ratio of blue to green marbles in Jar 2
is 8:1. There are 95 green marbles in all. How many more blue marbles are in Jar 1 than in Jar
2?
(A) 5 (B) 10 (C) 25 (D) 45 (E) 50

12 What is the greatest possible sum of the digits in the base-seven representation of a positive
integer less than 2019?
(A) 11 (B) 14 (C) 22 (D) 23 (E) 27

13 What is the sum of all real numbers x for which the median of the numbers 4, 6, 8, 17, and x is
equal to the mean of those five numbers?
15 35
(A) − 5 (B) 0 (C) 5 (D) 4 (E) 4

14 The base-ten representation for 19! is 121, 6T 5, 100, 40M, 832, H00, where T , M , and H denote
digits that are not given. What is T + M + H?
(A) 3 (B) 8 (C) 12 (D) 14 (E) 17

15 Two right triangles, T1 and T2 , have areas of 1 and 2, respectively. One side length of one
triangle is congruent to a different side length in the other, and another side length of the first
triangle is congruent to yet another side length in the other. What is the square of the product
of the third side lengths of T1 and T2 ?
28 32 34
(A) 3 (B) 10 (C) 3 (D) 3 (E) 12

16 In 4ABC with a right angle at C, point D lies in the interior of AB and point E lies in the
interior of BC so that AC = CD, DE = EB, and the ratio AC : DE = 4 : 3. What is the ratio
AD : DB?
√ √
(A) 2 : 3 (B) 2 : 5 (C) 1 : 1 (D) 3 : 5 (E) 3 : 2

© 2020 AoPS Incorporated 7


AoPS Community 2019 AMC 10

17 A red ball and a green ball are randomly and independently tossed into bins numbered with
positive integers so that for each ball, the probability that it is tossed into bin k is 2−k for
k = 1, 2, 3, . . . . What is the probability that the red ball is tossed into a higher-numbered bin
than the green ball?
1 2 1 3 3
(A) 4 (B) 7 (C) 3 (D) 8 (E) 7

18 Henry decides one morning to do a workout, and he walks 34 of the way from his home to his
gym. The gym is 2 kilometers away from Henry’s home. At that point, he changes his mind
and walks 34 of the way from where he is back toward home. When he reaches that point, he
changes his mind again and walks 43 of the distance from there back toward the gym. If Henry
keeps changing his mind when he has walked 34 of the distance toward either the gym or home
from the point where he last changed his mind, he will get very close to walking back and forth
between a point A kilometers from home and a point B kilometers from home. What is |A−B|?
2
(A) 3 (B) 1 (C) 1 51 (D) 1 41 (E) 1 12

19 Let S be the set of all positive integer divisors of 100, 000. How many numbers are the product
of two distinct elements of S?
(A) 98 (B) 100 (C) 117 (D) 119 (E) 121

As shown in the figure, line segment AD_ is trisected by points B and C so that AB = BC =
20
_ _
CD = 2. Three semicircles of radius 1, AEB, BFC, and CGD, have their diameters on AD, and
are tangent to line EG at E, F, and G, respectively. A circle of radius 2 has its center on F. The
area of the region inside the circle but outside the three semicircles, shaded in the figure, can
be expressed in the form
a √
· π − c + d,
b
where a, b, c, and d are positive integers and a and b are relatively prime. What is a + b + c + d?

E F G

A B C D

(A) 13 (B) 14 (C) 15 (D) 16 (E) 17

© 2020 AoPS Incorporated 8


AoPS Community 2019 AMC 10

21 Debra flips a fair coin repeatedly, keeping track of how many heads and how many tails she
has seen in total, until she gets either two heads in a row or two tails in a row, at which point
she stops flipping. What is the probability that she gets two heads in a row but she sees a
second tail before she sees a second head?
1 1 1 1 1
(A) 36 (B) 24 (C) 18 (D) 12 (E) 6

22 Raashan, Sylvia, and Ted play the following game. Each starts with $1. A bell rings every 15
seconds, at which time each of the players who currently have money simultaneously chooses
one of the other two players independently and at random and gives $1 to that player. What is
the probability that after the bell has rung 2019 times, each player will have $1? (For example,
Raashan and Ted may each decide to give $1 to Sylvia, and Sylvia may decide to give her dollar
to Ted, at which point Raashan will have $0, Sylvia would have $2, and Ted would have $1, and
and that is the end of the first round of play. In the second round Raashan has no money to
give, but Sylvia and Ted might choose each other to give their $1 to, and and the holdings will
be the same as the end of the second [sic] round.
1 1 1 1 2
(A) 7 (B) 4 (C) 3 (D) 2 (E) 3

23 Points A(6, 13) and B(12, 11) lie on circle ω in the plane. Suppose that the tangent lines to ω
at A and B intersect at a point on the x-axis. What is the area of ω?
83π 21π 85π 43π 87π
(A) 8 (B) 2 (C) 8 (D) 4 (E) 8

24 Define a sequence recursively by x0 = 5 and


x2n + 5xn + 4
xn+1 =
xn + 6
for all nonnegative integers n. Let m be the least positive integer such that
1
xm ≤ 4 + .
220
In which of the following intervals does m lie?
(A) [9, 26] (B) [27, 80] (C) [81, 242] (D) [243, 728] (E) [729, ∞]

25 How many sequences of 0s and 1s of length 19 are there that begin with a 0, end with a 0,
contain no two consecutive 0s, and contain no three consecutive 1s?
(A) 55 (B) 60 (C) 65 (D) 70 (E) 75


These problems are copyright © Mathematical Association of America (http://maa.org).

© 2020 AoPS Incorporated 9


AoPS Community 2019 AMC 10

© 2020 AoPS Incorporated 10


Art of Problem Solving is an ACS WASC Accredited School.
AoPS Community 2020 AMC 10

AMC 10 2020
www.artofproblemsolving.com/community/c1116106
by djmathman, countTO100, Frestho, Stran1234, GCA, Airplane50, P Groudon, franchester, owm, kc5170,
Owjebra, aie8920, kootrapali, a1b2, nsato, euler123, austinchen2005, Eyed, TheMathateer, Susanssluk,
montana mathlete, Williamgolly, proshi, Welp..., naenaendr, AOPS12142015, rrusczyk

– A

1 What value of x satisfies

3 5 1
x− = − ?
4 12 3
2 7 7 2 5
(A) − 3 (B) 36 (C) 12 (D) 3 (E) 6

2 The numbers 3, 5, 7, a, and b have an average (arithmetic mean) of 15. What is the average of
a and b?
(A) 0 (B) 15 (C) 30 (D) 45 (E) 60

3 Assuming a 6= 3, b 6= 4, and c 6= 5, what is the value in simplest form of the following expres-
sion?
a−3 b−4 c−5
· ·
5−c 3−a 4−b
abc 1 1 1 1
(A) − 1 (B) 1 (C) 60 (D) abc − 60 (E) 60 − abc

4 A driver travels for 2 hours at 60 miles per hour, during which her car gets 30 miles per gallon
of gasoline. She is paid $0.50 per mile, and her only expense is gasoline at $2.00 per gallon.
What is her net rate of pay, in dollars per hour, after this expense?
(A) 20 (B) 22 (C) 24 (D) 25 (E) 26

5 What is the sum of all real numbers x for which |x2 − 12x + 34| = 2?
(A) 12 (B) 15 (C) 18 (D) 21 (E) 25

6 How many 4-digit positive integers (that is, integers between 1000 and 9999, inclusive) having
only even digits are divisible by 5?
(A) 80 (B) 100 (C) 125 (D) 200 (E) 500

7 The 25 integers from −10 to 14, inclusive, can be arranged to form a 5-by-5 square in which the
sum of the numbers in each row, the sum of the numbers in each column, and the sum of the

© 2020 AoPS Incorporated 1


AoPS Community 2020 AMC 10

numbers along each of the main diagonals are all the same. What is the value of this common
sum?
(A) 2 (B) 5 (C) 10 (D) 25 (E) 50

8 What is the value of

1 + 2 + 3 − 4 + 5 + 6 + 7 − 8 + · · · + 197 + 198 + 199 − 200?

(A) 9, 800 (B) 9, 900 (C) 10, 000 (D) 10, 100 (E) 10, 200

9 A single bench section at a school event can hold either 7 adults or 11 children. When N bench
sections are connected end to end, an equal number of adults and children seated together
will occupy all the bench space. What is the least possible positive integer value of N ?
(A) 9 (B) 18 (C) 27 (D) 36 (E) 77

10 Seven cubes, whose volumes are 1, 8, 27, 64, 125, 216, and 343 cubic units, are stacked vertically
to form a tower in which the volumes of the cubes decrease from bottom to top. Except for
the bottom cube, the bottom face of each cube lies completely on top of the cube below it.
What is the total surface area of the tower (including the bottom) in square units?
(A) 644 (B) 658 (C) 664 (D) 720 (E) 749

11 What is the median of the following list of 4040 numbers?

1, 2, 3, ..., 2020, 12 , 22 , 32 , ..., 20202


(A) 1974.5 (B) 1975.5 (C) 1976.5 (D) 1977.5 (E) 1978.5

12 Triangle AM C is isoceles with AM = AC. Medians M V and CU are perpendicular to each


other, and M V = CU = 12. What is the area of 4AM C?

U V

M C

© 2020 AoPS Incorporated 2


AoPS Community 2020 AMC 10

(A) 48 (B) 72 (C) 96 (D) 144 (E) 192

13 A frog sitting at the point (1, 2) begins a sequence of jumps, where each jump is parallel to one
of the coordinate axes and has length 1, and the direction of each jump (up, down, right, or left)
is chosen independently at random. The sequence ends when the frog reaches a side of the
square with vertices (0, 0), (0, 4), (4, 4), and (4, 0). What is the probability that the sequence of
jumps ends on a vertical side of the square?
1 5 2 3 7
(A) 2 (B) 8 (C) 3 (D) 4 (E) 8

14 Real numbers x and y satisfy x + y = 4 and x · y = −2. What is the value of

x3 y3
x+ + + y?
y2 x2

(A) 360 (B) 400 (C) 420 (D) 440 (E) 480

15 A positive integer divisor of 12! is chosen at random. The probability that the divisor chosen is
a perfect square can be expressed as m n , where m and n are relatively prime positive integers.
What is m + n?
(A) 3 (B) 5 (C) 12 (D) 18 (E) 23

16 A point is chosen at random within the square in the coordinate plane whose vertices are
(0, 0), (2020, 0), (2020, 2020), and (0, 2020). The probability that the point is within d units of a
lattice point is 21 . (A point (x, y) is a lattice point if x and y are both integers.) What is d to the
nearest tenth?
(A) 0.3 (B) 0.4 (C) 0.5 (D) 0.6 (E) 0.7

17 Define
P (x) = (x − 12 )(x − 22 ) · · · (x − 1002 ).
How many integers n are there such that P (n) ≤ 0?
(A) 4900 (B) 4950 (C) 5000 (D) 5050 (E) 5100

18 Let (a, b, c, d) be an ordered quadruple of not necessarily distinct integers, each one of them in
the set {0, 1, 2, 3}. For how many such quadruples is it true that a · d − b · c is odd? (For example,
(0, 3, 1, 1) is one such quadruple, because 0 · 1 − 3 · 1 = −3 is odd.)
(A) 48 (B) 64 (C) 96 (D) 128 (E) 192

19 As shown in the figure below a regular dodecahedron (the polyhedron consisting of 12 con-
gruent regular pentagonal faces) floats in space with two horizontal faces. Note that there is
a ring of five slanted faces adjacent to the top face, and a ring of five slanted faces adjacent

© 2020 AoPS Incorporated 3


AoPS Community 2020 AMC 10

to the bottom face. How many ways are there to move from the top face to the bottom face
via a sequence of adjacent faces so that each face is visited at most once and moves are not
permitted from the bottom ring to the top ring?

(A) 125 (B) 250 (C) 405 (D) 640 (E) 810

20 Quadrilateral ABCD satisfies ∠ABC = ∠ACD = 90◦ , AC = 20, and CD = 30. Diagonals AC
and BD intersect at point E, and AE = 5. What is the area of quadrilateral ABCD?
(A) 330 (B) 340 (C) 350 (D) 360 (E) 370

21 There exists a unique strictly increasing sequence of nonnegative integers a1 < a2 < < ak
such that
2289 + 1
= 2a1 + 2a2 + + 2ak .
217 + 1
What is k?
(A) 117 (B) 136 (C) 137 (D) 273 (E) 306

22 For how many positive integers n ≤ 1000 is


     
998 999 1000
+ +
n n n

not divisible by 3? (Recall that bxc is the greatest integer less than or equal to x.)
(A) 22 (B) 23 (C) 24 (D) 25 (E) 26

23 Let T be the triangle in the coordinate plane with vertices (0, 0), (4, 0), and (0, 3). Consider
the following five isometries (rigid transformations) of the plane: rotations of 90◦ , 180◦ , and
270◦ counterclockwise around the origin, reflection across the x-axis, and reflection across
the y-axis. How many of the 125 sequences of three of these transformations (not necessar-
ily distinct) will return T to its original position? (For example, a 180◦ rotation, followed by a

© 2020 AoPS Incorporated 4


AoPS Community 2020 AMC 10

reflection across the x-axis, followed by a reflection across the y-axis will return T to its origi-
nal position, but a 90◦ rotation, followed by a reflection across the x-axis, followed by another
reflection across the x-axis will not return T to its original position.)
(A) 12 (B) 15 (C) 17 (D) 20 (E) 25

24 Let n be the least positive integer greater than 1000 for which

gcd(63, n + 120) = 21 and gcd(n + 63, 120) = 60.

What is the sum of the digits of n?


(A) 12 (B) 15 (C) 18 (D) 21 (E) 24

25 Jason rolls three fair standard six-sided dice. Then he looks at the rolls and chooses a subset
of the dice (possibly empty, possibly all three dice) to reroll. After rerolling, he wins if and only if
the sum of the numbers face up on the three dice is exactly 7. Jason always plays to optimize
his chances of winning. What is the probability that he chooses to reroll exactly two of the
dice?
7 5 2 17 1
(A) 36 (B) 24 (C) 9 (D) 72 (E) 4

– B

1 What is the value of


1 − (−2) − 3 − (−4) − 5 − (−6)?
(A) − 20 (B) − 3 (C) 3 (D) 5 (E) 21

2 Carl has 5 cubes each having side length 1, and Kate has 5 cubes each having side length 2.
What is the total volume of the 10 cubes?
(A) 24 (B) 25 (C) 28 (D) 40 (E) 45

3 The ratio of w to x is 4 : 3, the ratio of y to z is 3 : 2, and the ratio of z to x is 1 : 6. What is the


ratio of w to y?
(A) 4 : 3 (B) 3 : 2 (C) 8 : 3 (D) 4 : 1 (E) 16 : 3

4 The acute angles of a right triangle are a◦ and b◦ , where a > b and both a and b are prime
numbers. What is the least possible value of b?
(A) 2 (B) 3 (C) 5 (D) 7 (E) 11

5 How many distinguishable arrangements are there of 1 brown tile, 1 purple tile, 2 green tiles,
and 3 yellow tiles in a row from left to right? (Tiles of the same color are indistinguishable)
(A) 210 (B) 420 (C) 630 (D) 840 (E) 1050

© 2020 AoPS Incorporated 5


AoPS Community 2020 AMC 10

6 Driving along a highway, Megan noticed that her odometer showed 15951 (miles). This number
is a palindromeit reads the same forward and backward. Then 2 hours later, the odometer
displayed the next higher palindrome. What was her average speed, in miles per hour, during
this 2-hour period?
(A) 50 (B) 55 (C) 60 (D) 65 (E) 70

7 How many positive even multiples of 3 less than 2020 are perfect squares?
(A) 7 (B) 8 (C) 9 (D) 10 (E) 12

8 Points P and Q lie in a plane with P Q = 8. How many locations for point R in this plane are
there such that the triangle with vertices P, Q, and R is a right triangle with area 12 square
units?
(A) 2 (B) 4 (C) 6 (D) 8 (E) 12

9 How many ordered pairs of integers (x, y) satisfy the equation

x2020 + y 2 = 2y?

(A) 1 (B) 2 (C) 3 (D) 4 (E) infinitely many

10 A three-quarter sector of a circle of radius 4 inches together with its interior can be rolled up
to form the lateral surface area of a right circular cone by taping together along the two radii
shown. What is the volume of the cone in cubic inches?

√ √ √ √ √
(A) 3π 5 (B) 4π 3 (C) 3π 7 (D) 6π 3 (E) 6π 7

© 2020 AoPS Incorporated 6


AoPS Community 2020 AMC 10

11 Ms. Carr asks her students to read any 5 of the 10 books on a reading list. Harold randomly
selects 5 books from this list, and Betty does the same. What is the probability that there are
exactly 2 books that they both select?
1 5 14 25 1
(A) 8 (B) 36 (C) 45 (D) 63 (E) 2

12 The decimal representation of


1
2020
consists of a string of zeros after the decimal point, followed by a 9 and then several more
digits. How many zeros are in that initial string of zeros after the decimal point?
(A) 23 (B) 24 (C) 25 (D) 26 (E) 27

13 Andy the Ant lives on a coordinate plane and is currently at (−20, 20) facing east (that is, in
the positive x-direction). Andy moves 1 unit and then turns 90◦ degrees left. From there, Andy
moves 2 units (north) and then turns 90◦ degrees left. He then moves 3 units (west) and again
turns 90◦ degrees left. Andy continues his progress, increasing his distance each time by 1
unit and always turning left. What is the location of the point at which Andy makes the 2020th
left turn?
(A) (−1030, −994) (B) (−1030, −990) (C) (−1026, −994) (D) (−1026, −990) (E) (−1022, −994)

14 As shown in the figure below, six semicircles lie in the interior of a regular hexagon with side
length 2 so that the diameters of the semicircles coincide with the sides of the hexagon. What
is the area of the shaded regioninside the hexagon but outside all of the semicircles?

√ √
9 3

3 3 π

(A) 6 3 − 3π (B) 2 − 2π (C) 2 − 3 (D) 3 3 − π

9 3
(E) 2 −π

© 2020 AoPS Incorporated 7


AoPS Community 2020 AMC 10

15 Steve wrote the digits 1, 2, 3, 4, and 5 in order repeatedly from left to right, forming a list of
10, 000 digits, beginning 123451234512 . . . . He then erased every third digit from his list (that
is, the 3rd, 6th, 9th, . . . digits from the left), then erased every fourth digit from the resulting list
(that is, the 4th, 8th, 12th, . . . digits from the left in what remained), and then erased every fifth
digit from what remained at that point. What is the sum of the three digits that were then in
the positions 2019, 2020, 2021?
(A) 7 (B) 9 (C) 10 (D) 11 (E) 12

16 Bela and Jenn play the following game on the closed interval [0, n] of the real number line,
where n is a fixed integer greater than 4. They take turns playing, with Bela going first. At his
first turn, Bela chooses any real number in the interval [0, n]. Thereafter, the player whose turn it
is chooses a real number that is more than one unit away from all numbers previously chosen
by either player. A player unable to choose such a number loses. Using optimal strategy, which
player will win the game?
(A) Bela will always win. (B) Jenn will always win. (C) Bela will win if and only if n is odd. (D) Jenn will win
(E) Jenn will win if and only if n > 8.

17 There are 10 people standing equally spaced around a circle. Each person knows exactly 3
of the other 9 people: the 2 people standing next to her or him, as well as the person directly
across the circle. How many ways are there for the 10 people to split up into 5 pairs so that
the members of each pair know each other?
(A) 11 (B) 12 (C) 13 (D) 14 (E) 15

18 An urn contains one red ball and one blue ball. A box of extra red and blue balls lie nearby.
George performs the following operation four times: he draws a ball from the urn at random
and then takes a ball of the same color from the box and returns those two matching balls to
the urn. After the four iterations the urn contains six balls. What is the probability that the urn
contains three balls of each color?
1 1 1 1 1
(A) 6 (B) 5 (C) 4 (D) 3 (E) 2

19 In a certain card game, a player is dealt a hand of 10 cards from a deck of 52 distinct cards.
The number of distinct (unordered) hands that can be dealt to the player can be written as
158A00A4AA0. What is the digit A?
(A) 2 (B) 3 (C) 4 (D) 6 (E) 7

20 Let B be a right rectangular prism (box) with edges lengths 1, 3, and 4, together with its interior.
For real r ≥ 0, let S(r) be the set of points in 3-dimensional space that lie within a distance r
of some point B. The volume of S(r) can be expressed as ar3 + br2 + cr + d, where a, b, c, and
d are positive real numbers. What is ad bc
?

© 2020 AoPS Incorporated 8


AoPS Community 2020 AMC 10

(A) 6 (B) 19 (C) 24 (D) 26 (E) 38

21 In square ABCD, points E and H lie on AB and DA, respectively, so that AE = AH. Points
F and G lie on BC and CD, respectively, and points I and J lie on EH so that F I ⊥ EH and
GJ ⊥ EH. See the figure below. Triangle AEH, quadrilateral BF IE, quadrilateral DHJG, and
pentagon F CGJI each has area 1. What is F I 2 ?

D G C

F
H

A E B

7
√ √ 7
√ √
(A) 3 (B) 8 − 4 2 (C) 1 + 2 (D) 4 2 (E) 2 2

22 What is the remainder when 2202 + 202 is divided by 2101 + 251 + 1?


(A) 100 (B) 101 (C) 200 (D) 201 (E) 202

23 Square ABCD in the coordinate plane has vertices at the points A(1, 1), B(−1, 1), C(−1, −1),
and D(1, −1). Consider the following four transformations:

-L, a rotation of 90◦ counterclockwise around the origin;


-R, a rotation of 90◦ clockwise around the origin;
-H, a reflection across the x-axis; and
-V, a reflection across the y-axis.

Each of these transformations maps the squares onto itself, but the positions of the labeled
vertices will change. For example, applying R and then V would send the vertex A at (1, 1) to
(−1, −1) and would send the vertex B at (−1, 1) to itself. How many sequences of 20 trans-
formations chosen from {L, R, H, V } will send all of the labeled vertices back to their original
positions? (For example, R, R, V, H is one sequence of 4 transformations that will send the
vertices back to their original positions.)
(A) 237 (B) 3 · 236 (C) 238 (D) 3 · 237 (E) 239

© 2020 AoPS Incorporated 9


AoPS Community 2020 AMC 10

24 How many positive integers n satisfy


n + 1000 √
= b nc?
70
(Recall that bxc is the greatest integer not exceeding x.)
(A) 2 (B) 4 (C) 6 (D) 30 (E) 32

25 Let D(n) denote the number of ways of writing the positive integer n as a product

n = f1 · f2 · · · fk ,

where k ≥ 1, the fi are integers strictly greater than 1, and the order in which the factors
are listed matters (that is, two representations that differ only in the order of the factors are
counted as distinct). For example, the number 6 can be written as 6, 2 · 3, and 3 · 2, so D(6) = 3.
What is D(96)?
(A) 112 (B) 128 (C) 144 (D) 172 (E) 184


These problems are copyright © Mathematical Association of America (http://maa.org).

© 2020 AoPS Incorporated 10


Art of Problem Solving is an ACS WASC Accredited School.
AoPS Community 2021 AMC 10 -

www.artofproblemsolving.com/community/c1975144
by P Groudon, Professor-Mom, MathArt4, MathPirate101, A1001, fidgetboss 4000, Archimedes15, fran-
zliszt, samrocksnature, IceWolf10, kevinmathz, advancedjus, Grizzy, franchester, mathleticguyyy, aaja3427,
montana mathlete, knightime1010, forester2015, rrusczyk

– A

– February 4th, 2021

1 What is the value of


(22 − 2) − (32 − 3) + (42 − 4)?
(A) 1 (B) 2 (C) 5 (D) 8 (E) 12

2 Portias high school has 3 times as many students as Laras high school. The two high schools
have a total of 2600 students. How many students does Portias high school have? (A) 600 (B) 650 (C

3 The sum of two natural numbers is 17, 402. One of the two numbers is divisible by 10. If the
units digit of that number is erased, the other number is obtained. What is the difference of
these two numbers?
(A) 10, 272 (B) 11, 700 (C) 13, 362 (D) 14, 238 (E) 15, 426

4 A cart rolls down a hill, traveling 5 inches the first second and accelerating so that each suc-
cessive 1-second time
interval, it travels 7 inches more than during the previous 1-second interval. The cart takes 30
seconds to reach the
bottom of the hill. How far, in inches, does it travel? (A) 215 (B) 360 (C) 2992 (D) 3195 (E) 324

5 The quiz scores of a class with k > 12 students have a mean of 8. The mean of a collection of
12 of these quiz scores is 14. What is the mean of the remaining quiz scores in terms of k?
14−8 8k−168 14 k 14(k−12) 14(k−12)
(A) k−12 (B) k−12 (C) 12 − 8 (D) k2
(E) 8k

6 Chantal and Jean start hiking from a trailhead toward a fire tower. Jean is wearing a heavy
backpack and walks slower. Chantal starts walking at miles per hour. Halfway to the tower,
the trail becomes really steep, and Chantal slows down to miles per hour. After reaching the
tower, she immediately turns around and descends the steep part of the trail at miles per hour.
She meets Jean at the halfway point. What was Jean’s average speed, in miles per hour, until
they meet?

© 2021 AoPS Incorporated 1


AoPS Community 2021 AMC 10 -
12 13 24
(A) 13 (B) 1 (C) 12 (D) 13 (E) 2

7 Tom has a collection of 13 snakes, 4 of which are purple and 5 of which are happy. He observes
that • all of his happy snakes can add • none of his purple snakes can subtract, and • all of
his snakes that cant subtract also cant add
Which of these conclusions can be drawn about Toms snakes? (A) Purple snakes can add. (B)
Purple snakes are happy. (C) Snakes that can add are purple. (D) Happy snakes are not purple.
(E) Happy snakes can’t subtract.

8 When a student multiplied the number 66 by the repeating decimal,

1.abab = 1.ab,

where a and b are digits, he did not notice the notation and just multiplied 66 times 1.ab. Later
he found that his answer is 0.5 less than the correct answer. What is the 2- digit integer ab?
(A) 15 (B) 30 (C) 45 (D) 60 (E) 75

9 What is the least possible value of (xy − 1)2 + (x + y)2 for real numbers x and y?
1 1
(A) 0 (B) 4 (C) 2 (D) 1 (E) 2

10 Which of the following is equivalent to

(2 + 3)(22 + 32 )(24 + 34 )(28 + 38 )(216 + 316 )(232 + 332 )(264 + 364 )?

(A) 3127 +2127 (B) 3127 +2127 +2·363 +3·263 (C) 3128 −2128 (D) 3128 +2128 (E) 5127

11 11) For which of the following integers b is the base-b number 2021b − 221b not divisible by 3?
(A) 3 (B) 4 (C) 6 (D) 7 (E) 8

12 Two right circular cones with vertices facing down as shown in the figure below contain the
same amount of liquid. The radii of the tops of the liquid surfaces are 3 cm and 6 cm. Into
each cone is dropped a spherical marble of radius 1 cm, which sinks to the bottom and is
completely submerged without spilling any liquid. What is the ratio of the rise of the liquid
level in the narrow cone to the rise of the liquid level in the wide cone?
(A) 1 : 1 (B) 47 : 43 (C) 2 : 1 (D) 40 : 13 (E) 4 : 1

© 2021 AoPS Incorporated 2


AoPS Community 2021 AMC 10 -

13 What
√ is the volume
√ of tetrahedron ABCD with edge lengths AB = 2, AC = 3, AD = 4, BC =
13, BD = 2 5, and CD = 5?
√ √
(A) 3 (B) 2 3 (C) 4 (D) 3 3 (E) 6

14 All the roots of polynomial z 6 − 10z 5 + Az 4 + Bz 3 + Cz 2 + Dz + 16 are positive integers. What


is the value of B?
(A) − 88 (B) − 80 (C) − 64 (D) − 41 (E) − 40

15 Values for A, B, C, and D are to be selected from {1, 2, 3, 4, 5, 6} without replacement (i.e. no
two letters have the same value). How many ways are there to make such choices so that the
two curves y = Ax2 + B and y = Cx2 + D intersect? (The order in which the curves are listed
does not matter; for example, the choices A = 3, B = 2, C = 4, D = 1 is considered the same
as the choices A = 4, B = 1, C = 3, D = 2.)
(A) 30 (B) 60 (C) 90 (D) 180 (E) 360

16 In the following list of numbers, the integer n appears n times in the list for 1 ≤ n ≤ 200.

1, 2, 2, 3, 3, 3, 4, 4, 4, 4, , 200, 200, , 200

What is the median of the numbers in this list?


(A) 100.5 (B) 134 (C) 142 (D) 150.5 (E) 167

17 Trapezoid ABCD has AB k CD, BC = CD = 43, and AD ⊥ BD. Let O be the intersection of
the diagonals AC and BD, and let P be the midpoint of BD. GIven that OP = 11, the length

© 2021 AoPS Incorporated 3


AoPS Community 2021 AMC 10 -

AD can be written in the form m n, where m and n are positive integers and n is not divisible
by the square of any prime. What is m + n?
(A) 65 (B) 132 (C) 157 (D) 194 (E) 215

18 Let f be a function defined on the set of positive rational numbers with the property that f (a ·
b) = f (a) + f (b) for all positive rational numbers a and b. Suppose that f also has the property
that f (p) = p for every prime number p. For which of the following numbers x is f (x) < 0?
17 11 7 7 25
(A) 32 (B) 16 (C) 9 (D) 6 (E) 11

19 The area of the region bounded by the graph of

x2 + y 2 = 3|x − y| + 3|x + y|

is m + nπ, where m and n are integers. What is m + n?


(A)18 (B)27 (C)36 (D)45 (E)54

20 In how many ways can the sequence 1, 2, 3, 4, 5 be arranged so that no three consecutive terms
are increasing and no three consecutive terms are decreasing?
(A) 10 (B) 18 (C) 24 (D) 32 (E) 44

21 Let ABCDEF√be an equiangular hexagon. The lines AB, CD, and EF determine a √ triangle
with area 192 3, and the lines BC, DE, and F A determine a triangle with area 324 3. The

perimeter of hexagon ABCDEF can be expressed as m + n p, where m, n, and p are positive
integers and p is not divisible by the square of any prime. What is m + n + p?

(A)47 (B)52 (C)55 (D)58 (E)63

22 Hiram’s algebra notes are 50 pages long and are printed on 25 sheets of paper; the first sheet
contains pages 1 and 2, the second sheet contains pages 3 and 4, and so on. One day he
leaves his notes on the table before leaving for lunch, and his roommate decides to borrow
some pages from the middle of the notes. When Hiram comes back, he discovers that his
roommate has taken a consecutive set of sheets from the notes and that the average (mean)
of the page numbers on all remaining sheets is exactly 19. How many sheets were borrowed?
(A) 10 (B) 13 (C) 15 (D) 17 (E) 20

23 Frieda the frog begins a sequence of hops on a 3 × 3 grid of squares, moving one square on
each hop and choosing at random the direction of each hop up, down, left, or right. She does
not hop diagonally. When the direction of a hop would take Frieda off the grid, she ”wraps
around” and jumps to the opposite edge. For example if Frieda begins in the center square
and makes two hops ”up”, the first hop would place her in the top row middle square, and the
second hop would cause Frieda to jump to the opposite edge, landing in the bottom row middle

© 2021 AoPS Incorporated 4


AoPS Community 2021 AMC 10 -

square. Suppose Frieda starts from the center square, makes at most four hops at random,
and stops hopping if she lands on a corner square. What is the probability that she reaches a
corner square on one of the four hops?
9 5 3 25 13
(A) 16 (B) 8 (C) 4 (D) 32 (E) 16

24 The interior of a quadrilateral is bounded by the graphs of (x + ay)2 = 4a2 and (ax − y)2 = a2 ,
where a is a positive real number. What is the area of this region in terms of a, valid for all
a > 0?
8a2 4a 8a 8a2 8a
(A) (a+1)2
(B) a+1 (C) a+1 (D) a2 +1
(E) a2 +1

25 How many ways are there to place 3 indistinguishable red chips, 3 indistinguishable blue chips,
and 3 indistinguishable green chips in the squares of a 3 × 3 grid so that no two chips of the
same color are directly adjacent to each other, either vertically or horizontally?
(A) 12 (B) 18 (C) 24 (D) 30 (E) 36

– B

– February 10th, 2021

1 How many integer values satisfy |x| < 3π?


(A) 9 (B) 10 (C) 18 (D) 19 (E) 20

√ q q √
2 What is the value of (3 − 2 3) + (3 + 2 3)2 ?
2

√ √ √
(A) 0 (B) 4 3 − 6 (C) 6 (D) 4 3 (E) 4 3 + 6

3 In an after-school program for juniors and seniors, there is a debate team with an equal number
of students from each class on the team. among the 28 students in the program, 25
(A) 5 (B) 6 (C) 8 (D) 11 (E) 20.

4 At a math contest, 57 students are wearing blue shirts, and another 75 students are wearing
yellow shirts. The 132 students are assigned into 66 pairs. In exactly 23 of these pairs, both
students are wearing blue shirts. In how many pairs are both students wearing yellow shirts?
(A) 23 (B) 32 (C) 37 (D) 41 (E) 64

5 The ages of Jonie’s four cousins are distinct single-digit positive integers. Two of the cousins’
ages multiplied together give 24, while the other two multiply to 30. What is the sum of the
ages of Jonie’s four cousins?
(A) 21 (B) 22 (C) 23 (D) 24 (E) 25

© 2021 AoPS Incorporated 5


AoPS Community 2021 AMC 10 -

6 Ms. Blackwell gives an exam to two classes. The mean of the scores of the students in the
morning class is 84, and the afternoon classs mean score is 70. The ratio of the number of
students in the morning class to the number of students in the afternoon class is 34 . What is
the mean of the scores of all the students?
(A) 74 (B) 75 (C) 76 (D) 77 (E) 78

7 In a plane, four circles with radii 1, 3, 5, and 7 are tangent to line l at the same point A, but they
may be on either side of l. Region S consists of all the points that lie inside exactly one of the
four circles. What is the maximum possible area of region S?
(A) 24π (B) 32π (C) 64π (D) 65π (E) 84π

8 Mr. Zhou places all the integers from 1 to 225 into a 15 by 15 grid. He places 1 in the middle
square (eight row and eight column) and places the other numbers one by one clockwise, as
shown in part in the diagram below. What is the sum of the greatest and the least number that
appear in the second row from the top?

... ... ... ... ... ... ...


. . . 21 22 23 24 25 . . .
. . . 20 7 8 9 10 . . .
. . . 19 6 1 2 11 . . .
. . . 18 5 4 3 12 . . .
. . . 17 16 15 14 13 . . .
... ... ... ... ... ... ...

(A) 367 (B) 368 (C) 369 (D) 379 (E) 380

9 The point P (a, b) in the xy-plane is first rotated counterclockwise by 90◦ around the point (1, 5)
and then reflected about the line y = −x. The image of P after these two transformations is
at (−6, 3). What is b − a?
(A) 1 (B) 3 (C) 5 (D) 7 (E) 9

10 An inverted cone with base radius 12 cm and height 18 cm is full of water. The water is poured
into a tall cylinder whose horizontal base has a radius of 24 cm. What is the height in centime-
ters of the water in the cylinder?

© 2021 AoPS Incorporated 6


AoPS Community 2021 AMC 10 -

(A) 1.5 (B) 3 (C) 4 (D) 4.5 (E) 6

11 Grandma has just finished baking a large rectangular pan of brownies. She is planning to make
rectangular pieces of equal size and shape, with straight cuts parallel to the sides of the pan.
Each cut must be made entirely across the pan. Grandma wants to make the same number of
interior pieces as pieces along the perimeter of the pan. What is the greatest possible number
of brownies she can produce?
(A) 24 (B) 30 (C) 48 (D) 60 (E) 64

12 Let N = 34 · 34 · 63 · 270. What is the ratio of the sum of the odd divisors of N to the sum of
the even divisors of N ?
(A) 1 : 16 (B) 1 : 15 (C) 1 : 14 (D) 1 : 8 (E) 1 : 3

13 Let n be a positive integer and d be a digit such that the value of the numeral 32d in base n
equals 263, and the value of the numeral 324 in base n equals the value of the numeral 11d1
in base six. What is n + d? (A) 10 (B) 11 (C) 13 (D) 15 (E) 16

14 Three equally spaced parallel lines intersect a circle, creating three chords of lengths 38, 38,
and 34. What is the distance between two adjacent parallel lines?
(A) 5 21 (B) 6 (C) 6 12 (D) 7 (E) 7 12

15 The real number x satisfies the equation x + 1
x = 5. What is the value of x11 − 7x7 + x3 ?
(A) − 1 (B) 0 (C) 1 (D) 2 (E) 8

16 16. Call a positive integer an uphill integer if every digit is strictly greater than the previous
digit. For example, 1357, 89, and 5 are all uphill integers, but 32, 1240, and 466 are not. How many
uphill integers are divisible by 15? (A) 4 (B) 5 (C) 6 (D) 7 (E) 8

17 17. Ravon, Oscar, Aditi, Tyrone, and Kim play a card game. Each person is given 2 cards out of
a set of 10 cards numbered 1, 2, 3, . . . , 10. The score of a player is the sum of the numbers of
their cards. The scores of the players are as follows: Ravon–11, Oscar–4, Aditi–7, Tyrone–16,
Kim–17. Which of the following statements is true?
(A) Ravon was given card 3.
(B) Aditi was given card 3.
(C) Ravon was given card 4.
(D) Aditi was given card 4.
(E) Tyrone was given card 7.

© 2021 AoPS Incorporated 7


AoPS Community 2021 AMC 10 -

18 18. A fair 6-sided die is repeatedly rolled until an odd number appears. What is the probability
that every even
number appears at least once before the first occurrence of an odd number?
1 1 1 3 1
(A) 120 (B) 32 (C) 20 (D) 20 (E) 6

19 Suppose that S is a finite set of positive integers. If the greatest integer in S is removed from
S, then the average value (arithmetic mean) of the integers remaining is 32. If the least integer
is S is also removed, then the average value of the integers remaining is 35. If the greatest
integer is then returned to the set, the average value of the integers rises to 40. The greatest
integer in the original set S is 72 greater than the least integer in S. What is the average value
of all the integers in the set S?
(A) 36.2 (B) 36.4 (C) 36.6 (D) 36.8 (E) 37

20 The figure below is constructed from 11 line segments, each of which has length 2. The area
√ √
of pentagon ABCDE can be written as m + n, where m and n are positive integers. What
is m + n?
https://services.artofproblemsolving.com/download.php?id=YXR0YWNobWVudHMvNS8wLzE2ODI5Njhi
=\&rn=MjAyMSBBTUMgMTJCIDE1LnBuZw==
(A) 20 (B) 21 (C) 22 (D) 23 (E) 24
Proposed by djmathman

21 A square piece of paper has side length 1 and vertices A, B, C, and D in that order. As shown
in the figure, the paper is folded so that vertex C meets edge AD at point C, and edge BC
intersects edge AB at point E. Suppose that CD = 13 . What is the perimeter of 4AEC?

A C’ D

B C

2
√ 13 3
√ 7
(A) 2 (B) 1 + 3 3 (C) 6 (D) 1 + 4 3 (E) 3

© 2021 AoPS Incorporated 8


AoPS Community 2021 AMC 10 -

22 Ang, Ben, and Jasmin each have 5 blocks, colored red, blue, yellow, white, and green; and there
are 5 empty boxes. Each of the people randomly and independently of the other two people
places one of their blocks into each box. The probability that at least one box receives 3 blocks
all of the same color is m
n , where m and n are relatively prime positive integers. What is m + n?

(A) 47 (B) 94 (C) 227 (D) 471 (E) 542

23 A square with side length 8 is colored white except for 4 black isosceles right triangular regions

with legs of length 2 in each corner of the square and a black diamond with side length 2 2 in
the center of the square, as shown in the diagram. A circular coin with diameter 1 is dropped
onto the square and lands in a random location where the coin is completely contained within
the square, The probability
√ that the coin will cover part of the black region of the square can
be written as 196
1
(a + b 2 + π), where a and b are positive integers. What is a + b?

(A) 64 (B) 66 (C) 68 (D) 70 (E) 72

24 Arjun and Beth play a game in which they take turns removing one brick or two adjacent bricks
from one ”wall” among a set of several walls of bricks, with gaps possibly creating new walls.
The walls are one brick tall. For example, a set of walls of sizes 4 and 2 can be changed into any
of the following by one move: (3, 2), (2, 1, 2), (4), (4, 1), (2, 2), or (1, 1, 2). https://services.
artofproblemsolving.com/download.php?id=YXR0YWNobWVudHMvMy8zLzRmZWJlYWJkNDRjN2RmMDdjNGJiN
=\&rn=MjAyMSBBTUMgMTJCIDIyLnBuZw== Arjun plays first, and the player who removes the last
brick wins. For which starting configuration is there a strategy that guarantees a win for Beth?
(A) (6, 1, 1) (B) (6, 2, 1) (C) (6, 2, 2) (D) (6, 3, 1) (E) (6, 3, 2)

25 Let S be the set of lattice points in the coordinate plane, both of whose coordinates are integers
between 1 and 30, inclusive. Exactly 300 points in S lie on or below a line with equation y = mx.
The possible values of m lie in an interval of length ab , where a and b are relatively prime positive
integers. What is a + b?

© 2021 AoPS Incorporated 9


AoPS Community 2021 AMC 10 -

(A) 31 (B) 47 (C) 62 (D) 72 (E) 85


These problems are copyright © Mathematical Association of America (http://maa.org).

© 2021 AoPS Incorporated 10


Art of Problem Solving is an ACS WASC Accredited School.

You might also like